Download as pdf or txt
Download as pdf or txt
You are on page 1of 50

Integral Calculus 199

10
Integral Calculus
dx  2 x + 1
TOPIC 1 Ans. (3) 03 If ∫ = a tan − 1  
sin x (x + x + 1)
2  3 
2
Indefinite Integrals Let I = ∫ dx
sin x + cos3 x
3
 2x + 1 
1 tan x sec2 x + b 2  + C, x > 0where C is

01 The integral ∫ dx = dx  x + x + 1
tan3 x + 1
4 (x − 1) (x + 2)
3 5
the constant of integration, then
Put tan x = t
is equal to (where C is a constant of the value of 9 ( 3 a + b) is equal to
⇒ sec2 x dx = dt
integration) [2021, 31 Aug. Shift-I]
t dt t
[2021, 27 Aug. Shift-I]
I=∫ 3
t + 1 ∫ (t + 1)(t2 − t + 1)
1/ 4 5/ 4 = dt
3  x + 2 3  x + 2 Ans. (15)
(a)   + C (b)   +C
4  x −1  4  x −1  dx dx
Now,
t
=
A
+
Bt + C ∫ (x2 + x + 1)2 = ∫  2
  
1/ 4 5/ 4 2
4  x −1  4  x −1  (t + 1)(t2 − t + 1) t + 1 t2 − t + 1 2
(c)   + C (d)   +C   x + 1  +  3  
3  x + 2 3  x + 2 ⇒ t = A (2 − t + 1) + (Bt + C)(t + 1)  2  2  
 
 
Ans. (c) Comparing coefficients to both the sides
1 3
1 and solving them for A, B, C, we have Let x + = tanθ
∫ (x − 1) 3/ 4 (x + 2) 5 / 4 dx 1 1
A = − , B = ,C =
1 2 2
3 3 3 3
=∫
dx ⇒ dx = sec2 θ dθ
1 1 1 t+1
5/ 4 Hence, I = − ∫ dt + ∫ 2 dt 2
 x + 2 3 t+1 3 t −t + 1
  (x − 1)2 3
 x − 1 sec2 θ dθ
1 3 2
(2t − 1) +
x+2
=t 1 1 2
= − ln (t + 1 ) + ∫ 2 2 dt
∴ ∫ 9
(tan2 θ + 1 )2
x−1 3 3 t −t + 1 16
 (x − 1) − (x + 2)  1 1 8 sec2 θ dθ
⇒ 
( x − 1) 2
 dx = dt = − ln (t + 1) + ln (t2 − t + 1)
3 6
=
3 3
∫ sec4 θ
 
1 dt 8
+ ∫ = ∫ cos θ dθ
3 2
⇒ − dx = dt 2 2
(x − 1)2  1
2
 3 3 3
 t −  +   8 1 + cos 2θ
 2

1 dt
− ∫
4 1
= . +C
 2  =
3 3
∫ 2

3 t 5 / 4 3 t 1/ 4 1 1
= − ln (t + 1) + ln (t − t + 1)
2
1/ 4 3 6 4  sin 2θ 
4 x − 1 = θ +  +C
=   +C 3 3 2 
3  x + 2 1 2  2t − 1 
+ ⋅ tan−1  2x + 1
 +C
sin x 2 3  3  4 − 1  2x + 1 4
02 If ∫ dx = αlog e | = tan 

+
3 +C
sin 3 x + cos 3 x 1 1 2 3 3  3 3  2 x + 1
2
= − ln (tan x + 1) + ln (tan2 x − tan x + 1) 1+ 
3 6  3 
1 + tan x | + β log e | 1 − tan x + tan 2 x | 1  2 tan x − 1 
tan−1  2x + 1  1 2x + 1
+  +C tan− 1 
4
 2 tan x − 1   = + +C
+ γ tan − 1   + C, when C is 3 3 3 3 
3  3 (x2 + x + 1 )
 3  −1
⇒ α = ,β = , γ =
1 1 4 1
3 6 3 ∴ a= ,b =
constant of integration, then the 3 3 3
value of 18 (α + β + γ 2 ) is  −1 1 1 
So, 18(α + β + γ ) = 18  + +  = 3
2
Hence, 9( 3a + b) = 9 +  = 15
4 1
[2021, 31 Aug. Shift-II]  3 6 3  3 3
⇒ e x f (x) = x 3 + x2 + 4f (x) − 36
2e x + 3 e − x 1 07 The integral
04 If ∫ −x
dx = (ux + v log e On putting x = 4,
(2x − 1)cos (2x − 1) 2 + 5
4e + 7 ex
14 e 4f (4) = 44 + 4f (4) ∫ dx is equal
(4e + 7 e − x )) + C, where C is a
x  44  4x 2 − 4x + 6
⇒ f (4) =  4 
constant of integration, then u + v  e − 4 to (where, c is a constant of
is equal to [2021, 27 Aug. Shift-II] On differentiating w.r.t. x, we get integration) [2021, 18 March Shift-I]
Ans. (7) e x [f (x) + f ′ (x)] = 3x2 + 2x + 4f ′ (x) 1
−x
(a) sin (2 x − 1)2 + 5 + c
2e + 3e x
2e + 3 2x
f ′ (x) (e − 4) = 3x + 2x − e f (x)
x 2 x 2
I=∫ x dx = ∫ 2 x dx
4e + 7 e − x 4e + 7  44  1
⇒ f ′ (4) (e 4 − 4) = 48 + 8 − e 4  4  (b) cos (2 x + 1)2 + 5 + c
d 2
Let 2e2 x + 3 = A (4e2 x + 7)  e − 4
dx 1
56(e 4 − 4) − 44e 4 (c) cos (2 x − 1)2 + 5 + c
+ B (4e2 x + 7 ) ⇒ f ′ (4) (e 4 − 4) = 2
⇒ 2e2 x + 3 = (8A + 4B) e2 x + 7 B e4 − 4 1
(d) sin (2 x + 1)2 + 5 + c
Comparing both sides 12e 4 − 224 2
f ′ (4) =
3 1 (e 4 − 4)2
B = and A = Ans. (a)
7 28
According to the question, (2x − 1) cos (2x − 1)2 + 5
1 3
(8e2 x ) + (4e2 x + 7 ) Let I = ∫ dx
α = 12, β = 4
4x2 − 4x + 6
∴ I = ∫ 28 7 dx So, α + β = 16
4e2 x + 7 (2x − 1) cos (2x − 1)2 + 5
π β − sin2 x =∫ dx
1
= ln|4e2 x + 7 | + x + C
3 06 If ∫ (sin x) e dx = α −
3
(2x − 1)2 + 5
28 7 0 e
1 (2x − 1)2 + 5 = z2
∫ t e dt, then (α + β) is equal to
1 3 t Putting
= ln| e (4e + 7 e − x )| + x + C
x x
28 7
0 ⇒ 2(2x − 1) × 2⋅dx = 2z dz
………… . [2021, 27 July Shift-II] 1
=
1 1 −x
x + ln |4e + 7 e | + x + C
x 3 ⇒ (2x − 1) dx = z dz
28 28 7 Ans. (5) 2
π 2 cos z 1
1  13 1 −x  I = ∫ (sin3 x) e –sin x
dx ∴ I=∫ ⋅ z ⋅dz
=  x + ln|4e + 7 e | + C
x
0 z 2
14  2 2 
1 1
13 1 π 2 = ∫ cos z dz = sin z + c
⇒ u = and v = = ∫ (1 − cos2 x) sin x e − (1 − cos x)
dx 2 2
2 2 0
1
13 1 If f (x) = f (2a − x), then = sin (2x − 1)2 + 5 + c
∴ u+v= + =7 2a a
2
2 2
∫0 f (x) dx = 2∫ f (x) dx
0 Note You can also substitute
π /2 − (1 − cos 2 x )
05 Let f : [3, 5] → R be a twice I = 2∫
0
(1 − cos x) sin x e
2
dx (2x − 1)2 + 5 = z and then proceed.
differentiable function on (3, 5) Let cos2 x = t ⇒− 2cos x sin x dx = dt 5x 8 + 7 x 6
such that dt 08 If f (x) = ∫ dx, (x ≥ 0),
x ⇒ sinx dx = − (x 2 + 1 + 2x 7 ) 2
2 t
f (x) = e − x ∫ [3t 2 + 2t + 4f ′ (t)] dt. 1
3
1
= − 2∫ (1 − t) e t − 1
dt f (0) = 0 and f (1) = , then the value of
0 −2 t K
αeβ − 224 1 1 (1 − t) e t K is ………… . [2021, 18 March Shift-I]
If f ′ (4) = , then α + β is = ∫ dt
(eβ − 4) 2 e 0 t Ans. (4)
1 1 t 5x 8 + 7 x 6
e dt − ∫ t e t dt 
1 1

e ∫0 t
equal to ……… .[2021, 27 July Shift-I] =  Let I = ∫ dx
Ans. (16)
0  (x + 1 + 2x 7 )2
2

x 1 1 1
 5x 8 + 7 x 6
f (x) = e − x = (2 t e ) 0 − 2∫0 t e dt − ∫0 t e dt 
t 1 t t
=∫ dx
∫ [3t + 2t + 4f ′ (t)] dt
2
e  x 14 (x −5 + x −7 + 2)2
3
1 1 t  5x 8 + 7 x 6
f (x) = e − x [t 3 + t2 + 4f (x)] x3 = 2e − 3 ∫ t e dt 
e 0  x 14
=∫ dx
f (x) = e − x [(x 3 − 27) + (x2 − 9) 3 1 14 −5
+ x −7 + 2)2
= 2 − ∫ t e dt t x (x
+ 4 [f (x) − f (3)]] e 0
x 14
β 1
Now, f (3) = e −3 On comparing, α − ∫ t e t dt 5 x −6 + 7 x −8
3 e 0 ⇒ I=∫ dx
(x −5 + x −7 + 2)2
∫3 3t + 2t + 4 f ′ (t) dt = 0
2
⇒ α = 2 and β = 3
⇒ f (x) = e − x (x 3 + x2 + 4f (x) − 36) Putting x −5 + x −7 + 2 = z
∴ α + β =2+ 3= 5
⇒ − (5x −6 + 7 x −8 )dx = dz  x2 + 1  Ans. (b)
Let tan−1   = t
I = − ∫ 2 = −   + c
dz 1  x  e 3log e (2 x ) + 5e2 log e (2 x )
∴ I=∫ dx
z  −z 
x2 − 1 e 4 log e (x ) + 5e 3log e (x ) − 7 e2 log e (x )
1 ⇒ dx = dt
⇒ I= +c (x 4 + 3x2 + 1)
3 2

x + x −7 + 2
−5 e log e (2 x ) + 5e2 log e (2 x )
=∫ dx
dt
I1 = ∫
4 3 2
x7 ∴ = log|t | + C 1 e log ex + 5e log e (x ) − 7 e log e (x )
⇒ f (x) = +c t
x2 + 1 + 2x 7  x2 + 1  [using property a log x = log x a ]
= log tan−1   + C 1 8x + 5(2x)2
3
Given, f(0) = 0 ⇒ c = 0  x  =∫ 4 dx
x7 x + 5(x) 3 − 7 x2
∴ f (x ) = 2 1 1 (x2 + 1) − (x2 − 1)
x + 1 + 2x 7 I2 = ∫
2 ∫ x 4 + 3x2 + 1
log a x
dx = dx [using property a = x]
1 1 1 x 4 + 3x2 + 1
∴ f(1) = = = (given) 8x + 20 x
3 2
1+ 1+ 2 4 K 1 x2 + 1 1 x2 − 1 =∫ dx
= ∫
2 x + 3x + 1
4 2
dx − ∫ 4
2 x + 3x2 + 1
dx x 4 + 5x 3 − 7 x2
Hence,K = 4.
1 1 + 1 / x2 1 1 − 1 / x2 4x2 (2x + 5)
= ∫ 4 dx − ∫ 2 dx =∫ dx
09 For real numbers α, β, γ and δ, if 2 x + 3+ 1/x 2
2 x + 3 + 1 / x2 x (x2 + 5x − 7)
2

1 1 1 + 1 / x2 4(2x + 5)
 x 2 + 1 I2 = = ∫ dx =∫ dx
(x 2 − 1) + tan −1  

2 2  1
2
x2 + 5x + 7
 x  x −  + 5
 x
∫  x 2 + 1
dx 1
− ∫
1 − 1 / x2
dx
Let x2 + 5x − 7 = t, then (2x + 5)dx = dt
4dt
(x 4 + 3x 2 + 1) tan −1   I=∫
2
2  1 = 4log n t + c
 x +  + 1 t
 x   x
Put t = x2 + 5x − 7
1 x2 − 1  1
−1   x2 + 1 
  x 2 + 1  = tan   − tan−1   + C2
I = 4log e | x2 + 5x − 7 | + c
= α log e tan −1  

2 5  5x  2  x 
  x  
 x2 + 1  1 11 The value of the integral
I = log tan− 1   + tan− 1
 γ (x 2 – 1)   x 2 + 1  x  2 5 sinθ ⋅ sin2θ (sin6 θ + sin4 θ + sin2 θ) 
+ β tan −1   + δ tan −1 
  x  +C
  
 x     x2 + 1  1  x2 + 1   
  − tan− 1   + C 4 2
2 sin θ + 3 sin θ + 6 

where C is an arbitrary constant, then  5x  2  x  ∫  1 − cos 2θ


the value of 10(α + βγ + δ) is equal   x2 + 1    
to……… . = α log e  tan−1    + β tan−1  
[2021, 16 March Shift-II]   x  
  
Ans. (6)
 γ (x2 − 1)   x2 + 1 
− 1
x2 + 1   + δ tan−1   + C (given) is (where, c is a constant of integration)
(x − 1) + tan 
2
  x   x  [2021, 25 Feb. Shift-I]
 x  3
Let I=∫ dx 1 1 1 1
x2 + 1 
− 1
∴ α = 1, β = γ= and δ = − (a) [11 − 18 sin2θ + 9 sin4θ − 2 sin6 θ] 2 + c
(x + 3x + 1) tan 
4 2
 2 5, 5 2 18
 x  3
∴Required value of 10(α + βγ + δ) 1
x2 − 1 (b) [9 − 2 cos6 θ 3 cos4 θ − 6 cos2 θ] 2 + c
⇒ I=∫  1 1  10 + 1 − 5 
dx = 10  1 + −  = 10   18
x2 + 1 
−1   10 2   10  3
(x + 3x + 1) tan 
4 2
 1
 x  =6 (c) [9 − 2 sin6θ − 3 sin4θ − 6 sin2θ ] 2 + c
18
1
+∫ 4 dx 3
x + 3x2 + 1 10 The integral 1
(d) [11 − 18 cos2 θ + 9 cos4 θ − 2 cos6 θ] 2 + c
Again let e 3log e 2x + 5e 2log e 2x 18

I1 = ∫
x2 − 1
dx ∫ e 4log e x + 5e 3log e x − 7 e 2log e x dx, Ans. (d)
 x2 + 1  Let
(x 4 + 3x2 + 1) tan−1  
 x  x > 0, is equal to (where, c is a  sinθ⋅ sin2θ( sin6 θ + sin4 θ + sin2 θ) 
constant of integration)  
and I2 = ∫
dx  2 sin4 θ + 3 sin2 θ + 6 
[2021, 25 Feb. Shift-II] I =∫  dθ
x 4 + 3x2 + 1 1 − cos2θ
(a) log e | x2 + 5 x − 7 | + c  
∴ I = I 1 + I2 …(i)  
(b) 4 log e | x2 + 5 x − 7 | + c
 
(x − 1)
2
1
Now, I 1 = ∫ dx (c) log e | x2 + 5 x − 7 | + c Q sin2A = 2 sin A cos A
 x2 + 1  4
(x + 3x + 1) tan−1 
4 2
 and 1 − cos2A = 2 sin2 A
 x  (d) log e x2 + 5 x − 7 + c
sinθ⋅2 sinθ (sin6 θ + sin4 θ + sin2 θ) 2
14 The integral ∫   dx
Now, according to question, x
cos x − sin x 
2 sin4 θ + 3 sin2 θ + 6 ∫ 8 − sin2x dx  x sin x + cos x 
I=∫ dθ
2 sin2 θ sin x + cos x 
is equal to (where C is a constant of
I = ∫ cosθ (sin θ + sin4 θ + sin2 θ)
6 = a sin−1  +c integration) [2020, 4 Sep. Shift-I]
 b 
x tan x
2 sin4 θ + 3 sin2 θ + 6dθ  t  (sin x + cos x) (a) sec x − +C
⇒ sin− 1   + c = a sin− 1 +c x sin x + cos x
 3 b
= ∫ (t 6 + t 4 + t2 ) 2t 4 + 3t2 + 6 dt x tan x
sin x + cos x  (b) sec x + +C
⇒ 1 sin− 1  +c x sin x + cos x
= ∫ (t 5 + t 3 + t) 2t 6 + 3t 4 + 6t2 dt  3 
x sec x
− 1  sin x + cos x 
= a sin 
(c) tan x − +C
Let 2t 6 + 3t 4 + 6t2 = z +c x sin x + cos x
 b 
∴ dz = (12t 5 + 12t 3 + 12t) dt x sec x
(Qt = sin x + cos x ) (d) tan x + +C
∴ dz = 12(t 5 + t 3 + t) dt ∴ a=1 x sin x + cos x
b =3 Ans. (c)
1 1 z 3/ 2 Hence, (a, b) = (1, 3)
Now,
12 ∫ z dz = ×
12 3 /2
+c
Given integral I = ∫
x2
dx
 x 
−1 (x sin x + cos x)2
=
1 3/ 2
z +c 13 If ∫ sin   dx = A(x) tan −1 ( x )
18  1+ x  Q
d
(x sin x + cos x)
1 dx
= [2t 6 + 3t 4 + 6t2 ] 3/ 2 + c +B(x) + C, where C is a constant of
18
integration, then the ordered pair = x cos x + sin x − sin x = x cos x
1 x cos x
= [2 sin6 θ + 3 sin4 θ + 6 sin2 θ] 3/ 2 + c (A(x), B(x)) can be ∴ I = ∫ (x sec x) dx
18
[2020, 3 Sep. Shift-II] (x sin x + cos x)2
1
= [(1 − cos2 θ) {2(1 − cos2 θ)3 + 3 (a) (x − 1, x ) By integration by parts, we have
18 x cos x
(b) (x + 1, x ) I = x sec x ∫ dx
− 3 cos2 θ + 6}] 3/ 2 + c (x sin x + cos x)2
1 (c) (x + 1, − x )
= [(1 − cos2 θ) (2 cos4 θ (d) (x − 1, − x ) d x cos x 
18 − ∫  (x sec x) ∫ dx  dx
− 7 cos2 θ + 11) ] 3/ 2 + c (x sin x + cos x) 2
Ans. (c)  dx 
1
= [− 2 cos6 θ + 9 cos4 θ − 18 cos2 θ + 11] 3/ 2 The integral  1 
18 +c = x sec x  − 
1 I = ∫ sin−1
x
dx  (x sin x + cos x) 
= [11 − 18 cos2 θ + 9 cos4 θ − 2 cos6 θ] 3/ 2 1+ x (x sec x tan x + sec x)
18
+c +∫ dx
Let x = tan2 θ ⇒ dx = 2 tanθ sec2 θdθ, (x sin x + cos x)
cos x − sin x
12 If ∫ dx  tan2 θ 
x sec x (x sin x + cos x)
x sin x + cos x ∫ (x sin x + cos x)
=− + sec2 xdx
8 − sin 2x So, I = ∫  sin−1 2 tanθ sec2 θdθ
 1 + tan2 θ 
 x sec x
 sin x + cos x  = tan x − +C
= a sin − 1   + c, where   tanθ   x sin x + cos x
 b  = 2∫  sin−1    tanθ sec θdθ
2
  secθ   Hence, option (c) is correct.
c is a constant of integration, then −1
= 2 ∫ ( sin (sinθ)) tanθ sec θdθ 2
x
the ordered pair (a, b) is equal to 15 Let f (x) = ∫ dx (x ≥ 0). Then
[2021, 24 Feb. Shift-I] = 2∫ θ tanθ sec θdθ 2
(1 + x) 2
(a) (3, 1) (b) (1, 3) By integration by parts f (3) − f (1) is equal to
(c) (− 1, 3) (d) (1, − 3) I = 2 [θ∫ tan θ sec2 θdθ [2020, 4 Sep. Shift-I]
Ans. (b) d   π 1 3 π 1 3
cos x − sin x − ∫  θ∫ tan θ sec2 θdθdθ (a) − + + (b) − + +
Given, ∫ dx dθ   6 2 4 12 2 4
8 − sin2x π 1 3 π 1 3
 tan2 θ tan2 θ  (c) + − (d) + −
sin x + cos x  = 2 θ −∫ dθ
= a sin− 1  +c  2 2 
6 2 4 12 2 4
 b 
Put, sin x + cos x = t Ans. (d)
= θ tan2 θ − ∫ ( sec2 θ − 1)dθ
Also, sin2 x + cos2 x + 2 sin x cos x = t2 It is given that,
= θ tan2 θ − tanθ + θ + C
⇒ 2 sin x cos x = t2 − 1 x
= x tan −1
x − x + tan −1
x +C f (x) = ∫ dx, (x ≥ 0)
⇒ sin2x = (t2 − 1) (1 + x)2
−1
= (x + 1) tan x − x +C
and (cos x − sin x) dx = dt Put x = tan2 t ⇒dx = 2 tant sec2 tdt
cos x − sin x dx = A (x) tan−1 x + B (x) + C [given] 2 tan2 t sec2 t
Now, ∫ dx = ∫ ∴ f (x) = ∫ dt = ∫ 2 sin2 tdt
8 − sin2x So, (A (x), B (x)) = (x + 1, − x )
8 − (t2 − 1) sec4 t
Hence, option (c) is correct.
= ∫ (1 − cos2t)dt
= sin− 1   + c
dt t
=∫
9 − t2  3 [Q2 sin2 x = 1 − cos2x]
=t −
sin2t
+C 2 sinθ + 1 2 sinθ + 1 1 t2dt 1
2 ∫ t2
(a) (b) So, I=− = − ∫ dt
2 sinθ + 3 5 (sinθ + 3) 2
tant 5 (sinθ + 3) 5 (2 sinθ + 1)
So, = t − +C 1 1 1 
1/ 3
1 + tan2 t (c) (d) = − t + C = −  6 + 1 + C
2 sinθ + 1 sinθ + 3 2 2  sin x 
x
= tan−1 x − +C Ans. (d) 1
1+ x I = − cosec x (1 + sin x) + C ...(ii)
2 6 1/ 3
cosθ 2
 3  Since, I = ∫ dθ
∴ f (3) − f (1) =  tan−1 3 − + C  5 + 7 sinθ –2 cos2 θ On comparing Eqs. (i) and (ii), we get
 1+ 3  cosθ 1
=∫ dθ f (x) = − cosec2 x and λ = 3
 1  3 + 7 sinθ + 2 sin2 θ 2
−  tan−1 (1) − + C
π  1 π 
 1+ 1  Let, sinθ = t ⇒cos θ dθ = dt ∴ λf   = 3  − cosec2   
π 3 π 1 dt 1 dt 3  2  3 
= − − + ∴ I=∫ 2 = ∫
3 4 4 2 2t + 7t + 3 2 7 3
t + t+
2 3 4
= − × = −2
π 1 3 2 2 2 3
+ − = 1 dt
12 2 4 = ∫ Hence, option (c) is correct.
Hence, option (d) is correct. 2  7  2  3 49 
t +  +  –  dx
 4   2 16  19 The integral ∫ is
−x (e x + e −x ) (x + 4) (x − 3) 6/ 7 8/ 7
16 If ∫ (e 2x
+ 2e − e x
− 1)e dx 1
= ∫
dt
x −x 2  7 2  5 2 equal to (where C is a constant of
= g (x) e ( e + e ) + c, where c is a t +  –  
 4  4 integration) [2020, 9 Jan Shift-I]
−1/ 7
constant of integration, then g(0) is 7 5  x − 3
t+ – (a) −   +C
equal to [2020, 5 Sep. Shift-I] 1
= ×
1
log e 4 4 + C'  x + 4
(a) 2 (b) e 2 2 2× 5 t+ +
7 5 3/ 7
1  x − 3
(c) e (d) 1 4 4 4 (b)   +C
1 2t + 1 2  x + 4
Ans. (a) = log e + C' 1/ 7
5 2t + 6  x − 3
Given, integral (c)   +C
−x (e x + e − x ) 1 2 sinθ + 1  x + 4
I = ∫ (e 2x
+ 2e − e
x
− 1) e dx = log e + C, −13/ 7
5 sinθ + 3 1  x − 3
1 (d) −   +C
Let e x = t ⇒dx =
dt
where C = C ′– log e 2 13  x + 4 
t 5
 1 = A log e | B (θ)|+ C [given] Ans. (c)
 t +  dt
So, I = ∫  t2 + 2t − − 1 e  t 
1 The given integral,
  B (θ) 5(2 sinθ + 1)
t t ∴ = . dx
 t + 1 A (sinθ + 3) I=∫
  (x + 4) 8 / 7 (x − 3) 6 / 7
= ∫  t + 2 − 2 −  e
1 1  t
dt cos x dx
 t
t 18 If ∫ dx
 1
  t+  sin x (1+ sin 6 ) x) 2/ 3
3
(x + 4)2
= ∫   t −  +  1 − 2  + 1 e
1 1  t =∫
dt 1/ λ
  t   t   = f (x) (1 + sin x) + C
6 (x + 4) 8 / 7 (x − 3) 6 / 7
 1
t+  where c is a constant of (x + 4)2
 
= ∫  (1 + t)  1 − 2  + 1  e
1  t
dt integration, then is equal to dx
  t  
t+ 1 t+ 1 [2020, 8 Jan. Shift-I] (x + 4)2



t



t 9 =∫
= ∫ (1 + t)(1 − 1 /t2 ) e dt + ∫ e dt (a) 2 (b) −  x −3
6/ 7
8  
 t + 1
 
 t + 1
 
 t + 1
  9  x + 4
= (1 + t) e  t
− ∫ 1⋅ e  t
dt + ∫ e  t
dt (c) − 2 (d)
8 x −3 7
Now, let =t
t+
1
Ans. (c) x+4
x
+ e −x )
= (1 + t) e t
+ c = (1 + e x ) e (e +c
cos x (x + 4) − (x − 3)
x
+ e −x ) Given that ∫ dx ⇒ dx = 7t 6dt
= g (x) e (e +c [given] sin3 x (1 + sin6 x)2 / 3 (x + 4)2
∴ g (x) = 1 + e x = f (x)(1 + sin6 x) 1/ λ + C …(i) ⇒
7
dx = 7t 6dt
⇒ g(0) = 1 + 1 = 2 cos x (x + 4)2
Since, I = ∫ dx
cosθ sin3 x (1 + sin6 x)2 / 3 t 6dt
17 If ∫ dθ So, I=∫ = ∫ dt = t + C
I=∫
cos x t6
5 + 7 sinθ − 2cos 2 θ 2/ 3
dx
1/ 7
4  1   x −3
= Alog e | B(θ)| + C, where C is a sin x sin x  6 + 1
3
∴ I=  +C
 sin x   x + 4
B(θ)
constant of integration, then 1 −6 cos x
A Let + 1 = t then
3
dx = 3t2dt Hence, option (c) is correct.
sin6 x sin7 x
can be [2020, 5 Sep. Shift-II]
20 If for all real triplets (a, b,c), 1 −t  2  =∫
dx
Then, I=∫ dt = ∫  −1 + dt 2 /3
1+t 1+t x 7  6 + 1
1  1
f (x) = a + bx + cx 2 ; then ∫ f (x)dx is x 
0 = − t + 2log e |1 + t | + C
equal to 1
[2020, 9 Jan. Shift-I] = − tanθ + 2log e |1 + tanθ| + C Now, put 6 + 1 = t 3
  1  = λ tanθ + 2log e |f (θ)| + C (given)
x
(a) 2 3f (1) + 2 f    6
  2  ∴ (λ, f (θ)) = (−1, 1 + tanθ) (on comparing) ⇒ − 7 dx = 3t2dt
x
1  1  Hence, option (c) is correct.
(b) f (0) + f    5x dx t2
3  2  sin ⇒ = − dt
x7 2
1  1  22 ∫ 2 dx is equal to
1
(c) f (1) + 3f    x − t2dt
2 2  sin 2 1
2 So, I = ∫ = − ∫ dt
1   1  t2 2
(d) f (0) + f (1) + 4 f    (where,C is a constant of
 2  1/ 3
= − t + C = −  6 + 1 + C
6 1 1 1
integration) [2019, 8 April Shift-I] 2 2x 
Ans. (d)
(a) 2 x + sin x + 2 sin2 x + C Qt 3 = 1 + 1
It is given thatf (x) = a + bx + cx2 , then (b) x + 2 sin x + 2 sin2 x + C
1 1
 x6 
∫0 f (x) dx = ∫0 (a + bx + cx ) dx (c) x + 2 sin x + sin2 x + C
2
1 1
(d) 2 x + sin x + sin2 x + C = − 2 (1 + x 6 ) 1 / 3 + C
1 2x
 b c 
= ax + x2 + x 3 Ans. (c)
 2 3  0 = x ⋅f (x) ⋅ (1 + x 6 ) 1/ 3 + C [given]
5x 5x x On comparing both sides, we get
b c sin 2 sin cos
=a + + 2 2 2
2 3 Let I = ∫ dx = ∫ dx f (x) = − 3
1
x x x
6a + 3b + 2c sin 2 sin cos 2x
= 2 2 2
6 x
[multiplying by 2 cos in numerator and 24 The integral ∫ sec 2/ 3 x cosec 4/ 3 x dx
Q f (0) = a, f (1) = a + b + c 2
 1 denominator] is equal to (here C is a constant of
and 4f   = 4a + 2b + c sin3x + sin2x integration)
 2 =∫ dx
[2019, 9 April Shift-I]
1 6a + 3b + 2c sin x (a) 3 tan−1/ 3 x + C
∴ ∫ f (x) dx = (b) −3 tan−1/ 3 x + C
0 6 [Q 2sin A cos B = sin(A + B) + sin(A − B)
 1 and sin 2A = 2 sin A cos A] (c) −3cot −1/ 3 x + C
= f (0) + f (1) + 4f   
1
(3 sin x − 4 sin3 x) + 2 sin x cos x 3
6  2  =∫ dx (d) − tan−4 / 3 x + C
sin x 4
Hence, option (d) is correct.
[Qsin3x = 3 sin x − 4 sin3 x] Ans. (b)
dθ 2 4
21 If ∫ = = ∫ (3 − 4 sin2 x + 2 cos x) dx Let I = ∫ sec 3 x cosec 3 x dx
cos θ(tan2θ + sec2θ)
2
= ∫ [3 − 2(1 − cos2x) + 2 cos x] dx
λ tanθ + 2log e | f (θ)| + C where C is a [Q 2 sin2 x = 1 − cos 2x] =∫
dx
= ∫ [3 − 2 + 2 cos2x + 2 cos x] dx 2 4
constant of integration, then the = ∫ [1 + 2 cos2x + 2 cos x] dx cos3 x sin 3 x
ordered pair (λ ,f (θ)) is equal to dx
[2020, 9 Jan. Shift-II]
= x + 2 sin x + sin 2x + C
1
∫ 4
4 2
dx  sin x  3
(a) (1, 1 + tanθ) (b) (1, 1 − tanθ) 23 If ∫ = xf (x)(1 + x 6 ) 3 +C   cos 3 x cos3 x
 cos x 
(c) (−1, 1 + tanθ) (d) (−1, 1 − tanθ) x 3 (1 + x 6 ) 2/ 3
[dividing and multiplying by cos4 / 3 x
Ans. (c) where,C is a constant of integration, in denominator]
dθ then the function f (x) is equal to dx sec2 xdx
Given integral, I = ∫ =∫ = ∫
cos2 θ(tan2θ + sec2θ) [2019, 8 April Shift-II]
4 4

1 1 1 3 tan 3 x cos2 x (tan x) 3


sec θ cos2θ
2
(a) − (b) − (c) − (d)
=∫ dθ Now, put tanx = t ⇒sec x dx = dt 2
(sin2θ + 1) 6x 3 2x 3 2x2 x2 −4
+1
sec2 θ(1 − tan2 θ) Ans. (b) dt t3
=∫ dθ ∴ I = ∫ 4/ 3 = +C
(2 tanθ + 1 + tan2 θ) dx −4
Let I = ∫ t +1
x 3 (1 + x 6 ) 2 / 3 3
sec θ(1 − tanθ)
2
=∫ dθ 1 −3
1 + tanθ =∫
dx = −3 1 + C = 1
+C
2 /3
x 3 ⋅ x 4  6 + 1
Let, tanθ = t 1 t3 (tan x) 3
x 
⇒ sec2 θdθ = dt −
1
= −3 tan 3 x+C
25 If ∫ e sec x So, I = ∫
3 sec2θ dθ
=∫
3 sec2θ dθ Let
r 1
= x and = dx [Q n→ ∞]
(3 tan θ + 3 )
2 2
3 sec θ 2 2 4 4 n n
(sec x tan x f (x) + (sec x tan x + sec 2 x)) Then, upper limit of integral is 1 and
1 1 1 + cos2θ
dx = e sec x f (x) + C, then a possible = ∫ cos θ dθ = ∫
2
dθ lower limit of integral is 0.
27 27 2 1
choice of f (x) is 1 + cos2θ 
 So, p = ∫ (1 + x) 1/ 3 dx
[ 2019, 9 April Shift-II] Qcos2 θ =
 2  0
1  1 n r 
(a) x sec x + tan x + 1 1  sin2θ 
1
Q lim ∑ f   = ∫ f (x) dx 
54 ∫
2 = (1 + cos2θ) dθ = θ +  +C → ∞  
54  2   n n r =1 n 0

1
(b) sec x + tan x + 1
x − 1 1  2 tanθ  3  3 3 3
tan−1 
2 1 =  (1 + x) 4 / 3  = (24 / 3 − 1) = (2) 4 / 3 −
= +   +C
(c) sec x + x tan x −
1 54  3  108  1 + tan2 θ  4 0 4 4 4
2
 2 tanθ  2 2
(d) sec x − tan x −
1
Qsin2θ =  28 If ∫ x 5 e − x dx = g (x) e − x + C, where
2  1 + tan2 θ 
Ans. (b)  x − 1 C is a constant of integration, then
 
=
1 −1  x − 1  1  3  g (− 1) is equal to
Given, tan  + +C
54  3  54 x − 1
2
1 + 
[2019, 10 April Shift-II]
∫ e [(sec x tan x)f (x)
sec x

 3  (a) − 1 (b) 1 (c) −
1
(d) −
5
+ (sec x tan x + sec2 x)]dx
x − 1 1  x−1  2 2
tan−1 
1
=e sec x
⋅f (x) + C = +   +C Ans. (d)
54  3  18  (x − 1)2 + 32  2
On differentiating both sides w.r.t. x, we Let given integral,I = ∫ x 5 e − x dx
x − 1 1  x−1 
tan−1 
1
get = +  2  +C
54  3  18  x − 2x + 10  Put x2 = t ⇒2xdx = dt
e sec x [(sec x tan x)f (x) 1
+ (sec x tan x + sec2 x)] 1  −1  x − 1  3(x − 1)  So, I = ∫ t2 e − t dt
=  tan  + +C 2
=e sec x
f ′ (x) + e sec x (sec x tan x)f (x) 54  3  x2 − 2x + 10 
1
= [(− t2 e − t ) + ∫ e − t (2t) dt]
⇒ e sec x
(sec x tan x + sec2 x) = e sec x f ′ (x) It is given, that 2
⇒ f ′ (x) = sec x tan x + sec2 x  x − 1 
I = A  tan−1 
f (x)
+ 2 +C
[integration by parts]
So, f (x) = ∫ f ′ (x)dx   3  x − 2 x + 10  1
1 = [− t e + 2t (− e − t ) + ∫ 2e − t dt]
2 −t

= ∫ (sec x tan x + sec2 x)dx On comparing, we get A = and 2


54 1
= sec x + tan x + C f (x) = 3(x − 1). = [− t2 e − t − 2te − t − 2e − t ] + C
2
So, possible value off (x) from options, is
f (x) = sec x + tan x + .
1 27 e− t 2
=− (t + 2t + 2) + C
2  (n + 1) 1/ 3 (n + 2) 1/ 3 (2n) 1/ 3  2 2
lim  4/ 3 + + ... +  e− x
dx n→ ∞  n n 4/ 3 n 4/ 3  =− (x 4 + 2x2 + 2) + C
26 If ∫ 2
[Qt = x2 ] …(i)
(x − 2x + 10) 2
2 is equal to
Q It is given that,
[2019, 10 April Shift-I]
  x − 1 f (x)  2
I = ∫ x 5 e − x dx = g (x) ⋅ e − x + C
2

= A  tan −1  + 2  + C, 4
(a) (2) 4 / 3
3 4/ 3 4
(2) −
  3  x − 2x + 10 3
(b)
4 3 By Eq. (i), comparing both sides, we get
where, C is a constant of 3 3 4 3/ 4 1
(c) (2) 4 / 3 − (d) (2) g (x) = − (x 4 + 2x2 + 2)
integration, then 4 4 3 2
[2019, 10 April Shift-I] Ans. (c) 1 5
So, g(− 1) = − (1 + 2 + 2) = −
1 Let 2 2
(a) A = and f (x) = 9 (x − 1)  (n + 1) 1/ 3 (n + 2) 1/ 3 
27  + +…  2x 3 − 1
  29 The integral ∫
4/ 3 4/ 3
1 p = lim n n dx is equal to
(b) A = and f (x) = 3 (x − 1) n→ ∞  (2 n) 1/ 3  x4 + x
81  + 4/ 3 
1  n  (here C is a constant of integration)
(c) A = and f (x) = 3 (x − 1) n
(n + r) 1/ 3
= lim ∑
54 [2019, 12 April Shift-I]
1 n→ ∞
r =1 n
4/ 3
1 | x3 + 1 |
(d) A = and f (x) = 9 (x − 1)2 (a) log e +C
54 1 + r 
1/ 3 2 x2
  n1 / 3 1 (x + 1)
3 2
Ans. (c) n
 n +C
dx dx
= lim
n→ ∞
∑ n4 / 3
(b)
2
log e
| x3|
Let I = ∫ =∫ r =1
x3 + 1
(x2 − 2x + 10)2 ((x − 1)2 + 32 )2 1/ 3 (c) log e +C
1 n  r
Now, put x − 1 = 3 tanθ ⇒dx = 3 sec θ dθ 2 = lim ∑  1 + n 
n r = 1
x
n→ ∞ | x3 + 1 |
(d) log e +C
Now, as per integration as limit of sum. x2
Ans. (c) = t (cos 2 α) + (sin 2 α) log e | sint | + C 5x 8 + 7 x 6
Key Idea = (x − α) cos 2 α + (sin 2 α) log e | sin (x − α)| 32 If f (x) = ∫ dx, (x ≥ 0),
+C
(x 2 + 1 + 2x 7 ) 2
(i) Divide each term of numerator and
denominator by x2 . = A (x) cos 2 α + B (x) sin 2 α + C (given) and f (0) = 0, then the value of f (1) is
1 Now on comparing, we get [2019, 9 Jan. Shift-II]
(ii) Let x2 + = t
A (x) = x − α and 1 1 1 1
x (a) − (b) − (c) (d)
B (x) = log e | sin (x − α)| 2 4 4 2
2x − 1
3
Let integral is I = ∫ dx Ans. (c)
x4 + x
31 For x 2 ≠ nπ + 1, n ∈ N (the set of 5x 8 + 7 x 6
2x − 1 / x2 We have, f (x) = ∫ dx
=∫ dx natural numbers), the integral (x + 1 + 2x 7 )2
2
1
x2 +
x 2 sin(x 2 − 1) − sin2(x 2 − 1)  x8   x6 
5 14  + 7  14 
[dividing each term of numerator and ∫x 2 sin(x 2 − 1) + sin2(x 2 − 1)
dx
=∫
x  x 
denominator by x2 ] 2
dx
 1   x2 1 2x 7 
Put x2 + = t ⇒  2x +  − 2   dx = dt
1  7 + 7 + 7 
is equal to (whereC is a constant of
x   x  x x x 
integration ) [2019, 9 Jan. Shift-I]
dt
∴ I = ∫ = log e | (t) | + C (a)
1
log e |sec(x2 − 1)| + C (dividing both numerator and
t 2 denominator by x 14 )
= log e  x2 +  + C
1  x2 − 1
 x (b) log e sec   +C 5x − 6 + 7 x − 8
 2  =∫ dx
x +1
3
(x − 5 + x − 7 + 2)2
= log e +C 1
x (c) log e sec2 (x2 − 1) + C Let x− 5 + x− 7 + 2 = t
2
 x2 − 1 ⇒ (− 5x − 6 − 7 x − 8 )dx = dt
30 Let α ∈(0, π /2) be fixed. If the 1
(d) log e sec2   +C
2  2  ⇒ (5x − 6 + 7 x − 8 )dx = − dt
integral
dt
tan x + tanα Ans. (b) ∴ f (x) = ∫ − 2 = − ∫ t −2dt
∫ tan x − tanα dx = A(x) cos2α + B (x) 2 sin(x2 − 1) − sin2(x2 − 1)
t
Let I = ∫ x dx t −2 + 1 t− 1
=− + C =− +C
sin2α + C, where C is a constant of 2 sin(x2 − 1) + sin2(x2 − 1) −2+ 1 −1
integration, then the functions x2 − 1 1 1
Put =θ = +C= +C
A(x) and B (x) are respectively 2 t x + x− 7 + 2
−5

[2019, 12 April Shift-II] ⇒ x2 − 1 = 2θ ⇒ 2x dx = 2 dθ x7


⇒ x dx = dθ = +C
(a) x + α and log e |sin(x + α )| 2x + x2 + 1
7

(b) x − α and log e |sin(x − α )| 2 sin2θ − sin4θ


Now, I = ∫ dθ Q f(0) = 0
(c) x − α and log e |cos (x − α )| 2 sin2θ + sin4θ 0
∴ 0= + C ⇒C =0
(d) x + α and log e |sin(x − α )| 2 sin2 θ − 2 sin2 θ cos2 θ 0+0+ 1
=∫ dθ
Ans. (b) 2 sin2 θ + 2 sin2 θ cos2θ x7
∴ f (x) =
tan x + tanα  π (Qsin2A = 2 sin A cos A) 2x + x2 + 1
7
Let I = ∫ dx, α ∈  0, 
tan x − tanα  2 2 sin2θ(1 − cos2θ) 1 1
=∫ dθ ⇒ f(1) = =
sin x sin α 2 sin2θ(1 + cos2θ) 2(1) 7 + 12 + 1 4
+
cos x cos α 1 − cos2θ
=∫ dx =∫ dθ
sin x sinα 33 Let n≥ 2 be a natural number and
− 1 + cos2θ
cos x cos α π
2 sin2 θ
0 < θ < . Then,
sin x cos α + sin α cos x =∫ dθ 2
=∫ dx
sin x cos α − sin α cos x 2 cos2 θ 1

sin (x + α) [Q1 − cos2A = 2 sin A and


2 (sin n θ − sinθ) n cosθ
=∫
sin (x − α)
dx
1 + cos2A = 2 cos2 A] ∫ sin n + 1 θ
dθ is equal to

Now, put x − α = t ⇒dx = dt, so = ∫ tan2 θ d θ = ∫ tanθd θ (where C is a constant of


sin (t + 2α) integration)
I=∫ dt = log e | secθ | + C [2019, 10 Jan. Shift-I]
sin t n+ 1

sin t cos 2 α + sin 2α cos t  x2 − 1  n  1  n


=∫ = log e sec   + C (a) 2 1 − n + 1  +C
sint
dt
 2  n −1  sin θ 
n+ 1
 cost   x2 − 1 
= ∫  cos 2 α + sin 2 α  dt Qθ =  n  1  n
 sint  2  (b) 2 1 + n − 1  +C
 n −1  sin θ 
n+ 1
1  e −4 t e −4 t  x +1
n  1  n = t −∫ dt  36 If ∫ dx = f (x) 2x − 1 + C,
(c) 2 1 − n − 1  +C 3  −4 −4  2x − 1
n − 1  sin θ 
n+ 1 [using integration by parts] where C is a constant of
n  1  n 1  te −4 t e −4 t 
(d) 2 1 −  +C =  + +C
integration, then f (x) is equal to
n + 1  sinn − 1 θ  3  −4 −16  [2019, 11 Jan. Shift-II]
Ans. (c) 1 2 1
= − e −4 t [4t + 1] + C (a) (x + 2) (b) (x + 4)
(sin θ − sinθ)
n 1/ n
cosθ 48 3 3
Let I = ∫ dθ 2 1
sinn + 1 θ
3
e −4 x (c) (x − 4) (d) (x + 1)
=− [4x 3 + 1] + C [Qt = x 3] 3 3
Put sinθ = t ⇒ cosθ dθ = dt 48
(t n − t) 1/ n Ans. (b)
∴ I=∫ dt ∴ f (x) = −1 − 4x 3
tn + 1 (comparing with given equation) We have,
 n t 
1/ n x+1
t  1 − n  ∫ dx = f (x) 2x − 1 + C ...(i)
  t   1− x2 2x − 1
=∫ dt 35 If ∫ dx = A(x)( 1 − x ) + C, 2 m
tn + 1 x+1
x4 Let I = ∫ dx
t ( 1 − 1 / t n − 1) 1 / n for a suitable chosen integer m and 2x − 1
=∫ dt
tn + 1 a function A(x), where C is a Put 2x − 1 = t2
( 1 − 1 / t n − 1) 1 / n constant of integration, then ⇒ 2dx = 2tdt
=∫ dt
tn (A(x)) m equals [2019, 11 Jan. Shift-I] ⇒ dx = tdt
Put
1
1− n−1 = u 1 −1 t2 + 1
(a) (b) +1
t 9x4 3x 3 I=∫ 2
1
tdt = ∫ (t2 + 3) dt
or 1 − t − (n − 1) = u −1 1 t 2
(n − 1) (c) (d)
⇒ dt = du 27x 9 27x 6  t2 + 1 
tn Q2x − 1 = t ⇒ x =
2

dt du Ans. (c)  2 
⇒ =
tn n − 1 We have, 1  t3  t
1 =  + 3 t  + C = (t2 + 9) + C
+1 23
u 1/ ndu 1 − x2  6
⇒ I=∫ =
un
+C ∫ x4
dx = A (x) ( 1 − x2 ) m + C … (i)
2x − 1
n− 1 (n − 1)  + 1
1 = (2x − 1 + 9) + C [Qt = 2x − 1 ]
n  1 − x2 6
Let I = ∫ 4
dx
n+ 1 x 2x − 1
= (2x + 8) + C
n  1 − n − 1 
1 n
x2  2 − 1
1 6
 t 
= +C x  x+4
(n − 1) (n + 1) =∫ dx = 2x − 1 + C
n+ 1 x4 3
 1  n 1 On comparing it with Eq. (i), we get
n 1 − n−1 
x −1
 θ x2 x+4
= sin +C =∫ dx f (x) =
n2 − 1 x4 3
Qu = 1 − 1 and t = sinθ =∫
1 1
− 1 dx 37 The integral ∫ cos (log e x) dx is
 tn − 1  x 3 x2
1 −2 equal to (where C is a constant
1 Put − 1 = t2 ⇒ dx = 2tdt of integration) [2019, 12 Jan. Shift-I]
5 −4x3 3
34 If ∫ x e dx = e −4x f (x) + C,
2 3
x x
48 x

1
dx = − tdt (a) [cos(log e x) + sin(log e x)] + C
x 3 2
where C is a constant of
t3 (b) x [cos(log e x) + sin(log e x)] + C
integration, then f (x) is equal to ∴ I = − ∫ t2dt = − +C
[2019, 10 Jan. Shift-II] 3
3/ 2
(c) x [cos(log e x) − sin(log e x)] + C
(a) − 4 x 3 − 1 1  1 − x2  x
(d) [sin(log e x) − cos(log e x)] + C
= − .  2  + C
(b) 4 x 3 + 1 3  x  2
  1  
1/ 2
(c) − 2 x 3 − 1 Qt =  2 − 1  Ans. (a)
 x  
(d) − 2 x 3 + 1  Let I = ∫ cos(log e x)dx
1 1 1
Ans. (a) =− ( 1 − x 2 3
) + C …(ii) = x cos(log e x) − ∫ x (− sin(log e x)) ⋅dx
3 1 −4 x 3 3 x3 x
Given, ∫ x 5 e −4 x dx = e f (x) + C
48 On comparing Eqs. (i) and (ii), we get [using integration by parts]
In LHS, put x 3 = t 1 = x cos(log e x) + ∫ sin(log e x) dx
A (x) = − 3 and m = 3
⇒ 3x2dx = dt 3x = x cos(log e x) + x sin(log e x)
1 1
3 1
So, ∫ x 5 e −4 x dx = ∫ t e −4 tdt ∴ (A (x)) m = (A (x)) 3 = − − ∫ x (cos(log e x)) dx
3 27 x 9 x
[again, using integration by parts] Ans. (b) x 10
(b) +C
⇒ I = x cos(log e x) + x sin((log e x) − I We have, 2 (x 5 + x 3 + 1)2
x sin x ⋅ cos x
2 2
⇒ I = [cos(log e x) + sin(log e x)] + C I=∫ dx x5
2
(sin x + cos x ⋅ sin x
5 3 2 (c) +C
2 (x 5 + x 3 + 1)2
3x 13 + 2x 11
38 The integral ∫ dx is + sin3 x ⋅ cos2 x + cos5 x)2 − x 10
(2x 4 + 3x 2 + 1) 4 (d) +C
=∫
sin2 x cos2 x
dx
2 (x 5 + x 3 + 1)2
equal to (where C is a constant of {sin3 x (sin2 x + cos2 x) where, C is an arbitrary constant.
integration) [2019, 12 Jan. Shift-II] + cos3 x (sin2 x + cos2 x)}2 Ans. (b)
x4 sin2 x cos2 x 2x 12 + 5x 9
(a) +C =∫ dx Let I = ∫ dx
6 (2 x + 3x2 + 1) 3
4
(sin3 x + cos3 x)2 (x 5 + x 3 + 1) 3
x 12 sin2 x cos2 x 2x 12 + 5x 9
(b) +C =∫ dx =∫ dx
6 (2 x + 3x2 + 1) 3
4
cos6 x (1 + tan3 x)2 x 15 (1 + x − 2 + x − 5 ) 3
x4 tan2 x sec2 x 2x − 3 + 5x − 6
(c) +C =∫ dx =∫ dx
(2 x + 3x2 + 1) 3
4
(1 + tan x) (1 + x − 2 + x − 5 ) 3
3 2
12
x
(d) +C Put tan3 x = t Now, put 1 + x − 2 + x − 5 = t
(2 x + 3x2 + 1) 3
4
⇒ 3 tan2 x sec2 xdx = dt ⇒ (− 2x − 3 − 5x − 6 ) dx = dt
Ans. (b) 1 dt
∴ I= ∫ ⇒ (2x − 3 + 5x − 6 ) dx = − dt
3x 13 + 2x 11 3 (1 + t)2 dt
Let I = ∫ dx ∴ I = − ∫ 3 = − ∫ t − 3 dt
(2x 4 + 3x2 + 1) 4 −1 t
⇒ I= +C
3
+
2 3 (1 + t) t− 3+ 1 1
3
=− +C= 2 +C
=∫ x x5 −1 −3+ 1 2t
4
dx ⇒ I= +C
 3 1  3 ( 1 + tan3 x) x 10
2 + 2 + 4  = +C
 x x  2 (x + x 3 + 1) 2
5
40 Let I n = ∫ tan n x dx (n > 1). If
[on dividing numerator and denominator
by x 16 ] I 4 + I 6 = a tan 5 x + bx 5 + C, where C dx
3 1
42 The integral ∫ 3
equals
Now, put 2 + 2 + 4 = t is a constant of integration, then
x x the ordered pair (a, b) is equal to x (x2 4
+ 1) 4
 −6 4 
⇒  3 − 5  dx = dt [JEE Main 2017] [JEE Main 2015]
x x  1/ 4
(a)  − , 1 (b)  , 0 
1 1  x4 + 1
3 2 dt  5  5  (a)  4  + c (b) (x 4 + 1) 1/ 4 + c
⇒  3 + 5  dx = −  x 
x x  (c)  , − 1 (d)  − , 0 
2 1 1 1/ 4
 x4 + 1
− dt 1 t− 4 + 1 5   5  (c) − (x 4 + 1) 1/ 4 + c (d) −  4  +c
So, I = ∫ 4 = − × +C  x 
2t 2 −4+ 1 Ans. (b)
Ans. (d)
1
= 3 +C=
1
+C We have, I n = ∫ tann x dx
3 dx dx
6t  3
6 2 + 2 + 4 
1  ∴ I n + I n + 2 = ∫ tann x dx + ∫ tann + 2 x dx ∫ 3
=∫ 3
 x 
x 5  1 + 4 
x x2 (x 4 + 1) 4 1 4
 1  = ∫ tann x (1 + tan2 x) dx
3  x 
Qt = 2 + x2 + x 4  = ∫ tann x sec2 x dx
  1 4
x 12 Put 1 + 4 = t 4 ⇒ − dx = 4t 3dt
= + C tann + 1 x x x5
6 (2x 4 + 3x2 + 1) 3 = +C
n+ 1 dx −t 3dt
tan5 x ⇒ = − t 3dt = ∫ 3
39 The integral Put n = 4, we get I 4 + I 6 = +C x5 t
sin 2 x cos 2 x 5 = − ∫ dt = − t + c
∫ (sin 5 x + cos3 x sin 2 x dx ∴
1
a = and b = 0
= −  1 + 4  + c
1 4
1

5
 x 
+ sin 3 x cos 2 x + cos 5 x) 2 2x 12 + 5x 9
41 The integral ∫
1
dx is 1 x+
(x 5 + x 3 + 1) 3 43 The integral ∫ 1 + x −  e x is
is equal to [JEE Main 2018]  x
1 −1
(a) + C (b) +C equal to [JEE Main 2016] equal to [JEE Main 2014]
3 (1 + tan3 x) 3 (1 + tan3 x) − x5 1 1
x+ x+
1 −1 (a) +C (a) (x − 1) e x +C (b) x e x +C
(c) +C (d) +C (x + x + 1)
5 3 2
1 + cot x 3
1 + cot 3 x x+
1
x+
1
(c) (x + 1) e x +C (d) − xe x +C
(where C is a constant of integration)
Ans. (b) Now, let us assume that Ans. (d)
1
 1 + x − 1  e x + x dx 5 tan x sin x
I=∫ Let I = 2 ∫
∫   dx dx
π
 x tan x − 2 
sin  x − 
x+
1
x+
1  4
dx + ∫ x  1 − 2  e x dx
1 On multiplying by cos x in numerator and
=∫e x
π
 x  denominator, we get Put x − = t ⇒ dx = dt
5 sin x 4
x+
1
x+
1
x+
1 I=∫ dx
π
sin  + t  dt
d sin x − 2 cos x
=∫e x dx + x e x −∫ (x) e x
dx 4 
This special integration requires special ∴ I= 2∫
1 1 1 sin t
x+ x+ x+
=∫e x dx + x e x −∫ e x substitution of type
 dD r   1 1 
 N r = A (D r ) + B   , A and B are = 2∫ cott + dt
x+ 
2 
1 1
 1  x+  dx   2
Q∫  x − 2  e x = e x 
  x   constants. = log | sin t | + t + C
π
Let 5 sin x = A (sin x − 2 cos x) = x + logsin  x −  
1
x+ + C
= xe x +C + B (cos x + 2 sin x)   4 
⇒ 0 cos x + 5 sin x = (A + 2B) sin x
44 If ∫ f (x) dx = ψ (x), then ∫ x 5f (x 3 )dx is dx
+ (B − 2A) cos x 47 ∫ cos x + 3 sin x
is equal to
equal to [JEE Main 2013] On comparing the coefficients of sin x [AIEEE 2007]
and cos x, we get
1 3
[x ψ (x 3) − ∫ x2 ψ (x 3) dx] + C 1 x π
(a) A + 2B = 5 and B − 2A = 0 (a) log tan  +  + C
3 2  2 12 
On solving the above two equations in A
1 3
x ψ (x 3) − 3 ∫ x 3ψ (x 3) dx + C 1 x π
(b) and B, we get (b) log tan  −  + C
3 2  2 12 
1 3 A = 1 and B = 2
(c) x ψ (x 3) − ∫ x 2 ψ (x 3) dx + C x π
⇒ 5 sin x = (sin x − 2 cos x) (c) log tan  +  + C
3  2 12 
1 3 + 2 (cos x + 2 sin x)
(d) [x ψ (x 3) − ∫ x 3ψ (x 3) dx] + C 5 sin x x π
⇒I=∫ dx (d) log tan  −  + C
3
sin x − 2 cos x  2 12 
Ans. (c) Ans. (a)
(sin x − 2 cos x) + 2 (cos x + 2 sin x)
=∫ dx dx
Given, ∫ f (x) dx = ψ(x) (sin x − 2 cos x) Now, ∫
Let I = ∫ x 5f (x 3) dx cos x + 3 sin x
sin x − 2 cos x
=∫ dx dx
sin x − 2 cos x =∫
Put x3 = t 1 3 
(cos x + 2 sin x) 2  cos x + sin x 
⇒ x2dx =
dt
…(i) + 2∫ dx 2 2 
3 (sin x − 2 cos x)
π
= ∫ sec  x −  dx
d (sin x − 2 cos x) 1
1 1 = ∫ 1 dx + 2 ∫
∴ I = ∫ tf (t) dt = [t ψ(t) − ∫ ψ(t) dt] 2  3
3 3 (sin x − 2 cos x)
[using integration by parts] 1  x π π
= x + 2 log | (sin x − 2 cos x) | + k …(ii) = log tan  − +  + C
1 3 3 2 2 6 4
= [x ψ(x ) – 3 ∫ x2 ψ(x 3) dx] + C where, k is the constant of integration.
3 1 x π
[from Eq. (i)] Now, by comparing the value of I in Eqs. = log tan  +  + C
1 3 3 2  2 12 
= x ψ(x ) − ∫ x ψ(x ) dx + C
2 3 (i) and (ii), we get
3 a =2 sin x
48 If ∫ dx = Ax
sin x dx sin (x − α)
45 f the integral 46 The value of 2 ∫ is
π + B log sin(x − α) + C, then the value
sin  x − 
5 tan x
∫ tan x − 2 dx = x + a log  4
of (A, B) is [AIEEE 2004]
(a) (sin α ,cos α )
[AIEEE 2008]
| sin x − 2 cos x | + k , then a is equal (b) (cos α ,sin α )
to  π (c) (− sin α ,cos α )
[AIEEE 2012] (a) x − log cos  x −  + C
(a) –1 (b) –2  4 (d) (− cos α ,sin α )
(c) 1 (d) 2  π Ans. (b)
(b) x + log cos  x −  + C
Ans. (d)  4 Let I = ∫
sin x
dx
5 tan x sin (x − α)
Given Integral ∫ dx  π
tan x − 2 (c) x − log sin  x −  + C Put x − α = t ⇒ dx = dt
 4
5 tan x sin (t + α)
To find The value of ‘ a ’, if ∫ dx ∴ I=∫ dt
tan x − 2  π sin t
(d) x + log sin  x −  + C
 4 sin t cos α + cos t sin α
= x + a log | sin x − 2 cos x | + k …(i) ⇒ I=∫ dt
sin t
⇒ I = cos α ∫ 1 dt + sin α ∫
cos t 1 dx ⇒| A | = (J7 , 3 − J4 , 3) (J8 , 3 − J5 , 3)
dt = ∫ π (J9 , 3 − J6 , 3)

cos  x + 
sin t 2
⇒ I = cos α (t) + sin α log sin t + C 1  4 1/ 2 x 7 − x 4 1/ 2 x 8 − x 5
=∫ dx ⋅ ∫ dx ⋅
⇒ I = cos α (x − α) + sin α log π 0 x3 − 1 0 x3 − 1
sec  x +  dx
1
sin (x − α) + C 1
=
2 ∫  4 1/ 2 x 9 − x 6
∫0 x 3 − 1 dx
⇒ I = x cos α + sin α log sin (x − α) + C π x π
tan  + +  
log
1
= + C 1/ 2 4 1/ 2 5 1/ 2 6
[let C = − α cosα + C 1] 2   4 2 8  =∫ x dx ∫ x dx ⋅ ∫ x dx
0 0 0
But I = Ax + B log sin (x − α) + C 1   x 3 π 
= logtan  + + C
1/ 2 1/ 2 1/ 2
∴ x cos α + sin α log sin (x − α) + C x5 x6 x7 1
2  2 8  = ⋅ ⋅ =
= Ax + B log sin (x − α) + C 5
0
6 0
7 0
(210) 218
dx
On comparing both sides, we get 50 ∫ is equal to Now, | adj A − 1 | =
1
= ((210) ⋅218 )2
A = cosα and B = sinα x (x n + 1) [AIEEE 2002] | A |2
Alternate Solution 1  xn  = 1052 ⋅238
(a) log  n  + C
Given that, n  x + 1
52 The function f (x), that satisfies the
sin x 1  x n + 1
∫ sin (x − α) dx = Ax + B log sin (x − α) + C (b) log  n  + C condition
n  x  π /2

On differentiating both sides w.r.t. x, we  x 


(c) log  n  + C
n f (x) = x + ∫ sin x ⋅ cosy f (y) dy, is
get 0
 x + 1
sin x cos (x − α) [2021, 01 Sep. Shift-II]
=A+ B (d) None of the above 2
sin (x − α ) sin (x − α) (a) x + ( π − 2) sin x
Ans. (a) 3
⇒ sin x = A sin (x − α ) + B cos (x − α )
dx xn − 1 (b) x + ( π + 2) sin x
⇒ sin x = A (sin x cos α − cos x sin α) Let I=∫ =∫ n n dx
x (x + 1)
n
x (x + 1) π
+ B (cos x cos α + sin x sin α) (c) x + sin x
2
Put x n + 1 = t ⇒ nx n − 1 dx = dt
⇒ sin x = sin x (A cos α + B sin α) (d) x + (π − 2) sin x
1 dt 1  1 1
n ∫ t (t − 1) n ∫  t − 1 t 
+ cos x (B cos α − A sin α) ∴ I= =  −  dt Ans. (d)
On comparing the coefficients of π /2
sin x and cos x both sides, we get
= log 
1 t − 1
f (x) = x + ∫0 sin x ⋅ cos yf (y) dy
 +C
A cos α + B sin α = 1 …(i) n  t  π /2
Let K =∫ cos yf (y) dy … (i)
and B cos α − A sin α = 0 …(ii) 1  xn  0
= log  n  +C Then, f (x) = x + K sin x … (ii)
On solving Eqs. (i) and (ii), we get n  x + 1
From Eqs. (i) and (ii),
A = cos α, B = sin α
π /2
dx TOPIC 2
49 ∫ cos x − sin x is equal to f (x) = x + ∫0 sin x cos y (y + k sin y) dy
π /2
Definite Integrals = x + sin x ∫ y cos ydy
[AIEEE 2004] 0
1/ 2
1  x π x n k π /2
sin x ∫ sin2y dy+
(a)
2
log tan  −  + C
2 8  51 Let J n, m = ∫ xm −1
dx , ∀ n > m and 2 0
0 π ⋅2 k sin x
1  x f (x) = x + sin x ⋅ + … (iii)
(b) log cot   + C n, m ∈ N. Consider a matrix 2 2
2  2
A = [a ij ] 3 × 3 where From Eqs. (ii) and (iii),
1  x 3π 
(c) log tan  −  + C J −J , i≤ j k=
π −2 k
+
2 2 8  a ii =  6 + i , 3 i + 3, 3 . Then, 2 2
 0, i> j
1  x 3π  ⇒ k = π −2
(d) log tan  +  +C | adj A −1 | is
2 2 8  [2021, 01 Sep. Shift-II]
∴ f (x) = x + ( π − 2) sin x
(a) (15)2 × 2 42 (b) (15)2 × 2 34
Ans. (d)
(c) (105)2 × 2 38 (d) (105)2 × 2 36 53 Let [t] denote the greatest integer
dx
Let I = ∫ Ans. (c) ≤ t. Then the value of
cos x − sin x
a 11 a 12 a 13  1
1 dx A =  0 a22 a23  8 ⋅ ∫ ([2x] + | x |) dx is
=
2 ∫  1 1 
 
 cos x − sin x   0 0 a 33  −
1
 2 2  2
⇒| A | = a 11a22a 33 [2021, 31 Aug. Shift-I]
Ans. (5)  −2 πx 
1 16 log e x 2
1
=π 2
 π
cos
2  0
+ π2 57 ∫6 log e x 2 + log e (x 2 − 44x + 484)
dx
8 ∫ ([2X] + X dx  2 πx  
2
−1 / 2
 ( x − 1)  − cos  is equal to [2021, 27 Aug. Shift-I]
1  π 2   1
= − ≤ x 0 ⇒[2x] = − 1 2 2 π x
(a) 6 (b) 8
2 + π2 ∫ cos dx (c) 5 (d) 10
1 π 2
1
0 ≤ x < ⇒[2x] = 0 2 Ans. (c)
2 π2 2 πx 
= π2   +
2 2
(1 − 0) + 2 π ⋅  sin  16 lne (x2 )
1
≤ x < 1 ⇒[2x] = 1  π π π 2  1
Let I = ∫ dx
2
6 lne (x2 ) + lne (484 − 44x + x2 )
= 2 π + 2 π + 4(0 − 1) = 4 π − 4 = 4( π − 1)
0 1/ 2 1 16 lne (x2 )
=∫
I= ∫ (− 1 − x) dx + ∫ (0 + x)dx + ∫ ( 1 + x) dx 56 If 6 lne (x ) + lne (22 − x)2
2
dx
−1 / 2 0 −1 / 2 2 n
 x2   x2 
0 1/ 2
 x2 
1
 1   22   n2  16 2 lne x dx
= − x −  +  + x +  U n = 1 + 2  1 + 2  … 1 + 2  , =∫
2  −1 / 2  2  0 2  1/ 2  n  n   n  6 2 lne x + 2 lne (22 − x)
 
−4 16 lne x dx
 1 1  1  1  1 1  I=∫ ...(i)
= −  −  +   + 1 +  − +  2
then lim (U n ) n is equal to 6 lne x + lne (22 − x)
 2 8  8  2  2 8 n→ ∞
5 b b
=
8
[2021, 27 Aug. Shift-I] Q ∫a f (x)dx = ∫a f (a + b − x)dx
(a) e2 /16 (b) 4/ e 16 lne (22 − x)
5 ∴ I=∫
∴ 8I = 8. = 5 (c) 16 / e2 (d) 4 / e2 dx ...(ii)
8 6 lne (22 − x) + lne x
Ans. (a)
x −4 Adding Eqs. (i) and (ii), we get
54 If x φ (x)= ∫ (3t 2 − 2φ′ (t) dt, x > − 2, Let y = lim (U n )n
2
2I = ∫
16 ln x + ln (22 − x)
e e
dx
n→ ∞ 6 ln x + ln (22 − x)
5 e e
and φ(0) = 4, then φ(2) is y = lim 16 16

n→ ∞ 2I = ∫ dx = x 6 = 10
⋅3 
[2021, 31 Aug. Shift-I] −4 −4 −4
⋅2 6
 1  n2  22  n 2  32  n 2 
Ans. (4)  1 + 
2

 1 + 2
 
 1 + 2
 ... or I=5
x  n  n   n  
x φ(x) = ∫ 3t2 − 2φ′ (t) dt  
Taking log on both sides, we get
58 The value of the integral
5 1
x dx
⇒ xφ (x) = [t 3
− 2φ(t)] 5x n 
−4 
ln y = lim ∑  2 ⋅ r ln  1 + 2  
r2   ∫ (1 + x) (1 + 3x) (3 + x) is
⇒ x φ (x) = (x 3 − 125) − 2[φ (x) − φ(5)] n→ ∞
r =1  n  n  0
Now, φ(0) = 4 [2021, 27 Aug. Shift-II]
Now, replace lim Σ → ∫ 
⇒ 0 = − 125 − 2 [4 − φ( 5)] n→ ∞ π 3 π  3
r 1 (a)  1 −  (b)  1 − 
⇒ φ(5) = 133 /2 → x, → dx 8  2  4  6 
For φ(2), n n
Lower limit = 0 π  3 π  3
 133  (c) 1 −  (d) 1 − 
⇒ 2 φ (2) = (8 − 125) − 2  φ(2) −
 2  Upper limit = 1 8  6  4  2 
⇒ 4 φ(2) = 16 ⇒ φ(2) = 4 ∴
1
ln y = ∫ − 4x ln(1 + x2 )dx Ans. (a)
0
1 x dx
55 If [x] is the greatest integer ≤ x, Let 1 + x2 = t Let I = ∫
0 (1 + x) (1 + 3x) (3 + x)
2 dt
πx
then π 2 ∫  sin  (x − | x |) [x] dx is
⇒ xdx = Put x = t ⇒x = t2
2
 2 or dx = 2t dt
0 When x→ 0, t → 1 1 2t2dt
equal to and x → 1, t → 2 ∴ I=∫ 2
0 (t + 1) (3t2 + 1) (t2 + 3)
[2021, 31 Aug. Shift-II] 2
∴ ln y = ∫ − 2 ln t dt 1 (3t + 1) − (t2 + 1)
2
(a) 2 (π − 1) (b) 4 (π − 1) 1 =∫ dt
(c) 4(π + 1) (d) 2 (π + 1) = − 2(t ln t − t)21
0 (t2 + 1) (3t2 + 1) (t2 + 3)
Ans. (b) = − 2(2 ln 2 − 2 + 1) 1  1 1 
=∫  2 − 2  dt
 πx 2
= − 2(2 ln 2 − 1) 0 ( t + 3) (t2 + 1 ) + 2
+
I = π ∫ sin   (x − [x]) [x] dx
2
 ( t 3) ( 3t 1) 
0  2  1
⇒ ln y = ln + 2 1 1 1 3 1 
1  πx  16 =∫  2 − 2 − +  dt
= π2 ∫ sin   x 0 dx 0 2(t + 1) 2 ( + 3) 8 (3 2
+ 1) 8 ( 2
+ 3)
0  2  1  t t t 
⇒ y = e2
1 13 1 3 dt
2  πx  16 =∫
dt
=∫
dt
−∫
+ π2 ∫1 sin   (x − 1) dx
 2  0 2(t + 1)
2 0 8 (3t + 1)
2 0 8 (t2 + 3)
1
1
3 3  Ans. (b) π
1 
=  tan−1 t  −  tan−1 3t  2  1 + sin 2 x 
2 0  8 3 0 1 2 n − 1 n2
 3 t 
1
lim ∑
n→ ∞ n r = 0 n2 + 4r2
62 The value of ∫π  1 + π sin x  dx is
− tan−1  −
8 3 30 1 2n −1 1
= lim ∑ 2
2
n → ∞ n r =0
1 + 4 
π 3 π 3 π π 3π r [2021, 26 Aug. Shift-II]
= − − = −  n π 5π 3π 3π
8 8 3 8 6 8 16 2 (a) (b) (c) (d)
 1
π 3 =∫ dx 2 4 4 2
= 1 −  1 + 4x2
 2  Ans. (c)
8  0
1 1 π
= [tan−1 2x]20 = tan−1 4
59 The value of 2 2 2
1 + sin2 x
1/ 2  2 2 
1/ 2 I= ∫ 1 + π sin x
dx …(i)
  x + 1 +  x − 1 − 2 dx is π
∫   x − 1  x + 1  61 If the value of the integral
5 x + [x]

2
−1 2   −1

[2021, 26 Aug. Shift-I]


∫0 e x − [x] dx = αe + β, where I=∫
π /2 1 + sin2 (− x)
1 + π sin(− x )
– π /2

(a) log e 4 (b) log e 16 α, β ∈R, 5α + 6β = 0 and [x] denotes  b b 


the greatest integer less than or Q∫ f (x) dx = ∫ f (a + b − x) dx 
(c) 2 log e 16 (d) 4 log e (3 + 2 2 )  a a 
Ans. (b) equal to x, then the value of π / 2 1 + sin x
1 (α + β) 2 is equal to : =∫ dx
1/ 2 – π / 2 1 + π − sin x
2   x + 12  x − 1 2  [2021, 26 Aug. Shift-II]
π
  − 2
Let I = ∫  x −1
 +
 x + 1 
dx (a) 100 (b) 25 2
π sin x (1 + sin2 x)
−1
2
  (c) 16 (d) 36 I= ∫ 1 + π sin x
dx …(ii)
1 π
1
Ans. (b) −
2 2
5
2 x + 1 x − 1 2  x + [x]
I = ∫   − I=∫ dx = α e −1 + β Adding Eqs. (i) and (ii), we get
  dx e x − [x] π
 
 x − 1 x + 1 
−1  0
2
1 2 3 (1 + π sin x ) (1 + sin2 x)
2
1 I=∫
x x+ 1 x+2
dx + ∫ x − 1 dx + ∫ x −2 dx
2I = ∫ (1 + π sin x )
dx
e x
e e −π
2 0 1 2
x+ 1 x−1 2
= ∫ −
x−1 x+ 1
dx 4
x+3
5
x+4 π
−1 + ∫e x −3
dx + ∫ e x − 4 dx 2
⇒ 2I = ∫ (1 + sin
2
2 3 4 x) dx
1 Let I =I 1 + I2 + I 3 + I 4 + I 5 −π
2 2 2 π
(x + 1)2 − (x − 1)2 x+ 1 π
= ∫ (x − 1) (x + 1)
dx Here, I2 = ∫
e x −1
dx Put x = t + 1
⇒ 2I = [x] −2 π
2
+ 2 ∫ sin2 x dx
−1 1
2 ⇒ dx = dt 2 0
1 1
t+2
1
t
1
2 [Qsin2 x is an even function, so
2 =∫ dt = ∫ dt + ∫ dt π /2 π /2
∫ sin dx = 2∫ sin xd x]
4x 2 2
= ∫ (x − 1)(x + 1)
dx 0
et 0
et 0
et −π / 2 π 0
−1 1 2
I2 = I 1 + 2∫ e −t
dt = I 1 + 2(1 − e ) −1 1  π  π 1
2
1 1 ⇒ I =  −  −   + ∫ (1 − cos2x) dx
2 2 Similarly, 0 2 2  2  2 0
x −2x
= 2⋅4 ∫ dx = 4 ∫ 2 dx I 3 = I 1 + 4 ( 1 − e −1 )
π
(x − 1)(x + 1) x −1 π 1  sin2x  2
0 0 ⇒ I = + x−
1 I 4 = I 1 + 6 ( 1 − e −1 ) 2 2  2  0
= −4 [log (x2 − 1)] 0 2 I 5 = I 1 + 8 ( 1 − e −1 ) π 1  π  
+  − 0  − (0 − 0) 
 1   I = I 1 + I2 + I 3 + I 4 + I 5 2 2   2  
= − 4 log  − 1 − log | − 1 | π π 3π
  2   = 5I 1 + (2 + 4 + 6 + 8) (1 − e −1) ⇒ I= + =
2 4 4
 1 = 5I 1 + 20 (1 − e −1)
= − 4 log   = 4 ln2 = ln 16 1 63 Let the domain of the function
 2 I 1 = ∫ xe − 1dx = − [e − x (x + 1)] 10
2n − 1 2
0
f (x) = log 4 [log 5 (log 3 (18x − x 2 − 77))]
60 The value of lim
1
Σ 2
n
is = 1 − 2e − 1
be (a, b).
n→ ∞ n r = 0 n + 4r 2 ∴5I 1 + 20 (1 − e − 1) = 5 (1 − 2e − 1)
+ 20 (1 − e − 1)
Then the value of the integral
[2021, 26 Aug. Shift-I] b
= 25 − 30 e − 1 sin 3 x
1
(a) tan−1 (2)
2
1
(b) tan−1 (4)
2 ∴ α = − 30, β = 25
∫ [sin 3 x + sin 3 (a + b − x)] dx is
a
−1 1 Also it satisfy 5α + 6β = 0
(c) tan (4) (d) tan−1 (4) equal to …………
4 Now, (α + β)2 = (−30 + 25)2 = (−5)2 = 25 [2021, 27 July Shift-I]
1 π
Ans. (1) n
and ∑ by ∫ , 4
(1 + tan2 x) sec2 x
log 4 [log 5 {log 3 (18x − x − 77)}]
2
j =1 0
⇒ I=∫ dx
⇒ log 5 [log 3 (18x − x2 − 77)] > 0 1 0
1 + tan4 x
2x + 8
⇒ log 3 (18x − x2 − 77) > 5 =∫ dx Let tan x = t
2x + 4
⇒ (18x − x2 − 77) > 3 0
sec2 x dx = dt
1
⇒ x2 − 18x + 80 < 0 4 π
= ∫1+ dx When x = 0, t = 0 and when x = ,t = 1
⇒ (x − 8) (x − 10) < 0 0
2x + 4 4
1
⇒ 8 < x < 10 ⇔a < x < b  
1 1 + t2
4
= x + log e (2x + 4) ⇒ I=∫ dt
b
sin3 x  2  1 + t4
I=∫ dx …(i) 0 0

a
sin3 x + sin3 (a + b − x) = (1 + 2log e 6) − (0 + 2log e 4) 1 1+
1
b b  3 ⇒ I=∫ t2 dt
= 1 + 2 log e  
Using ∫ f (x)dx = ∫ f (a + b − x)dx  2 0t
2
+
1
a a t2
b
sin3 (a + b − x)
65 The value of the definite integral 1+ 2
1
I=∫ dx …(ii) π 1

sin3 (a + b − x) sin3 x 4 =∫ t dt
dx 2
∫ 0 t − 1  + 2
a
is equal to
(1 + e x cos x
) (sin4 x + cos4 x)  
Adding Eqs. (i) and (ii),

π  t
b
sin3 x 4 1
2I = ∫ dx [2021, 27 July Shift-I] Let t− =u
sin x + sin3 (a + b − x)
3
t
a π π
(a) −
⇒  1 + 2  dt = du
b (b) 1
sin3 (a + b − x) 2 2 2
+ ∫ sin3 x + sin3 (a + b − x) dx π π
 t 
a (c) − (d) When t = 0, u → − ∞
b 4 2
⇒ 2I = ∫ 1 dx ⇒ 2I = [x] ba ⇒ 2I = [b − a] When t = 1, u = 0
Ans. (b) 0
du
π I= ∫
a
b −a  4 u2 + 2
∴ I=  dx −∞
 2  I= ∫ (1 + e x cos x ) (sin4 x + cos4 x)
...(i)
0

π 1  − 1 u  
As b = 10 ⇒ a = 8 = tan   
2 
4
 2  − ∞
10 − 8 b b
So, I = =1 Using ∫ f (x)dx = ∫ f (a + b − x)dx
2 1   π π
= 0 −  − 2   = 2 2
1 n (2 j − 1) + 8n
64 The value of lim ∑
a a
is π
2  
n→ ∞ n j = 1 (2 j − 1) + 4n 4
dx
equal to [2021, 27 July Shift-I] I= ∫ (1 + e − x cos x ) (sin4 x + cos4 x)
66 The value of the definite integral

π
 3
(b) 2 − log e  
2 −
(a) 5 + log e   4 24
 2  3 dx
π
4
∫ 1 + 3 tan2x
is
 3 x cos x
(c) 3 + 2log e  
2 e dx π

 3
(d) 1 + 2log e  
 2
I= ∫ (1 + e x cos x ) (sin4 x + cos4 x)
...(ii) 24 [2021, 25 July Shift-I]
π
− π π π π
Ans. (d) 4 (a) (b) (c) (d)
3 6 12 18
1 n (2 j − 1) + 8n Adding Eqs. (i) and (ii),
lim
n→ ∞

n j = 1 (2 j − 1) + 4n
π
4
Ans. (c)
(1 + e x cos x )dx 5π

On dividing both numerator and


2I = ∫ (1 + e x cos x ) (sin4 x + cos4 x)
24
dx
denominator by n, −
π
4
Let I = ∫ 1 + 3 tan2x
π
2  −   + 8
j 1 π
24
1 n  n  n 4
lim ∑ dx b b
n→ ∞ n
j = 1 2
j  1
⇒ 2I = ∫ sin4 x + cos4 x Q ∫ f (x)dx = ∫ f (a + b − x)dx
  −  +4 −
π
 n  n 4
a a

1 π 5π
Now, lim = 0 4 24
n→ ∞ n dx dx
 
⇒ 2I = 2 ∫
sin x + cos x
4 4
= ∫ 3
2  + 8 
j π
 π 2
 n 1 + tan2 − x 
0
1  n 
So,  ∑ lim    
24
 1 4
 j = 1 n n→ ∞  j 
+
 ∴ 4 is an even function
 2   4   sin x + cos x
4
 5π
  n  24
π dx
j
Now, replacing by x, by dx
1 4
sec4 x
I= ∫ 1 + 3 cot2x
n n ⇒ I=∫ dx π

0
tan4 x + 1 24
5π Ans. (a)  −4 π 3 − π + π 
24 3 100 π
= (e −1 − 1)  
tan2x sin2 x απ 3  4π + 1 
2
= ∫ 1 + 3 tan2x ∫ dx = , α ∈R
π  x −  x 
  1 + 4 π2  4π3  4(1 − e −1) π 3
0  π  π   = − (e −1 − 1)  2 =
1 
24 e
5π 100 π  4π + 4 π2 + 1
sin2 x
24
1 + 3 tan2x ∫ dx  π3
50 (A − B) = 200 (1 − e −1)  2


x 
⇒ 2I = dx   
1 
0 π 
π 1 + 3 tan2x e  4π +
24 sin2 x
If f (x) = ∴ α = 200 (1 − e −1)
5π x  
 
π 
⇒ 2I = ∫ 1dx = [x] 24
π
e
69 The value of the integral
24 sin2 (x + π) sin2 x 1
f (x + π) = =
⇒ 2I =

24 e
x + π 

 π 

e
x 
 
π  ∫ log e ( 1 − x + 1 + x )dx is equal to
−1
π
π sin2 x [2021, 20 July Shift-I]
∴ I= 100 ∫ x dx
12 1 π 3 π
0
eπ (a) log e 2 + − (b) 2 log e 2 + −1
π −x 2 4 2 4
67 The value of the integral ⇒ 100 ∫ e π sin2 xdx π π 1
1 (c) log e 2 + − 1 (d) 2 log e 2 + −
∫ log(x + x + 1)dx is
2 0 2 2 2
π −x
−1  1 − cos2x  Ans. (c)
⇒ 100 ∫ e π
  dx 1
[2021, 25 July Shift-II]  2 
(a) 2 (b) 0
0
π −x
∫ log e ( 1− x + 1 + x )dx
−1
(c) −1 (d) 1 ⇒ 50 ∫ eπ (1 − cos2x)dx a a
Ans. (b)
π  −x
0
−x 
∫ f (x) dx = 2∫ f (x)dx, if f (− x) = f (x)
1 −a 0
Let I = ∫ log(x + x2 + 1)dx ⇒ 50 ∫  e{π − e π cos2xdx 
−1  A 142 4
3  1
0 B  So, 2∫ log e ( 1 − x + 1 + x )dx
Let f (x) = log(x + x2 + 1) π
 −x    0
π −x  e −1 − 1 
Now, f (− x) = log(− x + x2 + 1) e π  1
A = ∫ e dx = 
π
 =  −1  I = 2∫ log e ( 1 − x + 1 + x ) ⋅ 1 dx
 −1  
 x2 + 1 + x   0  
= log  ( x2 + 1 − x) ×  0  π 
0
 π
  x2 + 1 +  I
  x   = − π (e −1 −1) ⇒ = [log e ( 1 − x + 1 + x ) ⋅ x] 10
2
 x2 + 1 − x2  Using, 1 1
= log   e ax 1

∫ e cosbxdx = a2 + b2 (a cosbx + b sinbx) 2 1+ x 2 1− x
ax
 x2 + 1 + x  −∫ ⋅ x dx
 
1− x + 1+ x
 1  π  −1 
x 0
= log    
π 1  1− x − 1+ x 
1
 x + x2 + 1  B =∫e cos2xdx ⇒
I
= log e 2 − ∫  
  2 2 0  1 − x + 1 + x 
0
π
= log 1 − log(x + x2 + 1)  −x 
x
dx
 eπ  −1 1 − x2
= − log(x + x2 + 1) =   cos2x + 2 sin2x  ⇒
I
= log e 2 −
1
 4 + 1   π  2 2
⇒ f (− x) = − f (x)  π2  0 1 (1 − x) + (1 + x) − 2 1 − x2 
Qlog (x + x2 + 1) is an odd function,
∫   x
    dx
 e −1  −1  1  −1 ( 1 − x) − ( 1 + x)  1 − x2
1
∴∫ log (x + x2 + 1)dx = 0 =  
  −     0 
−1
 4 + 1 
 π   4 + 1   π  1
1 2(1 − 1 − x )
2
    I
100 π 2
sin x απ 3 π2 π2 ⇒
2
= log e 2− ∫
20 − 2x
∫ dx = , α ∈R, x
68 If e −1 − 1 ⋅ dx
 x  x 
1 + 4π 2 B=
 −   −  1 − x2
π  4 + 2 
0  π  π  1
e  π  1 
1
1 
= log e 2 + ∫  − 1 dx
I
where [x] is the greatest integer ⇒
∴ A −B 2 
20 1− x 2 
less than or equal to x, then the 
 
value of α is −1
 (e −1 − 1)  ⇒
I 1 −1
= log e 2 + [sin x − x] 0 1
[2021, 22 July Shift-II] ⇒ [−π (e − 1)] −   2 2
 −π  4 + 1  
(a) 200(1 − e −1)   π2  
I 1 π 
⇒ = log e 2 +  − 1
(b) 100(1 − e) 2 2 2 
π (e −1 − 1)
(c) 50(e − 1) = −π (e −1 − 1) + π
(4 π2 + 1) ∴ I = log e 2 + − 1
(d) 150 (e −1 − 1) 2
π π /2
70 Let a be a positive real number π 1
a
such that ∫ e x − [x] dx = 10e − 9,
2 ⇒ g (1) = ∫ cosf (x)dx
= ∫ (− 2)dx = − 2x] − π 2
2
−π / 2
0
π 2 π /2

where [x] is the greatest integer
less than or equal to x. Then, a is
2
[Q[− x] + [x] = −1, if x ∉1]
⇒ 2 g (1) = ∫ cosf (x)dx
−π / 2
equal to [2021, 20 July Shift-I]
 π  π π /2
2I = (− 2)   + (2)  −  = − 2 π
(a) 10 − log e (1 + e) 2  2 and g (0) = ∫ cosf (x) dx
(b) 10 + log e 2 −π / 2
⇒ I=− π ∴ 2 g (1) = g (0)
(c) 10 + log e 3
(d) 10 + log e (1 + e)
72 If f : R → R is given by f (x) = x + 1 , 74 Let f (x) and g (x) be two functions
Ans. (b)
a then the value of satisfying f (x 2 ) + g (4 − x) = 4x 3 and
x − [x ]
∫e dx = 10 e − 9 1  5 10  5(n − 1)  
lim f (0) + f   + f   +....+f  
g (4 − x) + g (x) = 0, then the value of
4
 n  n 
0
n→∞ n   n 
Let x = I + f and n ≤ a < n + 1
is
∫ ∫ (x) 2dx is ……… .
[x] = I, [a] = n [2021, 20 July Shift-II] −4
[2021, 18 March Shift-I]
1
x −0
2 3 (a) 3/2 (b) 5/2
∫e dx + ∫ e x − 1dx + ∫ e x − 2dx + .. + (c) 1/2 (d) 7/2
Ans. (512)
0 1 2
Ans. (d) Given,f (x2 ) + g (4 − x) = 4x 3
n a
x −n+ 1 and g (4 − x) + g (x) = 0
∫e dx + ∫ e x − ndx Let 4
n−1 1  5  5(n − 1)   Let I = ∫ f (x2 ) dx
I = f (0) + f   + f   + .. + f 
n 10

 e2 − e 1   e 3 − e2  n  n   n   n   −4
(e 1 − e 0 ) +   +  2
 n−1 4
 e   e   5r  1
 en − en − 1   ea − en 
⇒ I= ∑ f  n  ⋅ n = 2 ∫ f (x2 )dx (even function)
r =0
+ K+  n−1
 +  n
 0
 e   e  1 4
⇒ I = ∫ f (5x)dx (Qassume r/n as x) ⇒ I = 2⋅ ∫ [4x 3 − g (4 − x)] dx
a −n 0
⇒ n (e − 1) + [e − 1] = 10 e − 10 + 1 1 0
⇒ n = 10 ⇒ e a − 10 − 1 = 1 = ∫ (5x + 1)dx (Qf (x) = x + 1 ) 4 4
0
⇒ e a − 10 = 2 ⇒ a − 10 = log e 2 1 = 8 ∫ x 3 dx − 2 ∫ g (4 − x) dx
5  5 7
∴ a = 10 + log e 2 =  x2 + x  = + 1 − 0 = 0 0
 2 0 2 2
x 4 
4

71 If [x] denotes the greatest integer π = 8   − 2I 1 = 2(44 − 0 4 ) − 2I 1


2 cos
π
less than or equal to x then the 73 Let g (t) = ∫  t + f (x) dx,  4 0

π 4  = 29 − 2I 1
value of the integral 2
4
π
where f (x) = log e (x + x 2 + 1), x ∈R. where, I 1 = ∫ g (4 − x) dx
∫ −
2
π [[x] − sin x]dx is equal to
0
2 [2021, 20 July Shift-II]
Then, which one of the following is 4
correct ? [2021, 20 July Shift-II] Now, I 1 = ∫ g (4 − x) dx …(i)
(a) −π (b) π (a) g (1) = g (0) (b) 2g (1) = g (0) 0
4
(c) 0 (d) 1 (c) g (1) = 2g (0) (d) g (1) + g (0) = 0
⇒ I 1 = ∫ g [4 − (0 + 4 − x] dx
Ans. (a) Ans. (b) 0
π
2
We have,  b b 
π /2 Q∫ f (x) dx = ∫ f (a + b − x) dx 
I= ∫ ([x] + [− sin x])dx ...(i)
g (t) = ∫
π
cos  t + f (x)  dx  a 

π 4  a
−π / 2 4
2
b b ⇒ I 1 = ∫ g (x) = dx …(ii)
Using property, ∫ f (x)dx = ∫ f (a + b − x)dx π
 π π 
cos  t cos(f (x)) − sin t sinf (x) dx
0
g (t) = ∫ 2
a a −
π  4 4  Adding Eqs. (i) and (ii),
π 2
4
2
Given, f (x) = log(x + 1 + x2 ) 2I 1 = ∫ [g (x) + g (4 − x)] dx
I= ∫ ([− x] + [sin x])dx ...(ii)
0

π f (− x) = log (− x + 1 + x2 ) = − log (x + 1 + x2 )
2 ⇒ 2I 1 = 0
Add Eqs. (i) and (ii), f (− x) = − f (x) ⇒ I1 = 0
π f (x) is an odd function. (Qg (x) + g (4 − x) = 0, given)
2 π /2
π ∴ I = 29 − 2I 1
2I = ∫ ([x] + [− x]) + ([sin x] + [− sin x])dx ∴g (t) = ∫ cos t cosf (x)dx
4 ⇒ I = 29 = 512
π −π / 2

2
[Qsinf (x) is an odd function]
π /3
75 Let P (x) be a real polynomial of sinα ( π / 2 − x) [using expansion of cosx i.e.,
degree 3 which vanishes at x = − 3.
g (α) = ∫ α π  π 
dx
x2 x 4 x 6 x 8
π / 6 cos  − x  + sinα  − x  cos x = 1 − + − + − …]
Let P (x) have local minima at x = 1, 2  2  2! 4! 6! 8!
local maxima at x = − 1 and π /3
cosα x  1 1 1 1 
⇒ I =2 1− 1+ − + − + …
1 g (α) = ∫ cos x + sinα x
α
dx … (ii)  2! 4! 6! 8! 
∫ P (x) dx = 18, then the sum of all the π/6

Adding Eqs. (i) and (ii), ⇒


1 1
I = 2 − +
1 
− …
−1  2 24 720 
π /3
coefficients of the polynomial P (x) sinα x + cosα x
Now, I < 2 −
1 1 1 
is equal to …………… .
2g (α) = ∫ sinα x + cosα x
dx +
 2 24 720 

π/6
[2021, 18 March Shift-II]
π /3  1 1 
⇒ I < 1 − + 
Ans. (8) 2g (α) = ∫ 1⋅dx  12 360 
Let P ′ (x) = a (x − 1) (x + 1) π/6 360 − 30 + 1
⇒ I<
⇒ P ′ (x) = a (x2 − 1) = [x] ππ // 36 360
∴ P (x) = a ∫ (x2 − 1) dx π π π
= − = ⇒ I<
331
… (ii)
 x3  3 6 6 360
⇒ P (x) = a  − x  + C π
∴ g(α) =
Also, I > 2 − 
3  1 1
12  2 24 
According to the question, P (− 3) = 0 g(α) is constant function.
 1
a  − + 3 + C = 0 ⇒ I > 1 − 
27
∴It is even function.  12 
 3 
11
⇒ − 6a + C = 0 … (i) 77 Let f : R → R be defined as ⇒ I>
12
  x3
1
 
Now, ∫  a  − x  + C  dx = 18 (given) f (x) = e − x sin x. If F : [0, 1] → R is a 11 × 30
 3   ⇒ I>
−1   differentiable function, such that 12 × 30
x
⇒ 2C = 18
F (x) = ∫ f (t) dt, then the value of ⇒ I>
330
…(iii)
⇒ C =9 … (ii) 360
0
From Eqs. (i) and (ii), 1 From Eqs. (ii) and (iii), we get,
∫ [F ′ (x) + f (x)] e
x
− 6a + 9 = 0 dx lies in the 330 331
<I<
3 0 360 360
⇒ a=
2 interval [2021, 17 March Shift-II]
3  x3 
 327 329   330 331 
78 If the integral
∴ P (x) =  − x  + 9 (a) , (b) , 10 1
2 3   360 360   360 360  [sin2πx] −1

1 3  331 334   335 336 


∫ e x − [x] dx = αe + β e 2 + γ,

∴Sum of the all coefficient = − + 9= 8 (c) , (d) , 0


2 2  360 360   360 360  where α, β, γ are integers and [x]
Ans. (b) denotes the greatest integer less
76 Which of the following statements than or equal to x, then the value of
Given,f (x) = e − x ⋅ sin x
is correct for the function g(α) for x α + β + γ is equal to
α ∈R, such that and F (x) = ∫ f (t) dt
0 [2021, 17 March Shift-II]
π
Q F (x) is differentiable function. (a) 0 (b) 20 (c) 25 (d) 10
sin α x
3
∴F ′ (x) = f (x) × 1 − f (0) × 0 (using
g (α) = ∫ α α
dx Ans. (a)
π cos x + sin x
Newton-Leibnitz rule) 10 [sin2 πx ]
⇒ F ′ (x) = f (x) … (i) Let I = ∫ dx
6
1
0 e x − [x]
[2021, 17 March Shift-I] Let I = ∫ [F ′ (x) + f (x)] e x dx 10 [sin2 π x]
0 ⇒ I=∫ dx [Qx − [x] = { x }]
(a) g(α) is a strictly increasing function. 1 1
0 e {x }
1 = ∫ [f (x) + f (x)] e x dx = ∫0 2⋅f (x) ⋅ e
x
dx
(b) g(α) has an inflection point at α = − . 0 From above integrand, we observe that
2 [from Eq. (i)] [sin2πx]
is a periodic function with
(c) g(α) is a strictly decreasing function. 1 e {x }
⇒ I = 2⋅ ∫ f (x) ⋅ e x dx
(d) g(α) is an even function. 0 period ‘1’.
1 1 [sin2 πx]
Ans. (d) = 2⋅ ∫ e − x sin x ⋅ e x dx ∴ I = 10 ∫
π /3
0 0 e {x }
sinα x 1
g (α) = ∫ cosα x + sinα x dx … (i) = 2∫ sin xdx = 2 [− cos x] 10
0
[by the property of definite integral,
nT T
∫0 f (x) dx = n ∫ f (x) dx
π/6
b b = 2[− (cos 1 − cos0)] = 2(1 − cos 1) 0
Applying ∫ f (x) dx = ∫ f (a + b − x) dx ⇒ where f (x) is a periodic function with
a a   (1)2 (1) 4 (1) 6 (1) 8  period =T]
I = 2⋅  1 −  1 − + − + − …
  2! 4! 6! 8! 
1 [sin2 πx] From Eqs. (i) and (ii), Ans. (406)
⇒ I = 10 ⋅ ∫ dx
0 ex 20 I 10 − 10 I 9 − 9I 8 = 0 x
Given, y (x) = ∫ (2t2 − 15t + 10) dt
[Q{ x } = x, 0 ≤ x < 1 ] 0
⇒20I 10 = 10I 9 + 9I 8 comparing this to
⇒ ⇒ y ′ (x) = 2x2 − 15x + 10
 [sin2 πx]
1/ 2 [sin2 πx]  1 20 (I 10 ) = αI 9 + βI 8 , we get
I = 10  ∫ dx + ∫ dx  Equation of normal ⇒x + 3y = − 5
 0 ex 1/ 2 ex  α = 10 , β = 9 −1 −1
=
(− 1)  ∴ α −β = 1 [y ′ (x)] a , b
⇒ I = 10  ∫
1 / 2 0 1 3
dx + ∫ dx 
 0 ex 1/ 2 e x 
80 Let f : (0, 2) → R be defined as or [y ′ (x)] a , b = 3
2a2 − 15a + 10 = 3
 πx 
1 f (x) = log 2 1 + tan  . ⇒ 2a2 − 15a + 7 = 0
  4  ⇒ (2a − 1) (a − 7) = 0
2 1  2  1
0 1/2 1 Then, lim f   + f   + K+f (1) ∴ a = or 7
n→∞ n   n  n 2
–1 
As, a > 1, so, a = 7
is equal to……… .
Now, (7, b) lies on y (x),
[2021, 16 March Shift-I] a
∴ b = ∫ (2t2 − 5t + 10) dt
⇒ I = 10  0 − ∫ e − x dx 
1
Ans. (1) 0
 1 /2 
 πx 
f (x) = log2 1 + tan 
2 15
e− x 
1
 ⇒ b = a 3 − a2 + 10a
⇒ I = − 10   4   3 2

 − 1  1/ 2 2  1  2 3 15 2
f   + f   + … + f (1)
2 ⇒ b = (7) − (7) + 10 (7)
= lim
⇒ I = 10 [e − 1 − e − 1/ 2 ] n→ ∞ n 
  n  n  3 2
−1 n − 413
 1  r ⇒ b=
⇒ I = 10 e − 1 − 10 e 2 = 2 lim
n→ ∞
∑  n  f  n  6
r =1
−1  413 
 πx  So, a + 6b = 7 − 6  = − 406
log n 1 + tan  dx … (i)
−1 2 1
⇒ I = 10 e + (− 10) ⋅ e 2
+0  6 
log n 2 ∫0
… (i) Let I =
  4  
−1
∴ | a + 6b | = 406
Comparing Eq. (i) by αe − 1 + βe 2 + γ, we b b
as, ∫ f (x) dx = ∫ f (a + b − x) dx
82 Let f : R → R be a continuous
a a
get
α = 10, β = − 10 and γ = 0 So, x → 1 − x
π function such that f (x) + f (x + 1) = 2,
log n  1 + tan (1 − x)  dx
Hence,α + β + γ = 10 − 10 + 0 2 1
I= ∫  
8
log n 2 4 for all x ∈R. If I 1 = ∫ f (x)dx and
0
⇒ α + β + γ =0
 π πx  
log n 1 + tan −
2 1

log n 2 ∫0
e =
  dx 0
79 Let I n = ∫ x 19 (log | x |) n dx, where 4 4   3

1 2 1   1 − tan πx / 4   I 2 = ∫ f (x)dx, then the value of


n ∈ N. If (20) I 10 = αI 9 + βI 8 , for
= ∫ 1 + 
log n   dx −1
log n 2   1 + tan πx /4  
0
I 1 + 2I 2 is equal to……… .
natural numbers α and β, then α − β   [2021, 16 March Shift-I]
2 1  2 
is equal to …………… .
[2021, 17 March Shift-II]
=
log n 2 ∫ 0
log n 
 1 + tan π x
 dx Ans. (16)
 4  Given,f (x) + f (x + 1) = 2
Ans. (1) 8
πx

log n 2 − log n  1 + tan  dx
2 1
I 1 = ∫ f (x)
log n 2 ∫0
e
Given, I n = ∫ x 19 (log | x |) n dx =
1  4  0
⇒ … (ii) 3

 x20 
e and I2 = ∫ f (x) dx
e (ln | x |) n − 1 x20 Adding Eqs. (i) and (ii), we get
In =  (ln | x |n  − ∫ n⋅ ⋅ dx 2 1 −1
 20 1
1 x 20 2I =
log n 2 ∫0
log n 2 dx
Let f (0) = a
I=1 f (0) + f (1) = 2
(using integration by parts)
f (1) = 2 − a
e20 n e
⇒ In = − ∫1 (ln | x |) n − 1 ⋅ x 19dx 81 If the normal to the curve f (1) + f (2) = 2
20 20 x
f (2) = a and so on
e 20
n y (x) = ∫ (2t 2 − 15t + 10)dt at a point
⇒ In = − ⋅I n − 1 So, f (0) = f (2) = f (4) … = a
20 20 0
f (1) = f (3) = f (5) … = 2 − a
⇒ 20I n + nI n − 1 = e 20 (a, b) is parallel to the line
Clearly, f (x) is periodic with its period 2
Put n = 10 and n = 9, we get x + 3y = − 5, a > 1, then the value of units.
20 I 10 + 10 I 9 = e20
| a + 6b| is equal to……… . 2. 4
… (i)
[2021, 16 March Shift-I] So, I 1 = ∫ f (x) dx
and 20 I 9 + 9I 8 = e20 … (ii) 0
2
Then, ∞ tn − 1 + tm − 1
⇒ I 1 = 4 ∫ f (x) dx 2I mn = ∫ dt
f ( e) = ∫
e log e t
dt …(i)
0 (t + 1) m + n
0
3 1 1+ t 1 tn − 1 + t m −1
Now, I2 = ∫ f (x) dx 1 2I mn = ∫ dt
and f   = ∫
1 e
log e t
dt …(ii)
0 (t + 1) m + n
−1
e 1 1+ t ∞ t n − 1 + t m −1
x→ x + 1 +∫ dt
4 4 1 1 (t + 1) m + n
Let t = and put in Eq. (ii), we get
I2 = ∫ f (x + 1) dx = ∫ [2 − f (x)] dx u ∞ tn − 1
+ t m −1
Let I 1 = ∫
log  
0 0 1 dt
2 (t + 1) m + n
1
⇒ I2 = 8 − 2∫ f (x) dx  1 e  u  −1
f  = ∫ ⋅ du 1 −1
0
 e  1 1 + 1 u2 Let t = , then dt = 2 dz
2 u z z
⇒ 2I2 = 16 − 4∫ f (x) dx n− 1 m −1
=∫
e log u
du  1 +  
1
1 u (u + 1)  
0 0 z z 1
⇒ 2I2 = 16 − I 1 I 1 = ∫ (−1) m+n
. dz
Using change of variable 1
 1 + 1 z2
∴ I 1 + 2I2 = 16  
z 
f   = ∫
1 e log t
dt …(iii)
 e  1 t (t + ` 1) 0 zn − 1 + zm − 1
83 Consider the integral = −∫ dz
10 From Eqs. (i) and (iii), we get 1 (z + 1) m + n
[x] e [x]
I = ∫ x − 1 dx,
f (e) + f   = ∫
1 e log t e log t Put I 1 in 2I m , n
dt + ∫ dt
0 e e 1 1+ t 1 t (1 + t) 1 tn − 1 + tm − 1 0 zn − 1 + zm − 1
2I mn = ∫ dt− ∫ dz
where [x] denotes the greatest logt e 0 (t + 1) m + n 1 (z + 1) m + n
=∫ dt
integer less than or equal to x . t 1
1 tm − 1 + tn − 1
Then, the value of I is equal to 1 = 2∫ dt
Take log t =`v, then dt = dv 0 (t + ` 1) m + n
[2021, 16 March Shift-II] t
1 ⇒ α=1
(a) 9 (e − 1) (b) 45 (e + 1)  v2 
f (e) + f   =` ∫ vdv =   =
1 1 1
100
(c) 45 (e − 1) (d) 9 (e + 1) e 0
86 The value of ∑
n x − [x]
Ans. (c)
 2 0 2
∫n −e1 dx, where
f (e) + f   =
1 1 n= 1
10 10 ∴
[x] e [x] [x] e [x] e 2
We have, ∫ x −1
dx = e ∫ dx [x] is the greatest integer ≤ x, is
e ex 1 m−1 n− 1
85 If I m⋅ n = ∫
0 0
1 2 3 x (1 − x) dx, for [2021, 26 Feb. Shift-I]
0 e 2e2 0 (a) 100 (e − 1)
=e∫ dx + e ∫ x dx + e ∫ x dx + …
e x
e e m, n ≥ 1 and (b) 100 (1 − e)
m−1
0 1 2
1x + x n− 1
b
−x e −x b
⇒(e − a − e − b )
∫0 (1 + x) m + n
dx = αI m⋅ n , α ∈R, (c) 100 e
(d) 100 (1 + e)
⇒ ∫ e dx = –1
a a
then α equals _____ . Ans. (a)
2
⇒ e  − 2  + 2e 3  2 − 3 
1 1 1 1 [2021, 26 Feb. Shift-II] Let ‘x’ be any real number, then
e e  e e  Ans. (1) x = [x] + { x }, where [x] is integer part of x
+3e 4  3 − 4  + ...+ 9e 10  9 − 10 
1 1 1 1 1 and { x } is fractional part of x.
e Given,I mn = ∫ x m − 1 (1 − x) n − 1dx
e  e e  0 Then, x − [x] = { x }, Also period of { x } = 1
100 n 100 n
= (e − 1) + 2(e − 1) + 3(e − 1) +…+9(e − 1) Using substitution put
Now, ∑ ∑ ∫e
x − [x ]
∫e dx = {x }
1 dx
= (1 + 2 + 3 + … + 9)(e − 1) x= n=1 n−1 n=1 n−1
 9 × 10  t+1
=  (e − 1) −1 [Difference between upper and lower
 2  Then, dx = dt limit is 1 unit]
= 45(e − 1) (t + 1)2 1 2 100
tn − 1 = ∫ e { x }dx + ∫e dx + … + ∫e
{x } {x }
a 1 1 dx
x
log e t I mn = ∫ (−1) ⋅ ⋅ dt
84 For x > 0, if f (x) = ∫ dt, then
∞ (t + 1) n − 1 (t + 1) n − 1 (t + 1)2 0 1 99

(1 + t) 0 tn − 1 = e x ] 10 + e (x − 1) ]21 + … + e (x − 99 ) ] 100
1 = −∫ dt …(i) 99

f (e) + f   is equal to
1 ∞ (t + 1) m + n = (e − 1) + (e − 1) + … + (e − 1)
 e Similarly, = 100 (e − 1)
[2021, 26 Feb. Shift-II]
(a) 1 (b) –1
1
I mn = ∫ x n − 1 (1 − x) m − 1dx …(ii) π /2 cos 2 x
1 0 87 The value of ∫ dx is
(c) (d) 0 − π /2 1+ 3x
∞ tn − 1
2 ⇒ I mn = ∫
0 (t + ` 1) m + n [2021, 26 Feb. Shift-I]
Ans. (c) π π
x log From Eqs. (i) and (ii), we get (a) (b) 4π (c) (d) 2π
t
f (x) = ∫ e
dt 4 2
1 (1 + t)
Ans. (a)  −1 2  π /2
π /2 = 3  ∫ ( x2 − x − 2)dx + ∫ (− x2 + x + 2)dx  91 If I n = ∫ cot n x dx, then
cos x 2
−2 π /4
Let I = ∫ dx … (i)  −1 
1 + 3x [2021, 25 Feb. Shift-II]
− π /2  3 
−1
 x 3 x2 
2 
x x2 1 1 1
Using the property, = 3  − − 2x  −  − − 2x   (a) , , are in AP
 3 2  −2  3 2  −1  I2 + I 4 I 3 + I 5 I 4 + I 6
b b  
∫a f (x) dx = ∫a f (a + b − x) dx = 19
(b) I2 + I 4 , I 3 + I 5 , I 4 + I 6 are in AP
π /2 1 1 1
cos ( π / 2 − π / 2 − x)
2
1
I= ∫ 1 + 3π / 2 − π / 2 − x
dx
90 lim  +
n
+
n
+K
(c) , ,
I2 + I 4 I 3 + I 5 I 4 + I 6
are in GP
− π /2
π /2
n→ ∞  n
 (n + 1) 2
(n + 2) 2 (d) I2 + I 4 , (I 3 + I 5 )2 , I 4 + I 6 are in GP
cos2 x
= ∫ dx [Qcos(− x) = cos x]
1 + 3− x n  Ans. (a)
− π /2 +  is equal to π
π /2
3x cos2 x
(2n − 1) 2  2
I= ∫ dx …(ii) I n = ∫ cotn xdx
(1 + 3x ) [2021, 25 Feb. Shift-II]
− π /2 π
1 1 1
(a) 1 (b) (c) (d) 4
Adding Eqs. (i) and (ii), 2 3 4 π
π /2 π /2 2
cos2 x 3x cos2 x Ans. (b)
= ∫ cotn −2 x (cot2 x)dx
2I = ∫ 1 + 3x
dx + ∫ 1 + 3x
dx
1 + n
+
n
+K π
− π /2 − π /2
 n (n + 1)2 (n + 2)2  4
π /2
(1 + 3x ) cos2 x
π /2
Let L = lim   π π
= ∫ 1 + 3 x
dx = ∫ cos2 x dx n→ ∞ 
+
n
 (2n − 1)2


2
n −2
2
− π /2 − π /2
  I n = ∫ cot xcosec xdx − ∫ cotn −2 xdx
2
π /2 π π
1 + cos2x or
= ∫ 2
dx
 n n 
4 4
− π /2 + + K+ π
[Qcos2x = 2 cos2 x − 1 ]  (n + 0)2 (n + 1)2  2
1 sin2x  1
π /2 L = lim   I n + I n − 2 = ∫ cotn −2 x.cosec2xdx
= x+ = [ π] n→ ∞  n n n 
+ −
2  2  − π / 2 2  (n + n − 1)2 (n + n)2 (n + n)2  π
  4
⇒ 2I = π /2  n n n  Now, let cotx = u, then cosec2xdx = − dt,
π = lim  + +K +  0
⇒ I= n→ ∞ (n + 0) 2
 (n + 1) 2
(n + n)2  limit will be I n + I n − 2 = ∫ − t n − 2dt
4 1
 n  n−1
0
− lim   − (t)  0 (1) n − 1 
88 The value of the integral n→ ∞ (n + n) 2 =  = − − 
π   n − 1 1 n − 1 n − 1 
∫ | sin 2x | dx is ……… .
n
0 = lim Σ n
− lim
1
I n + I n −2 =
1
[2021, 26 Feb. Shift-I] n→ ∞ r =0 (n + r)2 n→ ∞ 4n
n− 1
n
Σ − 0 since, lim = 0 
Ans. (2) n 1 Now, put n = 4
= lim
π n→ ∞ r =0 (n + r)2  n→ ∞ n  1 1
Let I = ∫ | sin2x | dx ⇒ I2 + I 4 = , then =3 …(i)
0
Now, for solving limit summation, we 3 I2 + I 4
π /2
= 2∫ | sin2x | dx integrate it using some replacement. Put n = 5
0 n
L = lim Σ
1 1 1
[Qsin2x is periodic function] ⇒ I 5 + I 3 = , then =4 …(ii)
π /2 n→ ∞ r = 0 n(1 + r / n) 4 I3 + I5
=2∫ sin2x dx [sin2x is positive in
0 r 1
Take as x and as dx. Put n = 6
range (0, π /2)] n n 1 1
 − cos2x 
π /2 ⇒I 6 + I 4 = , then =5 …(iii)
=2 Lower limit is obtained by putting r = 0 in 5 I4 + I6
 2  r
0 , we get Lower limit = 0
n Here, from Eqs. (i), (ii) and (iii), we
= − [cos π − cos0] conclude
Upper limit is obtained by putting r = n in
= − (− 1 − 1) = 2 r 1 1 1
, we get , and are in AP with
I =2 n I2 + I 4 I 3 + I 5 I4 + I6
2
Upper limit = 1 common difference 1.
89 The value of ∫ |3x 2 − 3x − 6| dx is 1 1 −1 
1
−2 ∴ L=∫ dx = 1 3

………. . 0 (1 + x) 2 (1 + x)  0 92 The value of ∫ x 2 e [x ] dx, where [t]


[2021, 25 Feb. Shift-II] −1
Ans. (19) 1  1 denotes the greatest integer ≤ t, is
= −  − 1 =
2 2  2 [2021, 25 Feb. Shift-I]
∫−2|3x − 3x − 6|dx = I (say)
2
e−1 e+1 e+1 1
1 (a) (b) (c) (d)
2
∴ L= 3e 3 3e 3e
I = 3∫ |x2 − x − 2|dx 2
−2
Ans. (c) (a) − 2 − 3 + 1 (b) − 2 − 3 − 1 1
= ∫ |1 − 2x | dx +
2

1 2 [x 3 ] 0 ∫1 dx
Given, ∫ x e dx, where [t] is greatest (c) − 5 (d) − 4 1/ 2 1
−1 =∫ (1 − 2x) dx + ∫ (2x − 1) dx + 1
Ans. (b) 0 1/ 2
integer function. 3 = [x − x2 ] 10/ 2 + [x2 − x] 11/ 2 + 1
Q [x 3] = 0 ∀ x ∈ (0, 1) Let I = ∫ [x − 2x − 2] dx
2
1 1 1 1 1 1 3
and [x 3] = − 1 ∀ x ∈ (− 1, 0) 3
= − + 1 − 1 − + + 1 = + 1 = = 1.5
= ∫ [x2 − 2x + 1 − 3] dx 2 4 4 2 2 2
1 2 [x 3 ] 0 1 2 0
So, ∫− 1
x e dx = ∫ x2 e − 1dx + ∫0 x e dx 1
−1 3
= ∫ (x − 1)2 − 3]dx 97 Let [t] denote the greatest integer
1 0 1 2
∫− 1 ∫0
1
= x2dx + x dx 3 3 less than or equal to t. Then the
e = ∫ [(x − 1)2 ] dx + ∫1 − 3dx 2
1 x x 3
0
3
1 1 value of ∫ |2x − [3x]| dx is ……… .
1
= ×  +   Put x − 1 = t; dx = dt when x = 1 + = 0 and
e  3 − 1  3 0 x = 3t = 2 [2020, 2 Sep. Shift-II]
2 Ans. (1)
1 
× 0+ +   ∴ I= − 3[x] 31 + ∫0 [t ] dt
1 1 2
= 2
The integral,I = ∫ |2x − [3x]| dx
e  3   3  1
1 2 3 2 4/ 3 5/ 3
=
1
+ = 
1 1+ e

= −6+ ∫0 0 dt + ∫1 1dt + ∫ 2 2 dt + ∫ 3
3dt =∫ |2x − 3 | dx + ∫4 / 3 |2x − 4 |dx
3e 3  3e 
1
= − 6 + (0) + ( 2 − 1) + 2( 3 − 2) + 3(2 − 3) 2

= −6+ 2 − 1+ 2 3 −2 2 + 6−3 3 =∫
4/ 3
(3 − 2x) dx
+ ∫1 |2x − 5 |dx
93 Let f (x) be a differentiable function 1
I = − 1− 2 − 3 5/ 3 2
defined on [0, 2], such that + ∫4 / 3 (4 − 2x) dx + ∫5 / 3 (5 − 2x) dx
f ′ (x) = f ′ (2 − x), for all x ∈ (0, 2), f (0) = 1 a
= [3x − x2 ] 41 / 3 + [4x − x2 ] 54 // 33 + [5x − x2 ]25 / 3
and f (2) = e 2 . Then, the value of 95 If ∫ (| x | + | x − 2 |)dx = 22, (a > 2) and
2 −a  16   20 25 16 16 
=  4 − − 3 + 1 +  − − + 
∫ f (x) dx is [x] denotes the greatest integer
 9   3

9 3 9
25 25 
0 [2021, 24 Feb. Shift-II] −a +  10 − 4 − + 
 9
∫ (x + [x]) dx is equal to
(a) 1 − e2 (b) 1 + e2 3
≤ x, then  20 16 25 
(c) 2 (1 − e2 ) (d) 2 (1 + e2 ) = (2 + 6) +  − −  +
a  3 3 3
Ans. (b) ......... .  16 25 16 25 
Given, f(0) = 1 … (i) [2021, 24 Feb. Shift-I]
+ − − + + 
 9 9 9 9
f (2) = e2 21
… (ii) Ans. (3) =8− =8−7=1
f ′ (x) = f ′ (2 − x) a 3
Given, ∫ ( | x | + | x − 2 | ) dx = 22 Hence, answer 1.00 is correct.
Integrating w.r.t. x, −a

f (x) = − f (2 − x) + C 0 2 π

Put x = 0
⇒ ∫− a (− 2x + 2) dx + ∫0 (x + 2 − x) dx 98 ∫− π | π − | x || dx is equal to
a [2020, 3 Sep. Shift-I]
f (0) = − f (2) + C + ∫2 (2x − 2) dx = 22
π2
⇒ 1 = − e2 + C [from Eqs. (i) and (ii)] (a) 2 π2 (b) 2 π2 (c) π2 (d)
⇒ (x − 2x)2
|−0 a
+ (2x) + (x − 2x) = 22 |20 2
|2a 2
⇒ C = 1 + e2 Ans. (c)
⇒ a2 + 2a + 4 + a2 − 2a − (4 − 4) = 22
∴ f (x) = − f (2 − x) + 1 + e2 π
⇒ 2a2 = 18 Given integral ∫ |π − | x | | dx
or f (x) + f (2 − x) = 1 + e2 …(iii) −π
⇒ a2 = 9 π π
= 2 ∫ |π − | x | | dx = 2∫ |π − x | dx
2
Let I = ∫ f (x) dx …(iv) ⇒ a =3 0 0
0
−a −3 π
= 2 ∫ ( π − x) dx [Qx∈(0, π)]
∫a x + [x] dx = ∫
2
Also, I = ∫ f (2 − x) dx …(v) ∴ (x + [x]) dx 0
3
0 π
3  x  2
π  2
Now, adding Eqs. (iv) and (v), = − ∫ (x + [x]) dx = 2  πx −  = 2  π2 − 
−3 2 2 
2
2I = ∫ [f (x) + f (2 − x)] dx  0 
0 = − [− 3 − 2 − 1 + 1 + 2]  π2 
2
= − [− 3] = 3 = 2   = π2
2I = ∫ (1 + e ) dx 2
[from Eq. (iii)]  2 
0
∴Required value is 3.
2I = 2(1 + e ) 2 Hence, option (c) is correct.
2
∴ I = (1 + e2 )
96 The integral ∫ || x − 1| − x| dx is equal 99 If the value of the integral
94 The value of the integral 0 1/ 2 x2 k
3 to [2020, 2 Sep. Shift-I] ∫0 (1 − x 2 ) 3/ 2 dx is 6 , then k is
∫ [x − 2x − 2] dx, where [x] denotes
2
Ans. (1.5) 2
1 Given integral ∫ | | x − 1 | − x |dx equal to [2020, 3 Sep. Shift-II]
0
the greatest integer less than or 1 2 (a) 3 2 + π (b) 2 3 − π
= ∫ |− (x − 1) − x | dx + ∫1 | x − 1 − x | dx
equal to x, is [2021, 24 Feb. Shift-II] 0 (c) 2 3 + π (d) 3 2 − π
Ans. (b) Ans. (21) Ans. (a)
1/ 2 2 n 1 π /2
x 1
The integral, I = ∫ dx Since, ∫ { x }dx = n∫ xdx Given integral, I = ∫ dx …(i)
0
(1 − x2 ) 3/ 2
0 0 −π /2
1 + e sin x
π (as n ∈N , n > 1 )
Let x = sinθ, So, θ(0) = 0 and θ  =
1 b

 2 6 1 As we know that, ∫ f (x)dx


x  n 2

and dx = cosθdθ = n  = a

π/6  2 0 2 b
sin θ 2
= ∫ f (a + b − x)dx, so
∴ I= ∫ cosθ dθ n 1 2 3

0
(cos2 θ) 3/ 2 and ∫ [x]dx = ∫ 0 dx + ∫ 1dx + ∫ 2dx a
π /2 π /2
π/6 0 0 1 2
1 e sin x
= ∫ tan θ dθ
2 n I= ∫ 1+ e − sin x
dx = ∫ sin x
+1
dx …(ii)
0
+ ...+ ∫ (n – 1)dx −π /2 −π /2
e
π /6 n –1 On adding Eqs. (i) and (ii), we get
= ∫ (sec θ − 1)dθ =
2
[tanθ − θ] π0 / 6 π /2 π /2
1 + e sin x
0 = 0 + 1(2 − 1) + 2(3 − 2) + ....+ (n − 1)(n − (n − 1)) 2I = ∫ 1+ e sin x
dx = ∫ dx = π
π k1 n(n – 1) −π /2 −π /2
= − = [given] = 1 + 2 + 3 + ...+ (n − 1) =
3 6 6 2 π
⇒ I=
∴ k = 2 3 − π. n n(n − 1) 2
Q , , 10 n(n – 1) are in GP
2 2 1
Hence, option (b) is correct.
⇒ ,
1 n–1
, 10 (n – 1) are also in GP
104 If I 1 = ∫ (1 − x 50 ) 100 dx and
0
100 Let f (x) = | x − 2| and 2 2 1
[as n> 1] I 2 = ∫ (1 − x 50 ) 101 dx such that
g (x) = f (f (x)), x ∈ [0, 4]. Then 2
0
 n − 1 I 2 = αI 1 , then α equals to
3 ⇒  = 5(n – 1)
∫ (g (x) − f (x)) dx is equal to
0
[2020, 4 Sep. Shift-I]
 2  [2020, 6 Sep. Shift-I]
⇒n− 1 = 20 [as n> 1] (a)
5049
(b)
5050
3 1
(a) (b) (c) 0 (d) 1 ⇒ n = 21. 5050 5049
2 2 5050 5051
Ans. (d) (c) (d)
102 The integral 5051 5050
π /3
Given functionf (x) = | x − 2 |
∫ tan x ⋅ sin 3x (2 sec x ⋅ sin 3x
3 2 2 2
Ans. (c)
 x − 2, x ≥ 2 π /6
= , x ∈[0, 4] 1

2 − x, x < 2 + 3 tan x ⋅ sin6x) dx is equal to It is given thatI 1 = ∫ (1 − x 50 ) 100 dx


[2020, 4 Sep. Shift-II] 0
and g (x) = f (f (x)) = |f (x) − 2 | 1 7 1 9 1
(a) − (b) (c) − (d)
f (x) − 2, f (x) ≥ 2 9 18 18 2 and I2 = ∫ (1 − x ) dx 50 101
=
2 − f (x), f (x) < 2
0
Ans. (c) 1
2 − (2 − x), 0 ≤ x < 2 Given integral = ∫ (1 − x 50 )(1 − x 50 ) 100 dx
=
2 − (x − 2), 2 ≤ x ≤ 4
π/3 0
I= ∫ tan x ⋅ sin2 3x (2 sec2 x sin2 3x
3
1 1
 x, 0≤ x<2
= π/6 = ∫ (1 − x 50 ) 100 dx − ∫ x ⋅ x 49 (1 − x 50 ) 100 dx
4 − x, 2 ≤ x ≤ 4 + 3 tan x ⋅ sin6x)dx 0 0
3 π/3 1
∴ ∫ (g (x) − f (x))dx =∫ [2 tan3 x sec2 x sin4 3x Now, let I = ∫ x ⋅ x (1 − x 50 ) 100 dx
49
π/6
0 + 3 tan x (2 sin 3x cos3x)]dx
4 3 0
2 3
π/3  d  tan4 x sin4 3x   = [x ∫ x 49 (1 − x 50 ) 100 dx
= ∫ [x − (2 − x)]dx + ∫ [(4 − x) − (x − 2)]dx =∫
π/6
   dx
0 2  dx  2  − ∫ 1∫ x 49 (1 − x 50 ) 100 dx] 10
= [x − 2x]20
+ [6x −
2
x2 ]23 1 
π/3
1
= 18 − 9 − 12 + 4 = 1 = tan4 x sin4 3x  (1 − t) 100  (1 − t) 100  
 2  = t 1/ 50 ∫ dt − ∫  ∫ dt  dx 
π/6
Hence, option (d) is correct.  50  50  0
1  1  1
=  (9 × 0) –  × 1  = − .
2 9  18 [where, x = t]
50

101 Let {x} and [x] denote the fractional I2


part of x and the greatest integer I =0 −
π /2 1 50 × 101
≤ x respectively of a real number x. 103 The value of ∫ dx is
− π / 2 1 + e sin x I
n n ∴ I2 = I 1 − 2
If ∫ (x)dx,∫ [x]dx and 10(n 2 − n), [2020, 5 Sep. Shift-I] 5050
0 0
π π 5051
(n ∈ N, n > 1) are three consecutive (a) (b) ⇒ I2 = I 1
2 4 5050
terms of a GP, then n is equal to 3π
(c) π 5050
……… . [2020, 4 Sep. Shift-II] (d) Q I2 = αI 1 ∴ α =
2 5051
2
107 If θ 1 and θ 2 be respectively the   αx  0  − αx  2 
105 The integral ∫ e x ⋅ x x (2 + log e x)dx ⇒ 4α  
e
 +
e
 =5
1
smallest and the largest values of θ   α  −1  −α  0 
equals [2020, 6 Sep. Shift-II]  
in (0, 2π) − {π} which satisfy the ⇒4[(e 0 − e −α ) − (e −2α − e 0 )] = 5
(a) e (4 e + 1) (b) 4 e2 − 1 5
(c) e (4 e − 1) (d) e (2 e − 1) equation, 2cot 2 θ − + 4 = 0, ⇒ 4[1 − e − α − e −2 α + 1] = 5
sin θ
Ans. (c) ⇒ 4[2 − e − α − e −2 α ] = 5
θ
2 then ∫ 2 cos 2 3θdθ is equal to ⇒ 8e2 α − 4e α − 4 = 5e2 α
The integral,I = ∫ e x (2 + log e x) dx x x θ1 2α α
⇒ 3e − 4e − 4 = 0
[2020, 7 Jan. Shift-II]
1
4 + 16 + 48
Let e ⋅ x = t
x x π 1
(a) + (b)
π
(c)

(d)
π ⇒ eα =
3 6 3 3 9 6
⇒ [e x x x + e x x x (1 + log e x)] dx = dt
[Qe α > 0 ∀ α ∈ R]
⇒ e x x x (2 + log e x) dx = dt Ans. (b) 4+ 8
⇒ eα = =2
and at x = 1, t = e Given trigonometric equation for 6
at x = 2,t = 4e2 θ ∈ (0, 2 π) − { π } is ⇒ α = log e 2
4e 2 5
4e 2
2 cot2 θ − + 4=0 2 dx
∴ I= ∫ dt = [t] e = 4e2 − e = e (4e − 1) sinθ 109 If I = ∫
1
, then
⇒ 2 cos θ − 5 sinθ + 4 sin θ = 0 2x − 9x 2 + 12x + 4
3
e 2 2

⇒ 2 sin2 θ − 5 sinθ + 2 = 0 1 2 1 1 2 1
106 If f (a + b + 1 − x) = f (x), for all x, where ⇒ 2 sin θ − 4 sinθ − sinθ + 2 = 0
2 (a) <I < (b) <I <
6 2 8 4
a and b are fixed positive real ⇒2 sinθ(sinθ − 2) − 1(sinθ − 2) = 0 1 2 1 1 2 1
numbers, then ⇒ (sinθ − 2)(2 sinθ − 1) = 0 (c) < I < (d) <I <
9 8 16 9
1 b
a + b ∫a
x (f (x) + f (x + 1))dx is equal to Q sinθ∈ [−1, 1] − {0 }, for θ ∈ (0, 2 π) − { π } [2020, 8 Jan. Shift-II]
∴sinθ − 2 ≠ 0 Ans. (c) 2
[2020, 7 Jan. Shift-I] 1 π 5π dx
b+ 1 b+ 1 So, sinθ = ⇒ θ = , Given integral I = ∫
(a) ∫a + 1 f (x + 1)dx (b) ∫a + 1 f (x)dx 2 6 6 1 2x − 9x2 + 12x + 4
3

π 5π
b−1 b−1 ∴ θ1 = and θ2 = Let a function f (x) =
1
(c) ∫a − 1 f (x + 1)dx (d) ∫a − 1 f (x)dx 6 6
θ2 2x 3 − 9x2 + 12x + 4
∫ cos 3θdθ
2
Ans. (c) Now, the integral −1(6x − 18x + 12)
2

θ1
Then, f ′ (x) =
Let the integral 2(2x 3 − 9x2 + 12x + 4) 3/ 2
5π / 6
1 b
 1 + cos6θ 
I=
a+b ∫ a
x (f (x) + f (x + 1))dx …(i) = ∫ 
 2
 dθ
 =−
3(x2 − 3x + 2)
π /6 (2x − 9x2 + 12x + 4) 3/ 2
3

On applying property 5π / 6
(x − 1)(x − 2)
sin6θ 
= θ +
b b
1 = −3 3
∫a f (x)dx = ∫ f (a + b − x)dx , we get
a
2 6  π / 6 (2x − 9x2 + 12x + 4) 3/ 2
1 b 1  5π π  1 Qf′ (1) and f′ (2) are zero and f ′ (x) is positive
=  −  + (0 − 0)
I=
a+b ∫a
(a + b − x)(f (a + b − x)
2 6 6  12 ∀ x ∈ (1, 2), so f (x) is an increasing function
+ f (a + b − x + 1))dx ∀ x ∈ (1, 2), so f (1) < I < f (2)
1 4π π
1 b =   = 1 1
a + b ∫a
= (a + b − x)(f (x + 1) + f (x))dx …(ii) 2 6  3 ⇒ <I<
2 − 9 + 12 + 4 16 − 36 + 24 + 4
Qf (a + b + 1 − x) = f (x)  1 1 1 1
  108 The value ofα for which ⇒ <I< ⇒ < I2 <
∴f (a + b − x) = f (x + 1)  2
4α ∫ e − α | x | dx = 5, is
3 2 2 9 8
On adding integrals Eqs. (i) and (ii), we get −1 Hence, option (c) is correct.
a+b b [2020, 7 Jan. Shift-II] x
2I =
a + b ∫a
(f (x) + f (x + 1))dx
(a) log e 2
 3
(b) log e   ∫ t sin(10t)dt is equal to
110 lim 0
 2
⇒ I =  ∫ f (x)dx + ∫ f (x + 1)dx 
1 b b x→ 0 x
2  a   4 1 1 1
a
(c) log e 2 (d) log e   (a) 0 (b) (c) − (d) −
 3
=  ∫ f (x)dx + ∫ (a + b − x + 1)dx 
1 b b 10 10 5
2  a a  Ans. (c) [2020, 8 Jan. Shift-II]

=  ∫ f (x)dx + ∫ f (x)dx 
1 2
b b Ans. (a)
Given,4α ∫ e − α | x |dx = 5
x
2  a a 
∫ t sin(10t)dt
b −1
I = ∫ f (x)dx
0
⇒ 0 2  Given limit lim
a ⇒ 4α  ∫ e αx dx + ∫ e − αx dx  = 5 x→ 0x
Put x = t + 1, then upper limitt = b − 1 and  −1 0  On applying L’Hospital rule, we get
lower limitt = a − 1 and dx = dt, so   x, x ≥ 0  x sin(10 x)
I=∫
b−1
f (t + 1)dt = ∫
b−1
f (x + 1)dx Q| x | =  − x, x < 0  lim
a −1 a −1    x→ 0 1
π /4
(on applying Newton Leibniz rule) Then for the function F , the point ⇒ 2I = ∫ log e (1)dx = 0
= 0 sin(10 × 0) = 0 x = 1is [2020, 9 Jan. Shift-II]
−π / 4

Hence, option (a) is correct. ⇒ I = 0 = log e (1)


8
(a) not a critical point. x
2π x sin x
111 The value of ∫ dx is (b) a point of inflection. 114 Let f (x) = ∫ g (t)dt, where g is a
0 sin x + cos 8 x
8
(c) a point of local maxima.
0
equal to (d) a point of local minima.
non-zero even function. If
(a) 2π (b) 4π (c) 2 π 2
(d) π 2
Ans. (d) x
f (x + 5) = g (x), then ∫ f (t)dt equals
x
[2020, 9 Jan. Shift-I]
It is given,F (x) = ∫ t2 g (t)dt, then
Ans. (d) 1 0

x sin8 x F ′ (x) = x2 g (x) (by Newton-Leibniz rule) [2019, 8 April Shift-II]
Given integral is I = ∫ sin8 x + cos8 x dx t 5 x+5
g (t) = ∫ f (u)du ⇒ g(1) = 0 ∫ g (t)dt ∫ g (t)dt
0 … (i) Q (a) 5 (b)
On applying property, 1 x+5 5
a a
∴ F ′ (x = 1) = 12 ⋅ g (1) = 0 x+5
∫0 f (x) dx = ∫0 f (a − x) dx, we get
5

Now, F ′ ′ (x) = 2x g (x) + x2 g ′ (x) (c) 2 ∫ g (t)dt (d) ∫ g (t)dt


2π (2 π − x) sin8 x x+5
I=∫ dx … (ii) x 5
0 sin8 x + cos8 x = 2x ∫ f (u)du + x2f (x)
1 Ans. (d)
1 x
On adding integrals Eqs. (i) and (ii), we get Now, F ′ ′ (1) = 2∫ f (u)du + f (1) = 2 × 0 + 3 Given, f (x) = ∫ g (t) dt
2π 2 π sin8 x
2I = ∫ dx 1
(Qf(1) = 3 given) 0
0 sin8 x + cos8 x
=3 On replacing x by (−x), we get
−x
2π sin8 x Q F ′ ′ (1) = 3 > 0 and F ′ (1) = 0
⇒ I = π∫
0 sin8 x + cos8 x
dx
∴The point x = 1 is point of local minima
f (− x) = ∫ g (t)dt
0
for the function F.
On applying property, Now, put t = − u, so
2a a Hence, option (d) is correct. x
∫0 f (x) dx = 2∫ f (x)dx,
0 f (− x) = − ∫ g (− u)du
if f (2a − x) = f (x), we get 113 and g (x) = log e x, 0
x
π sin8 x (x > 0) then the value of the integral = − ∫ g (u)du = − f (x)
I = 2 π∫ dx π /4
0 sin8 x + cos8 x

Again, if we apply the same property,


∫ g (f (x))dx is
− π /4 [2019, 8 April Shift-I]
0

[Qg is an even function]


(a) log e 3 (b) log e e
if f (2a − x) = f (x), ⇒ f (− x) = − f (x) ⇒f is an odd function.
(c) log e 2 (d) log e 1
2a
then ∫ f (x) dx = 2 ∫ f (x) dx, we get
a
Now, it is given thatf (x + 5) = g (x)
0 0 Ans. (d) ∴ f (5 − x) = g (− x) = g (x) = f (x + 5)
π /2 sin8 x
I = 4π∫ dx …(iii) The given functions are [Qg is an even function]
0 sin x + cos8 x
8
g (x) = log e x, x > 0 ⇒ f (5 − x) = f (x + 5) …(i)
x
Now, on applying property, 2 − x cos x
a a and f (x) = Let I = ∫ f (t)dt
∫0 f (x) dx = ∫0 f (a − x) dx, we get 2 + x cos x 0
π /4
π /2 cos x 8
Let I=∫ g (f (x))dx Put t = u + 5 ⇒ t − 5 = u ⇒ dt = du
I = 4π∫ dx … (iv) −π / 4 x −5 x −5
0 cos8 x + sin8 x
On adding integrals (iii) and (iv), we get Then, I = ∫
 2 − x cos x 
π /4
log e   dx …(i)
∴ I= ∫ f (u + 5)du = ∫ g (u)du
−5 −5
π / 2 sin8 x + cos8 x
−π / 4  2 + x cos x 
2I = 4 π ∫ dx Put u = − t ⇒du = − dt, we get
5−x
0 cos8 x + sin8 x Now, by using the property 5

π /2  π
b b I=− ∫ g (−t)dt = ∫ g (t)dt
⇒ I = 2 π∫
0
dx = 2 π  
2 ∫ f (x) dx = ∫ f (a + b − x) dx, we get b
5
a
5−x
a a
⇒ I = π2 [Q− ∫ f (x)dx = ∫ f (x)dx and g is an even
π /4  2 + x cos x 
Hence, option (d) is correct. I=∫ log e   dx …(ii) a b
−π / 4  2 − x cos x  5 function]
112 Let a function f : [0, 5] → R be On adding Eqs. (i) and (ii), we get I= ∫ f ′(t)dt
5−x
  2 − x cos x  2 + cos x  
continuous, f (1) = 3 and F be defined 2I = ∫
π /4
log e   + log e   dx [by Leibnitz rule f ′(x) = g (x)]
−π /4  +
as:   2 x cos x  2 x cos x 
= f (5) − f (5 − x) = f (5) − f (5 + x)
x π /4  2 − x cos x 2 + x cos x 
F (x) = ∫ t 2g (t)dt, where =∫ log e  ×  dx [from Eq. (i)]
1 −π / 4  2 + x cos x 2 − x cos x  5 5
t
g (t) = ∫ f (u )du .
1
[Qlog e A + log e B = log e AB] = ∫ f ′ (t)dt = ∫ g (t)dt
5+ x 5+ x
π /2 sin 3 x
115 The value of ∫ dx is On applying the L’ Hopital rule, we get 2π
0 sin x + cos x 2f (x)f ′ (x) 118 The value of ∫ [sin 2x (1 + cos 3x)] dx,
l = lim
[2019, 9 April Shift-I] x →2 1 0
π−1 π −2 π−1 π −2  d φ 2 (x )
where [t] denotes the greatest
(a)
2
(b)
8
(c)
4
(d)
4
Q
 dx φ (x )
∫ f (t)dt = f (φ2 (x)) ⋅ φ2 ′ (x) integer function, is
 1
Ans. (c)
− f (φ1 (x)) ⋅ φ1 ′ (x)] [2019, 10 April Shift-I]
Key Idea Use property of definite So, l = 2f (2) ⋅f ′ (2) = 12f ′ (2) [Qf(2) = 6] (a) − π (b) 2π (c) − 2π (d) π
integral. f (x ) Ans. (a)
2tdt
b b
∫a f (x) dx = ∫a f (a + b − x) dx
∴ lim
x →2 ∫ x −2
= 12f ′ (2), if f(2) = 6
Given integral
6

π I=∫ [sin2x ⋅ (1 + cos3x)]dx
sin3 x 0
Let I = ∫ 2 dx …(i) 117 The value of the integral π
sin x + cos x
0
1 = ∫ [sin2x ⋅ (1 + cos3x)]dx
−1
∫ x cot (1 − x 2 + x 4 )dx is
0
On applying the property,

b b +∫ [sin2x ⋅ (1 + cos3x)]dx
∫a f (x)dx = ∫a f (a + b − x) dx, we get 0
[2019, 9 April Shift-II] π

π 1 π 1 = I 1 + I2 (let) ... (i)


cos3 x
π /2
(a) − log e 2 (b) − log e 2
I=∫ dx …(ii) 4 2 2 2 Now, I2 = ∫

[sin2x ⋅ (1 + cos3x)]dx
0 cos x + sin x π
π π
On adding integrals (i) and (ii), we get (c) −log e 2 (d) −log e 2 let 2π − x = t, upper limitt =0 and lower
π / 2 sin3 x + cos3 x 4 2 limitt = π
2I = ∫ dx Ans. (a)
0 sin x + cos x and dx = −dt
1 0
(sin x + cos x) (sin2 x + cos2 x Let I = ∫ x cot−1 (1 − x2 + x 4 ) dx So, I2 = − ∫ [− sin2x ⋅ (1 + cos3x)]dx
π π
− sin x cos x) π
= ∫2 dx 0
= ∫ [− sin2x ⋅ (1 + cos3x)]dx …(ii)
0 sin x + cos x Now, put x2 = t ⇒ 2xdx = dt 0
π π
 1  Lower limit at x = 0, t = 0 ∴ I = ∫ [sin2x ⋅ (1 + cos3x)]dx
=∫ 2 1 − (2 sin x cos x) dx
0  2  Upper limit at x = 1, t = 1
0
π

=∫
π
 1   1 
π /2
1
1 + ∫0 [− sin2x ⋅ (1 + cos3x)]dx
2  1 − sin2x  dx =  x + cos 2x  ∴ I = ∫ cot−1 (1 − t + t2 ) dt
0  2   4 0 20 [from Eqs. (i) and (ii)]
π
π  1 π 1 = ∫ (−1) dx]
=  − 0  + (−1 − 1) = − 1
  0
2  4 1
tan−1 
1
π 1 π−1
2 2
20∫
=
 1−t + t 
 dt
2 [Q[x] + [− x] = − 1, x ∉Integer]
⇒I = − = Qcot−1 x = tan−1 1  = −π
4 4 4
 x 
−1  t − (t − 1) 
1
1 119 The integral
116 If f : R → R is a differentiable = ∫ tan   dt π /3
 1 + t (t − 1) 
∫ sec x cosec x dx is equal
20 2/ 3 4/ 3
function and π /6
1 
1
lim f ( x) 2t dt =  ∫ ( tan−1 t − tan−1 (t − 1) dt  to [2019, 10 April Shift-II]
x → 2 ∫ (x − 2)
f (2) = 6, then is 2  0  (a) 35 / 6 − 32 / 3
6
 (b) 37 / 6 − 35 / 6
[2019, 9 April Shift-II] −1 x − y −1 −1 
Qtan 1 + xy = tan x − tan y  (c) 35 / 3 − 31/ 3
(a) 12 f′ (2) (b) 0  
(c) 24 f′ (2) (d) 2 f′ (2) 1 1
(d) 34 / 3 − 31/ 3
−1 −1
Ans. (a) Q ∫ tan (t − 1)dt = ∫ tan (1 − t − 1) dt Ans. (b)
π /3
0 0
Key Idea Let I = ∫ sec
2/ 3
1 x cosec 4 / 3x dx
−1
(i) First use L’ Hopital rule = − ∫ tan (t) dt π/6
(ii) Now, use formula 0 π /3
a a 1
d
φ 2 (x )
because ∫ f (x)dx = ∫ f (a − x) dx = ∫ cos 2/ 3
x sin4 / 3 x
dx

dx ∫ f (t)dt = f [φ2 (x)] ⋅ φ′2 (x) 0 0


π/6
φ 1 (x ) π /3
1 sec2 x
− f [φ1 (x)] ⋅ φ′1 (x) 1
So, I = ∫ ( tan−1 t + tan−1 t) dt = ∫ (tan x) 4 / 3
dx
f (x ) 20 π/6

∫ 2tdt
1 1
t [multiplying and dividing the
f (x )
2tdt = ∫ tan−1 tdt = [t tan−1 t] 10 − ∫ dt
Let l = lim
x →2 ∫ = lim 6
(x − 2) x →2 (x − 2) 0 0
1 + t2 denominator bycos 4/ 3 x]
6
[by integration by parts method] Put, tanx = t, upper limit, at x = π /3
 0 form, as f(2) = 6
 0  π 1 π 1 ⇒t = 3
= − [log e (1 + t2 )] 10 = − log e 2
4 2 4 2 and lower limit, at x = π /6 ⇒ t = 1 / 3
π /2cosx
and sec2 x dx = dt =∫ dx ⇒ 8 α2 + 8 α + 2 = 9 α2 + 9 α
3 0 cox + 1
3
dt t − 1/ 3 ⇒ α2 + α − 2 = 0
So, I = ∫ t 4/ 3
=
 − 1 / 3

 1/ π /2
x
2 cos2 − 1 ⇒ (α + 2) (α − 1) = 0
1/ 3 3
=∫ 2 dx
⇒ α = 1, − 2
= − 3  1/ 6 − 31/ 6 
1 0 x
2 cos2
3  2 From the options we get α = − 2
θ
= 3 ⋅3 1 / 6 − 3 ⋅3 − 1 / 6 = 3 7 / 6 − 3 5 / 6 [Qcosθ = 2 cos2 − 1 and π
2 122 The value of ∫ |cos x | 3 dx is
θ 0
π /2 cot x cosθ + 1 = 2 cos2 ]
120 If ∫ dx = m(π + n), 2 [2019, 9 Jan. Shift-I]
0 cot x + cosec x π /2  1 2 x 2 4
=∫  1 − sec  dx (b) −
then m ⋅ n is equal to 0  2 2
(a)
3 3
[2019, 12 April Shift-I] π /2 4
 x π 1
1 = x − tan = − 1 = ( π − 2) (c) 0 (d)
(a) − (b) 1  2  0 2 2 3
2
1 Since, I = m( π − n) Ans. (d)
(c) (d) −1 We know, graph of y = cos x is
2 1
∴ m ( π − n) = ( π − 2)
Ans. (d) 2 Y
π /2 cot x On comparing both sides, we get
Let I = ∫ dx 1
0 cot x + cosec x m = and n = − 2
cos x 2 X′ X
O π/2 π
π /2 1
=∫ sin x dx Now, mn = × (− 2) = − 1
0 cos x 1 2
+
sin x sin x Y′
π /2 cos x
=∫ dx 121 A value of α such that
0 1 + cos x α+1 ∴ The graph of y = | cos x | is
dx  9
=∫
π /2 cos x (1 − cos x)
dx ∫ (x + α) (x + α + 1) = log e  8 is Y
y=|cos x|
0 1 − cos2 x α
π /2 cos x − cos2 x [2019, 12 April Shift-II]
=∫ dx 1
0 sin2 x (a) − 2 (b)
π /2 2
=∫ (cosec x cot x − cot x) dx 2
0 1
π /2 (c) − (d) 2 X′
O π/2 π
X
=∫ (cosec x cot x − cosec x + 1) dx 2
2
0 Y′
= [− cosec x + cot x + x] π0 /2 Ans. (a)
π
π /2 π
 cos x − 1  α+ 1
dx ∴ I = ∫ | cos x |3 = 2 ∫ 2 | cos x |3dx
= x+ Let I = ∫
0 0
 sin x  0 π
α
(x + α) (x + α + 1) (Qy = | cos x | is symmetric about x = )
π /2 2
  2 x  α+ 1
  − 2 sin   (x + α + 1) − (x + α) π
  π 
= x +
 2

= ∫ (x + α) (x + α + 1)
dx = 2∫ 2 cos3x dx Qcos x ≥ 0 for x ∈ 0, 
 2  
 x
2 sin cos 
x α
0

 2 2  0 α+ 1 Now, as cos3x = 4 cos3 x − 3 cos x
 1 1 

= x − tan
x 
π /2 = ∫  −  dx
 x + α x + α + 1
1
∴ cos3 x = (cos3x + 3 cos x)
 2  0
α 4
π
π = [log e (x + α) − log e (x + α + 1)] αα + 1 2 2
4 ∫0
= −1 α+ 1
∴ I= ( cos3x + 3 cos x) dx
2   x + α  π
1 = log e  
= [ π − 2]   x + α + 1 α 1  sin3x 2
=  + 3 sin x 
2 2 3 0
= m [ π + n] 2α + 1 2α
[given] = log e − log e
1 2α + 2 2α + 1 1  1 3π π 1 
On comparing, we get m = and n = − 2 =  sin + 3 sin − sin0 + 3 sin0  
2  2α + 1 2α + 1 2   3 2 2   3  
∴ m⋅ n = − 1 = log e  × 
 2α + 2 2α 
=  (−1) + 3 − [0 + 0] 
Alternate Solution 1 1
π /2 cot x  9 2  3  
Let I = ∫ dx = log e   (given)
0 cot x + cosec x  8  3π  π π 
cos x (2 α + 1)2 9 Qsin 2 = sin π + 2  = − sin 2 = − 1
⇒ =  
π /2
=∫ sin x 4α (α + 1) 8
=  − + 3 =
dx 1 1 4
0 cos x 1
+ 2  3  3
sin x sin x ⇒8 [4α2 + 4α + 1] = 36 (α2 + α)
π /3 tan θ 1  π 1 1 1π 
123 If ∫ dθ =1 − , (k > 0),
Y
y=f(x) =  −1 +  + (0 + 1) + (1 − 0) +  − 1
0 2k sec θ 2  2 2 4 5 2 
 1 1 1  π π 
then the value of k is =  −1 + + −  +  + 
[2019, 9 Jan. Shift-II] X  2 4 5   2 10 
– √2 √2 ∴
1 O f(x) < 0 for – √2 < x < √2 −20 + 10 + 5 − 4 5 π + π
(a) 1 (b) (c) 2 (d) 4 + – – + = +
2 20 10
– √2 0 √2
Ans. (c) 9 3π 3
=− + = (4 π − 3)
tan θ
π/3 1 Note that the definite integral 20 5 20
We have, ∫ dθ = 1 − , (k >0)
2k sec θ
b 4
2
∫ (x − 2x )dx represent the area
0 2
x 1
π/3 tan θ
a 126 If ∫ f (t) dt = x 2 + ∫ t 2f (t)dt, then
Let I = ∫ dθ bounded byy = f (x) , x = a, b and the X 0 x
0 2k sec θ -axis. But between x = − 2 and  1
f ′   is
1 π / 3 tanθ x = 2, f (x) lies below the X-axis and so  2
2k ∫0
= dθ [2019, 10 Jan. Shift-II]
secθ value definite integral will be negative. 24 18
1 π/3 (sin θ) Also, as long as f (x) lie below the X-axis, (a) (b)
2k ∫0
= dθ the value of definite integral will be 25 25
1 6 4
(cos θ) minimum. (c) (d)
cosθ 25 5
∴ (a, b) = (− 2, 2) for minimum of I.
1 π / 3 sin θ
2k ∫0
= dθ Ans. (a)
cos θ π /2 dx
125 The value of ∫ , x 1 2
∫0 f (t) dt = x + ∫x t f (t)dt
2
− π / 2 [x] + [sin x] + 4 Given,
Let cosθ = t ⇒ − sinθ dθ = dt
⇒ sinθ dθ = − dt where [t] denotes the greatest On differentiating both sides, w.r.t. ‘x’,
for lower limit,θ = 0 ⇒t = cos0 = 1 we get
integer less than or equal to t, is
π π 1 f (x) = 2x + 0 − x2f (x)
for upper limit, θ = ⇒t = cos = [2019, 10 Jan. Shift-II]
 
d  
3 3 2 ψ (x )
1 1 d d
1 1/ 2 − dt −1 1/ 2 −
1 (a) (7π − 5)
12
(b)
12
(7π + 5) ∫
Q  f (t)dt  = f (ψ(x)) ψ(x) − f (φ(x)) φ(x) 
 dx  φ (x )  
2k ∫1 2k ∫
⇒ I= = t 2 dt dx dx
  
t 1 3 3
1
(c) (4 π − 3) (d) (4 π − 3) ⇒ (1 + x2 ) f (x) = 2x
10 20
 1
− +1 2 ⇒ f (x) =
2x
1 t 2  1
1
Ans. (d)
=−   =− [2 t ] 21 π
1 + x2
2k − 1 + 1  2k
Let I = ∫ 2
dx On differentiating w.r.t. ‘x’ we get

 2 1 −π [x] + [sin x] + 4 (1 + x2 )(2) − (2x) (0 + 2x)
2  1  2  1 
2 f ′ (x) =
=− − 1 = 1 −  −1 dx 0 dx (1 + x2 )2
2k  2 2  2 = ∫− π + ∫−1 [x] + [sin x] + 4
 k [x] + [sin x] + 4 2 + 2x − 4x2 2 − 2x2
2

1
2 = =
π (1 + x2 )2 (1 + x2 )2
Q I = 1− (given) 1 dx dx
+∫
[x] + [sin x] + 4 ∫ 1 [x] + [sin x] + 4
2 + 2 2
 1
2 − 2  
0 1
2  1  1 2 2−2 
∴ 1 −  = 1− ⇒ =1 −2, − π /2 < x < −1  1  2  4
2k  2  2 2k ∴ f′   = =
 − 1, − 1 ≤ x < 0  2  2
2
 1+ 1
2
  1   
⇒ 2 = 2k ⇒ 2k = 4 ⇒k = 2 Q [x] =   1+   
0≤ x< 1   2   4
b  0,  
 1, 1 ≤ x < π /2
124 Let I = ∫ (x 4 − 2x 2 ) dx. If I is 1 3
2−
a  − 1, − π /2 < x < −1 2 24
−1, − 1 < x < 0 = 2
= 2 =
minimum, then the ordered pair   5 25 25
and [sin x] =   
(a, b) is [2019, 10 Jan. Shift-I]  0, 0 < x < 1  4 16
(a) (− 2 , 0)  0, 1 < x < π /2
(b) (0, 2 ) [QFor x < 0 , − 1 ≤ sin x < 0 and for x > 0, 127 The value of the integral
(c) ( 2 , − 2 ) 0 < sinx ≤ 1] 2 sin 2 x
(d) (− 2 , 2 ) So, ∫−2  x  1 dx
−1 dx 0 dx 1 dx +
Ans. (d) I = ∫− π +∫
−1 −1 − 1 + 4 ∫0 0 + 0 + 4
+  π  2
b
We have, I = ∫ (x 4 − 2x2 )dx 2
− 2 − 1 + 4
a π (where, [x] denotes the greatest
dx
Let f (x) = x 4 − 2x2 = x2 (x2 − 2) +∫2 integer less than or equal to x) is
1 1+ 0 + 4
= x2 (x − 2) (x + 2) π [2019, 11 Jan. Shift-I]
−1 dx 0 dx 1 dx dx
=∫ (a) 4 − sin4
Graph of y = f (x) = x − 2x is 4 2
−π
1
+ ∫−1 2 ∫0 4
+ +∫
1
2
5
(b) 4
2 (c) sin 4 (d) 0
π π  1 
⇒ x   + (log e x − log e e) dx = dt
Ans. (d) x 1
sin x
2 2 6 4   x   t
Let I=∫ dx 5
1
t  1 
+ 
−2 1 x 1
2  π 
 
 3
⇒ ( 1 + log e x − 1) dx = dt
t
sin2 x 1
Also, let f (x) = 1 1 1 dt ⇒ (log e x) dx = dt
1 x ∴I= ⋅ ∫ t
+ 5
2 5 (1/ 3 ) t2 + 1
2  π  Also, upper limit x = e
1 1
sin2 (− x) = (tan−1 (t)) 1 ⇒ t = 1 and lower limit x = 1 ⇒t =
Then, f (− x) = 10 (1 / 3 ) 5 e
1  x
I = ∫  t2 −  ⋅ dt
+ − 1 1 1
1   1  ∴
2  π  =  tan−1 (1) − tan−1   1/ e  t t
10   9 3  
(replacing x by − x) ⇒ I=∫
1
(t − t −2 ) dt
sin2 x 1 π  1  1/ e
= =  − tan−1  
 9 3  
1
1  x  10  4   t2 1  
+  − 1 −   I =   +  
2   π  
 2 t   1
  − [x], if x ∈I  129 Let f and g be continuous e
Q[− x] =    1   1 
 − 1 − [x], if x ∉I  functions on [0, a] such that =  + 1 −  2 + e  
f (x) = f (a − x) and g (x) + g (a − x) = 4,   2   2e 
sin2 x
⇒ f (− x) = − = − f (x) a
1 x
+
then ∫ f (x) g (x) dx is equal to 3
= −e− 2
1
0
2  π  [2019, 12 Jan. Shift-I]
2 2e
i.e. f (x) is odd function a a 131 Let f : R → R be a continuously
(a) 4 ∫ f (x) dx (b) ∫ f (x) dx
∴ I =0 0 0
differentiable function such that
a a
 a  0, if f (x) is odd function  (c) 2 ∫ f (x) dx (d) − 3∫ f (x) dx 1
Q ∫− a f (x) dx =  a  0 0 f (2) = 6 and f ′ (2) = . If
 ∫0 48
 2 f (x) dx, if f (x) is even function
 Ans. (c) f ( x)

128 The integral Let


0
a
I = ∫ f (x) g (x) dx … (i) ∫6 4t 3 dt = (x − 2)g (x), then lim g (x)
x→ 2
π /4 dx is equal to
∫π / 6 sin2x (tan 5 x + cot 5 x) equals
a
= ∫ f (a − x) g (a − x) dx [2019, 12 April Shift-I]
0
(a) 18
Q a f (x) dx = a 
 ∫0 ∫0 f (a − x) dx  (b) 24
[2019, 11 Jan. Shift-II]
(c) 12
a
1 π  1  ⇒ I = ∫ f (x) [4 − g (x)] dx (d) 36
(a)  − tan− 1   0
54  3 3  Ans. (a)
[Qf (x) = f (a − x) and g (x) + g (a − x) = 4] f (x )
1  1  Given ∫ 4t 3dt = (x − 2) g (x)
tan− 1 
a a
(b)  = ∫ 4f (x) dx − ∫ f (x) g (x) dx 6
20  9 3 0 0
f (x )

1 π ⇒
a
I = 4 ∫ f (x) dx − I [from Eq. (i)] ∫ 4t 3 dt
− 1 1  ⇒ g (x) = 6 [provided x ≠ 2]
 − tan 
0
(c)  (x − 2)
10  4  9 3  a a
⇒ 2I = 4 ∫ f (x) dx ⇒ I = 2 ∫ f (x) dx f (x )
∫6 4t 3dt
0 0
π
(d) So, lim g (x) = lim
e x e x
2x
130 The integral ∫    −   
40 x→2 x→2 x −2
Ans. (c) 1  e  x Q0 form as x → 2 ⇒ f (2) = 6
 
π /4 dx log e x dx is equal to  0 
Let I = ∫ [2019, 12 Jan. Shift-II]
π /6 4(f (x)) 3f ′ (x)
sin2x (tan5 x + cot5 x) lim g (x) = lim
3 1 1 1 1 x→2 x→2 1
π /4 (1 + tan2 x) tan5 x (a) −e− 2 (b) −+ −
=∫ dx 2 2e 2 e 2 e2  d φ 2 (x ) 
π / 6 2 tan x (tan10 x + 1) 1 1 3 1 1 Q
 dx ∫φ f (t) dt = f (φ 2 (x)), φ 2′ (x) − f (φ 1 (x)) ⋅ φ ′1 (x)

(c) − e − 2 (d) − − 2 1 (x )
 2 tan x  2 e 2 e 2e
Qsin2x =  On applying limit, we get
 1 + tan2 x  Ans. (a) 1
1 π / 4 tan4 x sec2 x lim g (x) = 4(f (2)) 3f ′ (2) = 4 × (6) 3 ,
= ∫ dx e  x  2 x  e  x  x→2 48
Let I = ∫   −    log e x dx
2 π / 6 (tan10 x + 1)    x   1
 e
1
 Qf (2) = 6 and f ′ (2) =
 48 
Put tan x = t
5
[Qsec x = 1 + tan x]
2 2 x
Now, put   = t ⇒ x log e   = log t
x x 4 × 216
⇒ 5 tan x sec x dx = dt
4 2 e e = = 18
48
⇒ x (log e x − log e e) = log t
2 3π / 4 dx
132 ⇒ log l = ∫ log (1 + x) dx 135 ∫ is equal to
 n
0 π /4 1 + cos x
n n 1 2
lim  2 2 + 2 2 + 2 2 + ... +  
⇒ log l = log (1 + x) ⋅ x − ∫
1 
⋅ x dx  [JEE Main 2017 (Offline)]
n → ∞  n +1 n +2 n +3 5 n 1+ x
 0 (a) − 2 (b) 2 (c) 4 (d) − 1
is equal to [2019, 12 Jan. Shift-II] 2 x + 1 − 1 Ans. (b)
⇒ log l = [log (1 + x) ⋅ x]20 − ∫ dx
(a) tan− 1 (3) (b) tan− 1 (2) 0 1+ x 3π / 4
dx 3π / 4 1 − cos x
Let I = ∫
1 + cos x ∫π / 4 1 − cos2 x
= dx
(c) π / 4 (d) π /2 2  1  π/4
⇒ log l = 2 ⋅ log 3 − ∫  1 −  dx 3π / 4 1 − cos x
Ans. (b) 0  1+ x =∫ dx
Clearly, π/4 sin2 x
⇒ log l = 2 ⋅ log 3 − [x − log 1 + x ] 20
 n n n 1  3π / 4
lim  + 2 2 + 2 2 + ...+  ⇒ log l = 2 ⋅ log 3 − [2 − log 3] =∫ (cosec 2 x − cosec x cot x)dx
n→ ∞  n2 + 12 n +2 n +3 5n  π/4
⇒ log l = 3 ⋅ log 3 − 2 = [− cot x + cosec x] 3ππ/ /44
 n n n
= lim  2 2 + 2 2 + 2 2 ⇒ log l = log 27 − 2 ⇒ l = e log 27 − 2 = [(1 + 2) − (− 1 + 2)] = 2
n→ ∞  n + 1 n +2 n +3 27
= 27 ⋅ e − 2 = 2
n 
+ ....+ 2  e 136 The integral
n + (2n)2  sin 2 x π /2 4 log x 2
2n
= lim ∑ 2 2
n 134 The value of ∫
− π /2 1 + 2x
dx is ∫2 log x 2 + log(36 − 12x + x 2 )dx is
n→ ∞ r = 1 n + r
2n [JEE Main 2018] equal to [JEE Main 2015]
1 1 2 dx
= lim ∑ ⋅ =∫ π π
n→ ∞ r = 1 2 0 1 + x2 (a) (b) (a) 2 (b) 4 (c) 1 (d) 6
1 +  
r n
8 2 Ans. (c)
 n π
(c) 4π (d) Central Idea Apply the property
 pn
1 r p  4
Q lim ∑ f   = ∫ 0f (x) dx 
b b

 n→ ∞
r =1 n n 
Ans. (d) ∫a f (x)dx = ∫a f (a + b − x) dx and then
= [tan−1 x]20 = tan−1 2 Key Idea ad(d)
b b 4 log x2
 (n + 1)(n + 2) K 3n
1/ n
Use property = ∫ f (x) dx = ∫ f (a + b − x) dx Let I=∫ dx
133 lim   is equal 2 log x + log(36 − 12x + x2 )
2
a a
n→ ∞  n 2n  4 2log x
π /2
sin2 x =∫ dx
to [JEE Main 2016] Let I= ∫ 1 + 2x dx 2 2log x + log(6 − x)2
18 27 − π /2 4 2log xdx
(a) (b)  π π  =∫
e4 e2 π /2 sin2  − + − x  2 2 [log x + log(6 − x )]
 2 2 
9
(c) 2 (d) 3log 3 − 2 ⇒ I= ∫ π π

dx
+ −x ⇒ I=∫
4 log xdx
…(i)
e − π /2
1+ 2 2 2 2 [log x + log(6 − x)]
Ans. (b)  b  4 log(6 − x)
I=∫
b
⇒ dx …(ii)
1
Q∫ f (x)dx = ∫ f (a + b − x)dx  2 log(6 − x) + log x
 (n + 1) ⋅ (n + 2) K (3n)  n  a 
Let l = lim   a
n→ ∞  n2 n  Q bf (x)dx = bf (a + b − x)dx 
 ∫a ∫a 
π /2
1 sin2 x
(n + 1) ⋅ (n + 2) ... (n + ⇒ I= ∫ dx
= lim 
2n)  n 1 + 2− x adding Eqs. (i) and (ii), we get
 − π /2
n→ ∞  n2 n  4 log x + log(6 − x)
π /2 2I = ∫ dx
2x sin2 x 2 log x + log(6 − x)
1 ⇒ I= ∫ 2x + 1
dx
4
 n + 1  n + 2  n + 2n   n
= lim   ⇒ 2I = ∫ dx = [x]24
− π /2
  K 
n→ ∞   n   n   n  2
π /2
 2x + 1  ⇒ 2I = 2 ⇒ I = 1
Taking log on both sides, we get
⇒ 2I = ∫ sin2 x  x
 2 + 1
 dx
− π /2
1
log l = lim π /2 π /2
137 The integral
n→ ∞ n
π x x
⇒ 2I = ∫ sin x dx ⇒ 2I = 2 ∫ sin x dx
2 2

  1  2  2n   − π /2
∫0 1 + 4 sin 2
2
− 4 sin dx is equal
2
log  1 + n   1 + n  ...  1 + n  
0
   [Q sin2 x is an even function] to [JEE Main 2014]
1 π /2 π /2

⇒ log l = lim (a) π −4 −4 −4 3
⇒ I= ∫ sin xdx ⇒ I= ∫ cos xdx
2 2 (b)
n→ ∞ n 3 π
0 0
(c) 4 3 − 4 (d) 4 3 − 4 −
  2n   a a 
log 1 +  + log  1 +  + ... + log  1 + 
1 2 3
  Q∫ f (x)dx = ∫ f (a − x)dx 
n  n  n    0  Ans. (d)
0
π /2
1 2n 2
⇒ log l = lim ∑ log  1 +  π  x
r π /2 π π x
n→ ∞ n
r =1
 n 
⇒ 2I = ∫ dx ⇒ 2I = [x] 0 ⇒ I = 4 ∫0  1 − 2 sin  dx = ∫0 | 1 − 2 sin |dx
 2 2
0
π
 x π  x x f ′′( π) = − π < 0
= ∫ 3  1 − 2 sin  dx = ∫π  1 − 2 sin  dx 139 If g (x) = ∫ cos 4t dt, then g (x + π) is
0  2  2  So, f (x) has local maximum at x = π.
3 0

equal to f ′′(2 π) = π > 0


  x − a, x ≥ a  [AIEEE 2012]
Q| x − a | = − (x − a), x < a  g (x ) So, f (x) has local minimum at x = 2π.
   (a) (b) g (x ) + g (π )
g (π ) 1 8 log (1 + x)
π
x
(c) g (x ) − g (π ) (d) g (x ) ⋅ g (π )
141 The value of ∫ dx is
 x 3 x 0
1+ x2
= x + 4 cos − x + 4 cos
 2  0  2  π
[AIEEE 2011]
Ans. (b, c)
3 x
π Given Integral g (x) = ∫ cos 4t dt π π
= 4 3 −4− 0 (a) log 2 (b) log 2
3 8 2
To find g (x + π) in terms of g (x) and g( π).
x (c) log2 (d) π log2
138 Statement I The value of the g (x) = ∫ cos 4t dt Ans. (d)
0
π /3 dx
integral ∫ is equal to t =x + π 1 8 log (1 + x)
π / 6 1 + tan x ⇒ g (x + π) = ∫ cos 4t dt Let I = ∫ dx
t =0 0 (1 + x2 )
x x+π
π/6 . = ∫ cos 4t dt + ∫x cos 4t dt Put x = tanθ ⇒ dx = sec2 θd θ
0
Statement II When x = 0 ⇒ tanθ = 0
= g (x) + I 1 [say]
b b ∴ θ=0
∫ f (x) dx = ∫ f (a + b − x) dx
a a where, I 1 = ∫
x+π π
cos 4t dt = ∫ cos 4t dt When x = 1 = tanθ
[JEE Main 2013] x 0 π
⇒ θ=
(a) Statement I is true Statement II is [using definite integral property] 4
π / 4 8log (1 + tanθ)
true; Statement II is a correct = g( π) ∴ I=∫ ⋅ sec2 θd θ
explanation of Statement I π
0 1 + tan2 θ
I 1 = 
sin 4t  π/4
Also,  =8∫ log (1 + tan θ) d θ
(b) Statement I is true, Statement II is  4 0 0
true; Statement II is not a correct π/4
sin 4 π sin 0  I =8∫
explanation of Statement I =  −  =0 0
log (1 + tan θ) d θ …(i)
 4 4  a a
(c) Statement I is true, Statement II is Using ∫ f (x) dx = ∫ f (a − x) dx
i.e., the value of I 1 is zero. 0 0
false
Since, g ( π) = I 1 = 0, so value of g (x + π) ∴
π
log 1 + tan  − θ  d θ
(d) Statement I is false, Statement II is π/4
does not depend on g( π) and we can add I =8∫
true or subtract g ( π) to or from g (x).
0  4 
Ans. (d) Hence, g (x + π) = g (x) + g ( π) π/4  1 − tan θ 
=8∫ log 1 +  dθ
π/3 dx or g (x) − g ( π) 0
 1 + tan θ 
Let I = ∫ …(i)
π/6 1+ tan x  5π  π/4  2 
140 For x ∈ 0,  , define =8∫ log   dθ …(ii)
π/3 dx  2
0
 1 + tan θ 
∴ I=∫
π/6
π  f (x) = ∫
x On adding Eqs. (i) and (ii), we get
1+ tan  − x  t sint dt. Then, f has
2  0
π/4   2  
π/3 [AIEEE 2011] 2I = 8 ∫ log { 1 + tan θ} + log   d θ
dx
=∫ 0
  1 + tan θ  
π / 6 1 + cot x (a) local minimum at π and 2π.
π/4
(b) local minimum at π and local ⇒ I =4∫ log 2 d θ
π/3 tan x dx 0
⇒ I=∫ …(ii) maximum at 2π.
π/6 1+ tan x (c) local maximum at π and local = 4⋅log 2(θ) 0π / 4
minimum at 2π. π
On adding Eqs. (i) and (ii), we get = 4 log 2⋅  − 0 
(d) local maximum at π and 2π. 4 
π/3
2I = ∫ dx Ans. (c)
π /6 = π log2
x
⇒ 2I = [x] ππ // 36 Here, f (x) = ∫ t sin t dt, where
0
142 Let [ ] denotes the greatest integer
1  π π π  5 π
∴ I= − = x ∈  0,  function, then the value of
2  3 6  12  2  1. 5

Hence, Statement I is false. f ′ (x) = { x sin x − 0 } …(i) ∫0 x [x 2] dx is


[AIEEE 2011]
b b [using Newton-Leibnitz formula]
But ∫a f (x) dx = ∫ f (a + b − x) dx is a true
a ∴ f ′ (x) = x sin x = 0 ⇒ sin x = 0 (a)
5
(b) 0
4
statement by property of definite ∴ x = π,2 π 3 3
(c) (d)
integrals. 1 2 4
f ′′(x) = x cos x + sin x
2 x
π
Ans. (d) ⇒ I=∫ [cot ( π − x)] dx e log t e t log t  − dt 
0 =∫ dt − ∫ × 2 
Here, ∫
1. 5
x [x ] dx 2 1 1+ t 1 (1 + t)  t 
π
0 =∫ [− cot x] dx …(ii) e log t e log t
1 2 1. 5
0 =∫ dt + ∫1 dt
I = ∫ x ⋅0 dx + 1+ t t (1 + t)
0 ∫1 x ⋅ 1 dx + ∫ 2
x ⋅2 dx On adding Eqs. (i) and (ii), we get
1

π π e log t e log t e log t


2I = ∫ =∫ dt + ∫1 dt − ∫
[cot x] dx + ∫0 [− cot x] dx dt
2
 x2  1 1+ t t 1 (1 + t)
= 0 +   + [x2 ] 1. 25 0

 2 1 =∫
π
(− 1) dx
e log t  1 1 1 
=∫ dt Qt (1 + t) = t − t + 1 
1 0 1
= {2 − 1} + {(1.5)2 − 2} t  
2  − 1 if x ∉Z  e
Q[x] + [− x] =    (log t)2  1 1
1 1 1 3 =  = [(log e) − (log 1) ] =
2 2
= + 225
. − 2= + =   0,if x ∈ Z  2 2 2
2 2 4 4  1
= [− x] π0 = − π
π 147 The solution for x of the equation
143 Let p (x) be a function defined on R ∴ I=− x dt π
f (3x)
= 1,
2
∫ 2 2 = 2 is [AIEEE 2007]
such that lim
1 sin x t t −1
x → ∞ f (x)
145 If I = ∫ dx and 3
p ′ (x) = p ′ (1 − x), for all x ∈[0, 1],
0 x (a) − 2 (b) π (c) (d) 2 2
1 cos x 2
J=∫
1
p(0) = 1 and p(1) = 41. Then, ∫ p (x) dx 0
dx. Then, which one of Ans. (a)
0 x πx dt
is equal to [AIEEE 2010] the following is true? [AIEEE 2008] Given,
t t −1
∫2 2 2
=
(a) 41 (b) 21 2 2
(a) I > and J < 2 (b) I > and J > 2 π
(c) 41 (d) 42 3 3 ⇒ [sec−1 t] x =
2 2
Ans. (b) 2 2 π
(c) I < and J < 2 (d) I < and J > 2 ⇒ sec−1 x − sec−1 2 =
3 3 2
We have, p′(x) = p′ (1 − x), ∀ x ∈ [0 , 1],
Ans. (c) π π 3π
p (0) = 1, p (1) = 41 [given] ⇒ sec−1 x = + =
1 sin x 1 x 2 4 4
On integrating, we get Since, I = ∫ dx < ∫ dx, 3π
0 x 0 x ∴ x = sec =− 2
∫ p′ (x) dx = ∫ p′ (1 − x) dx because in x ∈ (0, 1), x > sin x
4
⇒ p (x) = − p (1 − x) + C 6 x
⇒ I<∫
1 2
x dx = [x 3/ 2 ] 10 ⇒ I <
2 148 The value of ∫ dx is
Put x = 1; p(1) = − p(0) + C 3
9− x + x
0 3 3
⇒ 41 = − 1 + C 1 [AIEEE 2006]
1 cos x 1 −
⇒ C = 42 and J = ∫ dx < ∫ x 2 dx (a) 3/2 (b) 2
0 x 0
∴ p (x) + p (1 − x) = 42 (c) 1 (d) 1/2
1 = 2 [x 1/ 2 ] 10 = 2 Ans. (a)
Now, I = ∫ p (x) dx
0 ∴ J<2 6 x
1 Let I = ∫ dx …(i)
⇒ I = ∫ p (1 − x) dx 3 9− x + x
146 If F (x) = f (x) + f   ,
0 1
Q a f (x) dx = a f (a − x) dx   x 6 9− x
 ∫ 0 ∫0 x log t
=∫ dx
 9−9+ x + 9− x
where f (x) = ∫
3
dt. Then, F (e) is
1 1 1+t
⇒ 2I = ∫ [(p (x) + p (1 − x)] dx [on adding] 6 9− x
0 equal to [AIEEE 2007] ⇒ I=∫ dx …(ii)
1
3 x + 9− x
= ∫ 42dx = 42 1
(a) (b) 0 (c) 1 (d) 2
0
2 On adding Eqs. (i) and (ii), we get
∴ I = 21 Ans. (a) 6 x + 9− x
2I = ∫ dx = [x] 63 = 6 − 3
π log t x
x + 9− x
Since, f (x) = ∫
3
144 ∫ [cot x] dx, where [] denotes the 1 1+ t
dt and
3
0 ∴ I=
2
F (e) = f (e) + f  
greatest integer function, is equal 1
e − π /2
to [AIEEE 2009] 149 ∫ [(x + π) 3 + cos 2 (x + 3π)] dx is
− 3π / 2
π π F ( e) = ∫
e log t 1/ e log t
(a)
2
(b) 1 (c) − 1 (d) −
2

1 1+ t
dt + ∫1 1+ t
dt
equal to [AIEEE 2006]

1  π4  π π
Ans. (d) Put t = in second integration (a)   +   (b)
π t  32   2  2
Let I = ∫ [cot x] dx …(i) e log t e − log t  1 
∴ F ( e) = ∫ dt + ∫  π π4
1  t 
0 d
1 1+ t 1
1+ (c)   − 1 (d)
 4 32
t
Ans. (b) a Ans. (a)
− π /2
151 The value of ∫ [x]f ′ (x) dx, a > 1,
1
Let I = ∫
2
[(x + π) + cos (x + 3 π)] dx
3 2
 (log x − 1) 
− 3π /2
− π /2
where [x] denotes the greatest ∫  1 + (log x)2  dx
⇒ I=∫ [(x + π) 3 + cos2 x] dx …(i) integer not exceeding x, is
− 3π /2
[AIEEE 2006] (log x)2 + 1 − 2 log x
 π 3 π 3 =∫ dx
− π /2  (a) [a] f (a ) − { f (1) + f (2) + ... + f ([a])} [(log x)2 + 1]2
⇒ I=∫  − − − x − π
− 3π /2
  2 2  (b) [a] f ([a]) − { f (1) + f (2) + ... + f (a )}
(log x)2 + 1 − 2 x  log x ⋅ 
1
2  π 3π  (c) af ([a]) − { f (1) + f (2) + ... + f (a )} 
+ cos  − − − x   dx =∫ x dx
 2 2  (d) af (a ) − { f (1) + f (2) + ... + f ([a])} [(log x)2 + 1]2
Q b f (x) dx = b f (a + b − x) dx  Ans. (a)
 ∫ a ∫a  =∫
d x 
Since,   dx
a 2 3 dx
 (log x) 2
+ 1 
∫ [x]f ′ (x) dx = ∫ f ′ (x) dx + ∫2 2f ′ (x) dx
− π /2
⇒ I=∫ [− (x + π) 3 + cos2 x] dx …(ii) 1 1 x
− 3π /2 = +C
a (log x)2 + 1
On adding Eqs. (i) and (ii), we get
− π /2
+ ... + ∫ [a ] [a]f ′ (x) dx
Alternate Solution
2I = ∫ 2 cos2 x dx = [f (x)]21 + 2 [f (x)]23 + ... + [a][f (x)] a[a ] 2
− 3π / 2
 (log x − 1) 
− π /2 = f (2) − f (1) + 2f (3) − 2f (2) + ... Let I = ∫  2
dx
− π /2  sin 2x   1 + (log x) 
=∫ (1 + cos 2x) dx = x + + [a] f (a) − [a] f ([a])
− 3π / 2  2  −3π /2
= [a] f (a) − {f (1) + f (2) + ... + f ([a])} and t = log x
 π sin (− π)  3 π sin (−3 π)   ⇒ x = e t ⇒ dx = e t dt
= − + − − + 
 2 2  2 2  152 2
t−1  t
π 3π ∴ I =∫ 2
e dt
=− + =π 1 1 2 4 n 
lim  2 sec 2 2 + 2 sec 2 2 +...+ 2 sec 21 1 + t 
2 2 n→ ∞n n n n n 
π (1 + t2 ) − 2t
∴ I= is equal to [AIEEE 2005] =∫ ⋅ e tdt
2 ( 1 + t2 ) 2
π 1
150 ∫ xf (sin x) dx is equal to (a)
2
tan 1 (b) tan1
=∫
et
⋅dt − ∫
2te t
⋅dt
0
[AIEEE 2006] 1 1 1+ t 2
( 1 + t2 ) 2
π π π /2 (c) cosec 1 (d) sec 1
(a) π ∫ f (sin x ) dx (b) ∫0 f (sin x ) dx 2 2 Using by parts,
0 2 Ans. (a) 1 −1
π /2 π I= ⋅et − ∫ ⋅2t ⋅ e t dt
(c) π ∫ f (cos x ) dx (d) π ∫ f (cos x ) dx 1 + t2 ( 1 + t2 ) 2
Let A = lim  2 sec2 2 + 2 sec2 2
1 1 2 4
0 0
n→ ∞  n n n n 2t e t
Ans. (c) −∫ dt
+ ... + 2 sec2 1
n ( 1 + t2 ) 2
π
Let I = ∫ x f (sin x) dx …(i) n 
0 et 2t e t 2t e t
⇒ I=∫
π
( π − x)f [sin ( π − x)] dx

1 1 1
2
= lim  sec2   + sec2  
2 2
2 =
1+ t 2
+ ∫ (1 + t2 )2 dt − ∫ (1 + t2 )2 dt
0 n→ ∞ n  n  n n  n
π  et x
⇒ I=∫ ( π − x) f (sin x) dx ∴ I= +C= +C
…(ii) n 
2
+ ... + sec2   
0 n 1 + t2 (log x)2 + 1
n  n 
On adding Eqs. (i) and (ii), we get 
π
2I = ∫ πf (sin x) dx 1 n r 2 r
2 154 Let f : R → R be a differentiable
0 = lim
n→ ∞
∑   sec  n 
n r = 1  n  1
π π
function having f (2) = 6, f ′ (2) =   .
⇒ I=
2 ∫0 f (sin x) dx …(iii)
1  48
⇒ A = ∫ x sec2 (x2 ) dx 3
 2a 2 f (x) dx, if f (2a − x) = f (x) 
a 0 f ( x) 4t
Q∫ f (x) dx =  ∫0  dt Then, lim ∫ dt is equal to
Put x2 = t ⇒ xdx = x→ 2 6 x −2
 0
 0 ,if f (2a − x) = − f (x)  2
π /2 1 1 1 1 [AIEEE 2005]
2 ∫0
⇒ I= π∫ f (sin x) dx A= sec2 t dt = [tan t] 10 = tan 1
0 2 2 (a) 18 (b) 12
π π 2 (c) 36 (d) 24
Put − x =t⇒ x = −t  (log x − 1) 
2 2 153 ∫  2
dx is equal to Ans. (a)
Put dx = − dt in Eq. (iii), we get  1 + (log x)  f (x )

π π /2 f (x ) 4t 3 ∫6 4t 3 dt
I= ∫
2 − π /2
f (cos t) dt [AIEEE 2005] lim
x→2 ∫6 x −2
dt = lim
x→2 (x − 2)
x xe x
π π /2 π /2 (a) + C (b) +C  form 0 
= ∫ f (cos x) dx = π ∫ f (cos x) dx (log x )2 + 1 1 + x2
2 − π /2 0  0 
x log x
[Qf (cos x) is an even function] (c) 2 +C (d) +C [by Leibnitz’s rule]
x +1 (log x )2 + 1
π /2 π /2
4 {f (x)} 3 3 ⇒ A∫ f (sin x) dx = π ∫ f (sin x) dx
= lim
x→2 1
f ′ (x) = 4 { f (2)} 3f ′ (2) 157 The value of ∫ |1 − x 2 | dx is 0 0
−2
QI = A π /2 f (sin x) dx,given
= 4 × (6) 3 ×
1 [AIEEE 2004]  ∫0 
48 28 14 7 1
(a) (b) (c) (d) ∴ A= π
 1  3 3 3 3
Qf (2) = 6 and f ′ (2) = ,given
 48  ex
Ans. (a) 160 If f (x) = ,
= 18 3 −1 1+ ex
∫ −2 |1 − x | dx = ∫ (x2 − 1) dx
2
f ( a)
1 2 1 3 −2
I1 = ∫ xg [x (1 − x)] dx and
155 If I 1 = ∫ 2 x
dx, I 2 = ∫ 2 x
dx, 1 3 f ( − a)
0
2 x2
0
2 3
+ ∫−1 (1 − x2 ) dx + ∫1 (x2 − 1) dx
I2 = ∫
f ( a)
g [x (1 − x)] dx, then the
I3 = ∫ 2 dx and I 4 = ∫ 2 x dx,  x3  
−1 1 3 f ( − a)
1 1 x3   x3  I2
=  − x  + x −  +  − x 
value of is
then [AIEEE 2005] 3  −2  3  −1  3 1 I1 [AIEEE 2004]
(a) I 3 > I 4 (b) I 3 = I 4
=  − + 1 + − 2 +  1 − + 1 − 
1 8 1 1 (a) 2 (b) – 3 (c) –1 (d) 1
(c) I 1 > I2 (d) I2 > I 1  3 3   3 3 Ans. (a)
Ans. (c)
+  9 − 3 − + 1
1 ex
Given that, 4 4 20 28  3  Given that, f (x) =
1 2 1 3 = + + = 1 + ex
I 1 = ∫ 2 x dx I2 = ∫ 2 x dx, 3 3 3 3
0 0 ea
∴ f (a) = …(i)
2 2 π / 2 (sin x + cos x) 2 1 + ea
158 The value of ∫
2 3
I 3 = ∫ 2 x dx and I 4 = ∫ 2 x dx dx
1 1 0 1 + sin 2x e −a 1
x3 x2 and f (−a) = = …(ii)
Since, 2 <2 for 0 < x < 1 is [AIEEE 2004] 1 + e −a 1 + e a
and
3
2 x > 2 x for x > 1
2 (a) 0 (b) 1 On adding Eqs. (i) and (ii), we get
(c) 2 (d) 3
1 3 1 2 f (a) + f (−a) = 1
∴ ∫0 2 dx < ∫ 2
x x
dx Ans. (c)
0
⇒ f (a) = 1 − f (−a)
2 3 2 2 π /2 (sin x + cos x)2
and ∫1 2 x dx > ∫ 2 x dx
1
Now, ∫
0 1 + sin 2x
dx Let f (−a) = t
⇒ f (a) = 1 − t
⇒ I2 < I 1 and I 4 > I 3 π /2 (sin x + cos x)2 1− t
2 =∫ dx Now, I 1 = ∫ xg [x (1 − x)] dx …(iii)
π cos x 0
(sin x + cos x)2 t
156 The value of∫ dx, where
−π 1+a x QI = bf (x) dx = bf (a + b − x) dx 
∫a ∫a
π /2
=∫ (sin x + cos x) dx  
a > 0, is [AIEEE 2005] 0
π 1− t
(a) 2π (b) = [− cos x + sin x] π0 / 2 ⇒ I1 = ∫ (1 − x) g [x (1 − x)] dx …(iv)
a π π t
π = − cos + sin + cos 0 − sin 0
(c) (d) aπ 2 2 On adding Eqs. (iii) and (iv), we get
2 1− t
= −0 + 1 + 1 − 0 = 2 2I 1 = ∫ g [x (1 − x)] dx = I2 [given]
Ans. (c) t
π cos2 x π π /2 I2
Let I = ∫ dx, a > 0 …(i) 159 If ∫ xf (sin x) dx = A ∫ f (sin x) dx, ∴ =2
−π 1+ ax 0 0 I1
Put x = − x, we get then A is equal to [AIEEE 2004] n
1
I=∫
π cos2 x
dx …(ii)
(a) 0 (b) π (c) π/4 (d) 2π 161 lim
n→ ∞
∑ n
e r / n is equal to
−π 1 + a −x r =1
Ans. (b) [AIEEE 2004]
π
On adding Eqs. (i) and (ii), we get Let I = ∫ xf (sin x) dx …(i) (a) e (b) e − 1 (c) 1 − e (d) e + 1
0
π (1 + a x ) cos2 x π
π Ans. (b)
2I = ∫ dx = ∫ cos2 x dx ∴ I=∫ ( π − x)f [sin ( π − x)] dx
−π (1 + a )x − π n
1 r /n 1

0
π Now, lim e = ∫ e x dx = [ e x ] 10
π  cos2 x + 1  ⇒ I=∫ ( π − x)f (sin x) dx …(ii) n→ ∞ n 0
=∫   dx 0
r =1
−π  2  =e−1
On adding Eqs. (i) and (ii), we get
π
1   sin 2 x
= 

+ x  2I = ∫
π
(x + π − x) f (sin x) dx 162 If f ( y) = e y , g ( y ) = y; y > 0 and
2  2   −π 0 t
π F (t) = ∫ f (t − y )g ( y ) dy, then
⇒ 2I = π ∫ f (sin x) dx 0
1  sin 2 π  sin (−2 π)  0 [AIEEE 2003]
= + π − − π 
2  2  2 
⇒ 2I = 2 π ∫
π /2
f (sin x) dx (a) F (t) = 1 − e (1 + t) −t

1 π 0 (b) F (t) = e t − (1 + t)
⇒ 2I = ( π + π) ∴ I = π /2
2 2 ⇒ I= π∫ f (sin x) dx (c) F (t) = te t
0
(d) F (t) = te − t
d  e sin x 
Ans. (b) Again using L’Hospital rule, 167 Let F (x) =   , x > 0.
Given that,f ( y) = e y , g ( y) = y 2x ⋅ 2 sec2 x2 ⋅ tan x2 ⋅ 2x + 2 sec2 x2 dx  x 
t = lim
and F (t) = ∫ f (t − y) g ( y) dy x→ 0 − x sin x + cos x + cos x 3 sin x3
4
0 If ∫ e dx = F (k ) − F (1), then
0 + 2 sec 0 2
x 1
t
= ∫ e t − y ⋅ y dy = e t
t −y = =1
0 ∫0 e y dy 0 + 2 cos 0 one of the possible value of k,is
[AIEEE 2003]
= e t  (− ye − y ) t0 − ∫ 1 (− e − y ) dy 
t 1
 0  165 The value of I = ∫ x (1 − x) n dx is (a) 15 (b) 16 (c) 63 (d) 64
0
Ans. (d)
= e t [(− te − t − 0) − ( e − y ) t0 ] [AIEEE 2003]
d e sin x
−t −t 1 1 Given, F (x) = , x>0
= e [− te
t
−e + 1] (a) (b) dx x
n+1 n+2
∴ F (t) = e t − (1 + t) On integrating both sides, we get
1 1 1 1
b (c) − (d) + e sin x
163 If f (a + b − x) = f (x), then ∫ xf (x) dx n+1 n+2 n+1 n+2 F (x) = ∫ dx …(i)
a x
Ans. (c) 4 3x2
is equal to [AIEEE 2003] 43 3 3

a+b b Given,
1
I = ∫ x (1 − x) dx n Also, ∫1 x
e sin x dx = ∫
1 x3
⋅ e sin x dx

2 ∫a
(a) f (b − x) dx 0

Put 1 − x = z ⇒ − dx = dz = F (k) − F (1)


a+b b
2 ∫a
(b) f (x) dx 0 Let x 3 = z ⇒ 3x2 dx = dz
∴ I = ∫ (1 − z) z (− dz)n
sin z
1 64 e
b −a b ∴ ∫1 dz = F (k) − F (1)
2 ∫a
(c) f (x) dx 1
= ∫ (1 − z) z dz n z
0
a+b b ⇒ 1 = F (k) − F (1)
[F (z)] 64
2 ∫a
(d) f (a + b + x) dx Q bf (x) dx = − a f (x) dx 
 ∫a ∫b  [from Eq.(i)]
Ans. (b) 1 ⇒ F (64) − F (1) = F (k) − F (1)
b
= ∫ (z n − z n + 1) dz ∴ k = 64
0
Let I = ∫ xf (x) dx …(i)
1
a
b zn + 1 zn + 2  168 If f (x) is a function satisfying
⇒ I = ∫ (a + b − x) f (a + b − x) dx = −  f ′ (x) = f (x) with f (0) = 1 and g (x) is a
a
 n + 1 n + 2 0
b 1 1
function that satisfies
⇒ I = ∫ (a + b − x)f (x) dx = − f (x) + g (x) = x 2 . Then, the value of
a
n+ 1 n+ 2 1
[Qf (a + b − x) = f (x), given] ∫ f (x)g (x) dx, is
0 [AIEEE 2003]

b
I = (a + b) ∫ f (x) dx − ∫ x f (x) dx
1 + 2 4 + 3 4 
b
  e2 5 e2 3
a a (a) e − − (b) e + −
 + . . . + n 
4
b 2 2 2 2
⇒ I = (a + b) ∫ f (x) dx − I 166 lim 2 2
e 3 e 5
n5
a
n→ ∞ (c) e − − (d) e + +
[from Eq. (i)]  1 + 23 + 3 3  2 2 2 2
 
b
⇒ 2I = (a + b) ∫ f (x) dx Ans. (c)
a
 + . . . + n 3  Given, f ′ (x) = f (x) and f(0) = 1
a + b  b − lim is equal to
∴ I=
 2 
 ∫a f (x) dx n→ ∞ n 5 Let f (x) = e x …(i)
2
x [AIEEE 2003] Also, f (x) + g (x) = x 2

∫0 sec 2 t dt (a) 1/30 (b) 0 ⇒ g (x) = x2 − e x …(ii)


164 The value of lim is (c) 1/4 (d) 1/5 1 1
x→ 0 x sin x
∫ 0 f (x) g (x) dx = ∫ 0 e (x − e ) dx
x 2 x
Now,
Ans. (d)
[AIEEE 2003]
(a) 3 (b) 2  1 + 24 + 34   1 + 23 + 33  [from Eqs. (i) and (ii)]
(c) 1 (d) –1     1 1
= ∫ x2 e x dx − ∫ e2 x dx
 + ... + n   + ... + n 
4 3
Ans. (c) lim  − lim 0 0
n→ ∞ n5 n→ ∞ n5 1 2x 1
x2 = [x2 e x − ∫ 2 xe x dx] 10 − [e ] 0
lim
∫0 sec2 t dt
 form 0  1 n r 1
4
1 n r
3 2
x→ 0 x sin x  0  = lim
n→ ∞
∑   − lim × lim ∑  
n r = 1 n  n → ∞ n n → ∞ n r = 1  n 
1
= [x2 e x − 2 xe x + 2 e x ] 10 − (e2 − 1)
2
Using L’Hospital rule, 1 1
1 1 1 = [(x2 − 2 x + 2) e x ] 10 − e2 +
= ∫ x 4 dx − lim
n ∫0
sec2 x2 ⋅ 2x × x 3 dx 2 2
= lim 0 n→ ∞ 1 1
x → 0 x cos x + sin x = [(1 − 2 + 2) e 1 − (0 − 0 + 2) e 0 ] − e2 +
1
 d x2   x5  1 2 2
=   −0 =
Q dx

∫0 sec 2 t dt = sec 2 x2 ⋅ 2x (Leibnitz’s rule)
  5 0 5 = e −2−
e2 1
+ =e−
e2 3

2 2 2 2
10x
169 ∫ | sin x | dx is equal to = [x] 1 2 + [2x] 3
+ [3x]2 3 π 
2 sin  − x 
0 π /2 2 
[AIEEE 2002] = 2 − 1+ 2 3 −2 2 + 6−3 3 I=∫ dx
(a) 20 (b) 8 (c) 10 (d) 18
0
π  π 
cos  − x  + sin  − x 
=5− 3 − 2 2  2 
Ans. (a)
π 2x (1 + sin x)
Since, | sin x | is a periodic function with 172 ∫ dx is equal to I=∫
π /2 cos x
dx …(ii)
period π. −π
1 + cos x 2 0 sin x + cos x
[AIEEE 2002]
10 π π
∴ ∫0 | sin x | dx = 10 ∫
0
| sin x | dx
(a)
π2
(b) π 2
On adding Eqs. (i) and (ii), we get
4 π /2
π
= 10 ∫ sin x dx = 10 [− cos x] 0π π 2I = ∫ 1 dx
0
0 (c) 0 (d)
2 π
= 10 [− cos π + cos 0] = [x] π0 / 2 ⇒ I =
Ans. (b) 4
= 10 [1 + 1] = 20
π 2 x (1 + sin x)
π /4 Let I = ∫ dx TOPIC 3
170 If I n = ∫ tan n x dx, then 1 + cos2 x
−π
0
=∫
π 2x
dx + ∫
π 2 x sin x Applications of Integrals
lim n[I n + I n + 2] is equal to − π 1 + cos2 x − π 1 + cos2 x
dx
n→ ∞
[AIEEE 2002]
1 ⇒ I =0 + 4∫
π x sin x
dx …(i)
174 The area, enclosed by the curves
(a) (b) 1 (c) ∞ (d) 0 0 1 + cos2 x y = sin x + cos x and y = |cos x − sin x|
2
π
 2x and the lines x = 0, x = , is
Ans. (b) Q is an odd function 2
 1 + cos x
2
π/4
Since, In = ∫ tann x dx [2021, 01 Sep. Shift-II]
0
2x sin x  (a) 2 2 ( 2 − 1) (b) 2( 2 + 1)
π/4 and is an even function
∴ In + 2 = ∫ tann + 2 x dx 1 + cos x2
 (c) 4( 2 − 1) (d) 2 2 ( 2 + 1)
0
π ( π − x) sin ( π − x) Ans. (a)
Now, I n + I n + 2 = ∫
π/4
tann x dx ⇒ I =4∫ dx
0
0 1 + cos2 ( π − x) Area
π/4 π /2
+ ∫0 tann + 2 x dx π π sin x = ∫ ((cos x + sin x) − | cos x − sin x |) dx
⇒ I =4∫ dx 0
π/4
0 1 + cos2 x π /4
=∫ tan x (1 + tan x) dx
n 2
=∫ ((cos x + sin x) − (cos x − sin x)) dx
0 π x sin x 0
π/4
−4 ∫ dx π /2
1 + cos2 x + ∫π / 4 ((cos x + sin x) − (sin x − cos x)) dx
0
=∫ sec2 x tann x dx
0
π sin x π /4 π /2
Put tan x = t ⇒ I = 4π ∫ dx − I [from Eq. (i)] =2∫ sin x dx + 2 ∫ cos x dx
0 1 + cos2 x 0 π /4
⇒ sec x dx = dt 2
π sin x −1   1 
1 ⇒ I =2π ∫ dx = 2 + 1 + 2 1 − 
tn + 1  1 n 1 0 1 + cos x 2  2   2
∴ I n + I n + 2 = ∫ t dt =   =
0
 n + 1 0 n + 1 = 2 2 ( 2 − 1)
Put cos x = t
n ⇒ − sin x dx = dt
⇒ lim n [I n + I n + 2 ] = lim 175 If the line y = mx bisects the area
n→ ∞ n→ ∞ n + 1 −1 1
∴ I = −2π ∫ dt enclosed by the lines x = 0 and y = 0,
= lim
1
=1
1 1 + t2
3
n→ ∞
1+
1
= 2 π[tan t] 1−1 −1 x = and the curve y = 1 + 4x – x 2 ,
n 2
π π
2 =2π  +  then 12 m is equal to
171 ∫ [x 2] dx is equal to  4 4 
0 [AIEEE 2002] [2021, 31 Aug. Shift-II]
= π2
(a) 2 − 2 (b) 2 + 2 Ans. (26)
(c) 2 − 1 (d) − 2 − 3 + 5 π /2 sin x x=0
y=mx
173 ∫ dx is equal to
Ans. (d) 0 sin x + cos x
2 [AIEEE 2002]
∫0 [x2 ] dx
(a)
π
(b)
π O
x=
3 y=0
4 2 2
1
= ∫ [x2 ] dx +
2 3 y = 1 + 4x – x2
0 ∫1 [x2 ] dx + ∫ 2
[x2 ] dx (c) 0 (d) 1
2 Ans. (a)
+ ∫ 3
[x2 ] dx
π /2 sin x
According to the question,
1 3/ 2 3/ 2
1 2 3 2 Let I = ∫
cos x +
dx …(i)
2 ∫0
(1 + 4x − x2 ) dx = ∫ mx dx
0
= ∫ 0 dx + ∫1 1 dx + ∫ 2 dx + ∫ 3 dx 0 sin x
0 2 3
1  x3  
3/ 2
m 3/ 2 177 The area of the region Now, required area, A0 =
⇒   x + 2x −   = [x] 0
2 0 2
2  3 
0
2 S = {(x, y) : 3x 2 ≤ 4y ≤ 6 x + 24} is ∫ f (x) dx + ∫ −f (x) dx
[2021, 26 Aug. Shift-I] −1 0
3 9 9 9m
⇒ + − = Ans. (27) 0
2 2 8 4
Y = ∫ (2x 3 − 3x2 − 12x) dx
39 −1
⇒ m= ⇒ 12m = 26
18 2
+ ∫ (12x + 3x2 − 2x 3) dx
3
y= x2 B y=3x/2 +6 0
176 The area of the region bounded by 4
0 2
the parabola (y − 2) 2 = (x − 1) , the (0, 0) x 4   x4 
A =  − x 3 − 6x2  + 6x2 + x 3 − 
tangent to it at the point whose 2  −1  2 0
ordinate is 3 and the X-axis is X¢ X =
114
[2021, 27 Aug. Shift-II] (–4, 0) (–2, 0) (0, 0) (4, 0) 4
(a) 9 (b) 10 (c) 4 (d) 6 Y¢ ∴4A = 114
Ans. (a)  3  3x 
We have, y =   x2 and y =   + 6 179 If the area of the bounded region
Given parabola,  4 2
R = (x, y) :max {0,log e x } ≤ y ≤ 2x , ≤ x ≤ 2
1
(y − 2)2 = (x − 1 ) … (i) 3x 2
3x
⇒ = + 6 ⇒ 3x2 = 6x + 24  2 
Since, Ordinate = y = 3 4 2
Then, x = 2 ⇒ x2 − 2x − 8 = 0 ⇒ (x − 4) (x + 2) = 0 is, α (log e 2) −1 + β (log e 2) + γ , then the
Point on parabola (2, 3) ⇒ x = −2, 4 value of (α + β − 2γ)2 is equal to
Differentiating Eq. (i) w.r.t. x, we get ⇒ y = 3, 12 [2021, 27 July Shift-I]
dy
2(y − 2) =1 A(−2, 3) and B(4, 12) (a) 8 (b) 2 (c) 4 (d) 1
dx
 3x   3x2 
4
Ans. (b)
∴ Required area = ∫  + 6 −   dx
Y
−2
2   4  R = {(x, y) :max {0,log e x } ≤ y ≤ 2x ,
1
4 ≤ x ≤ 2}
(2, 3)  3x2 x3  2
= + 6x − 
 4 4  −2 4
= [(12 + 24 − 16) − (3 − 12 + 2)]
(5, 0) = (20 + 7)
X′ X
(–4, 0) y=2x
= 27 sq units
Y′ y=loge x
178 Let a and b respectively be the √2
dy 1 points of local maximum and local
= y=0
dx 2(y − 2) minimum of the function
f (x) = 2x 3 − 3x 2 − 12x . If A is the
At (2, 3)
dy 1 total area of the region bounded by x=1/2 x=1 x=2
= y = f (x), the X-axis and the lines  0 1
dx 2 , ≤ x< 1
x = a and x = b, then 4A is equal to max {0,log e x } =  2
log e x ,1 ≤ x ≤ 2
Equation of tangent at (2, 3)
[2021, 26 Aug. Shift-II]
1
y − 3 = (x − 2) Ans. (114)
1 2
2 Area = ∫ 2x dx + ∫ (2x − log e x)dx
or x − 2y + 4 = 0 We have,f (x) = 2x 3 − 3x2 − 12x 1 1

Intersection point of parabola on X-axis ∴ f ′(x) = 6x2 − 6x − 12 = 6(x2 − x − 2) 2


1 2
is = 6(x + 1)(x − 2)  2x   2x 
=  +  − [x log e x − x] 1
2
y = 0, x = 5 i.e (5, 0) f ′(x) = 0
 log e 2  1  log e 2  1
Intersection point of tangent and X-axis ⇒ x = −1 and 2 2
y = 0, x = − 4 i.e (− 4, 0) ∴x = − 1 and 2 are critical points 2− 2 4−2
= + − [(2 log e 2 − 2) − (0 − 1)]
Area of shaded region log e 2 log e 2
3
= ∫ [(y − 2)2 + 1 − (2y − 4)] dy 4− 2
0
= − 2 log e 2 + 1
2 log e 2
3 –1 0
= ∫ (y2 − 6y + 9)dy On comparing with expression,
0
3 α(log e 2) − 1 + β log e 2 + γ
 y3  α = 4 − 2, β = − 2, γ = 1
=  − 3y2 + 9y  = 9 sq unit.
 3 0 ∴a = − 1 and b = 2 ∴ (α + β − 2γ)2 = (4 − 2 − 2 − 2)2 = 2
180 The area of the region bounded by ⇒ x = 1, − 2 (Reject -2) 1
0 ≤ f (t) ≤ for all t ∈(1, 3]. The largest
y − x = 2 and x 2 = y is equal to Area of shaded region 2
1 1
[2021, 27 July Shift-II]
= ∫ (4 − 2x) dx − ∫ 2x2 dx
possible interval in which g (3) lies
16 2 9 4 is [2021, 18 March Shift-II]
(a) (b) (c) (d) 0 0
3 3 2 3
 3 
(a)  −1, − 
1 1
= [4x − x2 ] 10 −  x 3 
2 (b) − , − 1
Ans. (c)  3  0  2   2 
 
The given equations are
= (4 −1) −   = (c)  , 2
2 7 1
y − x =2 …(i)  3 3  3 
(d) [1, 3]
and x2 = y …(ii)
Ans. (c)
x2=y Y 182 The area (in square units) of the x
region bounded by the curves Given, g (x) = ∫ f (t) dt
y–x=2
x 2 + `2y − 1 = 0, y 2 + 4x − 4 = 0 and 0

y 2 − 4x − 4 = 0, in the upper half 3 1 3


∴ g (3) = ∫ f (t) dt = ∫ f (t) dt + ∫ f (t) dt
X′
–1 2
X plane is ……… .
0 0 1
[2021, 22 July Shift-II]
1 3 1 3
Ans. (2) 1 1
Y′
 1 1
⇒ ∫ 3 dt + ∫ 0 ⋅dt ≤ g (3) ≤ ∫ 1 dt + ∫ 2 dt
C 1 ⇒ x + 2y − 1 = 0 ⇒  y −  = 4⋅ x2
2 0 1 0 1
From Eqs. (i) and (ii),  2 8 1
x2 = x + 2 ⇒ ≤ g(3) ≤ 1 + 1
C2 ⇒ y2 + 4x − 4 = 0 ⇒y2 = − 4(x + 1) 3
⇒ x − x −2=0
2
C 3 ⇒ y2 − 4x − 4 = 0 ⇒y2 = 4(x + 1) 1
⇒ (x + 1) (x − 2) = 0 ⇒ ≤ g(3) ≤ 2
C2 C3 3
⇒ x = − 1, 2 (0,
2)
A
Area of shaded region 0, 1
2 184 The area bounded by the curve
= ∫ [(x + 2) − x2 ] dx 2
−1 4y 2 = x 2(4 − x )(x − 2) is equal to
2 B C (1, 0)
 x2 x3  (–1, 2) D
y=1 [2021, 18 March Shift-II]
=  + 2x −  2
2 3  −1 π 3π
(a) (b)
1 1 8 8
=  2 + 4 −  −  − 2 + 
8
 3 2 3 3π π
C1 (c) (d)
9 2 16
= sq units
2 Ans. (c)
x = –1
Given, equation of curve is
181 The area (in sq. units) of the region, A is the point of intersection ofC2 and C 3
4y2 = x2 (4 − x) (x − 2)
given by the set {(x, y) ∈R × R| x ≥ 0, (0, 2).
2x 2 ≤ y ≤ 4 − 2x} is  1 ⇒ 4y2 = x2 (4 − x) (x − 2)
B  0, 
[2021, 25 July Shift-I]  2 | x|
8 17 ⇒ | y| = (4 − x) (x − 2)
(a) (b) C is the point of intersection ofC 1 and C2 2
3 3 (1, 0).
13 7  y =2 x =1   x (4 − x)(x − 2), x > 0
(c) (d) Area = 2  ∫ x dy − ∫ y dx  
3 3 y =0 y= 2

3
1 x =0 x
424 3 1 424 − (4 − x) (x − 2), x < 0
Ans. (d) Area of BCD  2
Area of ADC
 x≥0 x
 2
 4 − y2  1
 1 − x2  Let y1 = (4 − x) (x − 2)
 y ≥ 2x
2
= 2∫   dy − ∫   dx 2
y ≤ 4 − 2x 0
4  0
2  x
 y2 = − (4 − x) (x − 2)
Y  y 3   x x 3  
2 1 2

2x y=x 2 = 2 y −  −  −   Y
+y  12  0  2 6  0 
=4 
= 2 −  = 2
4 1
(0, 4)
 3 3 X′ X
(1, 2) O (2, 0) (3, 0) (4, 0)
(1, 0) x
(0, 0) (2, 0)
X 183 Let g (x) = ∫ f (t) dt, where f is
X 0 Y′
Y continuous function in [0, 3] such
1 and domain x ∈[2, 4]
4 − 2x = 2x2 that ≤ f (t) ≤ 1 for all t ∈[0, 1] and [Q (4 − x)(x − 2) ≥ 0
⇒ 2x + 2x = 4
2 3
⇒(x − 2) (x − 4) ≤ 0 ⇒2 ≤ x ≤ 4]
4
Ans. (a) 1
∴Required area = ∫ (y 1 − y2 ) dx =×4×2=4
y 2
2
4 A B Since, only one curve is given, here
= ∫ x (4 − x) (x − 2) dx …(i) assume the area bounded by x-axis.
2 Then, the area will be 4 square unit.
b b
x
Using property, ∫ f (x) dx = ∫ f (a + b − x) dx O C π/2 188 The graphs of sine and cosine
a a
functions, intersect each other at a
From Eq. (i), number of points and between two
4
Area = ∫ (6 − x) (4 − x)(x − 2) dx … (ii) consecutive points of intersection,
2 the two graphs enclose the same
From Eqs. (i) and (ii), A1 is the area of region OAB. area A. Then A 4 is equal to ………… .
4
[2021, 25 Feb. Shift-I]
2A = 6∫ (4 − x) (x − 2) dx A2 is the area of region OBC.
π 1  Ans. (64)
2 Coordinate of B is  , 
4  4 2 Y
⇒ A = 3∫ 1 − (x − 3)2 dx π
2 Now, A1 = ∫ ( cos x − sin x)dx
4
0
π 3π
⇒ A = 3. .(1)2 = = [sin x + cos x] π0 / 4
2 2 π π p 5p/4
=  sin + cos  − (0 + 1) O p/4 p/2
X
 4 4
185 Let f : [−3, 1] → R be given as
= 2 −1
min {(x + 6), x 2 }, − 3 ≤ x ≤ 0 π π
f (x) =  . A2 = ∫ 4 sin x dx + ∫π2 cosx dx
2
 max { x , x }, 0≤ x ≤ 1  0
4
π π

If the area bounded by y = f (x) and A2 = [− cos x] 04 + [sin x] 2π Required area of shaded region
X -axis is A, then the value of 6A is 5π /4
A=∫
4
(sin x − cos x) dx
equal to ………… . = 2 ( 2 − 1) π /4

[2021, 17 March Shift-II] Now, A1 : A2 = ( 2 − 1) : 2 ( 2 − 1) = [− cos x − sin x] 5π π/ 4/ 4


A1 : A2 = 1 : 2  5π 5π  π π 
+ sin  −  cos + sin  
Ans. (41)
= −   cos
A = Area bounded by y = f (x) and X-axis. and  4 4   4 4 
−2 0 1 A1 + A2 = ( 2 − 1) + 2 ( 2 − 1)  1 1   1 1 
= −  − −  − + 
= ∫ (x + 6) dx + ∫ x dx + ∫
2
2  
x dx = ( 2 − 1)( 2 + 1)
−3 −2 0
 2 2  2
A1 + A2 = 2 − 1 = 1 4
 x2 
−2
 x3  1 0 ∴ A= =2 2
=   + 6[x] −− 23 +   +  x 3/ 2 
2 Therefore, 2
  0 A1 : A2 = 1 : 2,
 2 − 3  3  − 2 3 ⇒ A 4 = (2 2) 4 = 64
41 A1 + A2 = 1
=
6 189 The area of the region
41 187 The area bounded by the lines
∴ 6A = 6 × ⇒ 6A = 41 R = {(x, y): 5x 2 ≤ y ≤ 2x 2 + 9} is
6 y = || x − 1 | − 2 | is ……… .
[2021, 24 Feb. Shift-II]
[2021, 26 Feb. Shift-I]
(a) 11 3 sq units
186 Let A 1 be the area of the region Ans. (4)
(b) 12 3 sq units
bounded by the curves y = sin x, Given, y = | | x − 1 | − 2 |
(c) 9 3 sq units
y = cos x and y-axis in the first Required area is area of ∆PQR.
Q (1, 2) (d) 6 3 sq units
quadrant. Also, let A 2 be the area
of the region bounded by the Ans. (b)
(0, 1)
curves y = sin x, y = cos x, x-axis and Given, R = {(x, y) : 5x2 ≤ y ≤ 2x2 + 9}
P
π Here, we have two curves y = 5x2 and
x = in the first quadrant. Then, (–1, 0) A (1, 0) R (3, 0)
y = 2x2 + 9, point of intersection of both
2
[2021, 26 Feb. Shift-II]
curves is find by solving both equations
i.e.
(a) A1 : A2 = 1 : 2 and A1 + A2 = 1
1 5x2 = 2x2 + 9
(b) A1 = A2 and A1 + A2 = 2 Area = × (Base) × (Height)
2 ⇒ x2 = 3
(c) 2A1 = A2 and A1 + A2 = 1 + 2
=
1
× (PR) × (AQ ) ⇒ x=± 3
(d) A1 : A2 = 1 :2 and A1 + A2 = 1
2
y 1  43 1  1 175 1/ 2
y=5x2 y=2x2+9 =
× +  ×7= × ×7  x2 2x 3/ 2 
2  3 4 2 12 = 4 x − − 
 2 3 2 0
1 175
∴ 24A = 24 × × × 7 = 1225 1 1 2 
2 12 =4 − −
9  2 8 12 
But this question is wrong as in question
x it is mentioned that the triangle is Y
–√3 O √3
formed with the positive X-axis which 2 (0, 1)
contradicts the solution. 2y =–x (1/2,1/2) 2y2=x
3
∴ Area = ∫ (2x + 9 − 5x ) dx
2 2
− 3
191 The area (in sq. units) of the part of x+y=1
3
= 2∫ (9 − 3x2 ) dx the circle x 2 + y 2 = 36, which is X′
(–1/2,0) O
X
0 (–1, 0) (1/2,0) (1, 0)
= 2[9x − x 3] 0 3 outside the parabola y 2 = 9x, is
[2021, 24 Feb. Shift-I]
= 2[9 3 − 3 3]
(a) 24 π + 3 3 (b) 24 π − 3 3 (0, –1)
= 12 3 sq units. (c) 12 π + 3 3 (d) 12π − 3 3 Y′
Ans. (b)  12 − 3 − 4  5
190 If the area of the triangle formed =4  = sq units
Given, equation of circle ⇒x + y = 36 2 2
 24  6
by the positive X-axis, the normal Equation of parabola ⇒y2 = 9x
and the tangent to the circle We have to find area of shaded region. 193 Area (in sq. units) of the region
(x − 2) 2 + (y − 3) 2 = 25 at the point Y | x| | y|
(5, 7) is A, then 24A is equal to … . outside + = 1 and inside the
2 3
[2021, 24 Feb. Shift-II]
y2=9x x2 y2
36

P
ellipse + = 1 is
=

Ans. (1225)
y2

4 9
+
x2

Given, circle, (x − 2)2 + (y − 3)2 = 52 [2020, 2 Sep. Shift-I]


c = (2, 3) (a) 6 (π − 2)
X′ X
r=5 O 3 6 (b) 3(π − 2)
Y
(c) 3 (4 − π )
(d) 6 (4 − π )
P (5, 7) Q Ans. (a)
A Equation of given curves
Y′ |x | |y |
(2,3) + =1 … (i)
Required area 2 3
= πr2 − 2 ∫ 9x dx + ∫ 36 − x2 dx 
3 6 2 2
X x y
M O Q N  0 3  and + =1 … (ii)
4 9
6
36 − x2 + 18 sin− 1  
x x On plotting the graph of given curves due
= 36 π − 12 3 − 2
2 6 3
to symmetry, we can say the required
Equation of normal at P (i.e. PA line)  area (area of shaded region)
9 3 
 7 − 3 = 36 π − 12 3 − 29 π −  + 3 π  
⇒ (y − 7) =   (x − 5) Y
 5 − 2   2   B (0, 3)

⇒ 3y − 21 = 4x − 20 = (24 π − 3 3) x2 y
2
+ =1
4 9
⇒ 4x − 3y + 1 = 0
−1 192 The area (in sq. units) of the region
Therefore, M =  , 0  [Put y = 0 in (–2, 0) A
 4  A = {(x, y):| x | +| y | ≤ 1, 2y 2 ≥| x |} is X′
A′ O (2, 0)
X

above equation] [2020, 6 Sep. Shift-I]


Now, equation of tangent at P. 1 7 1 5
(a) (b) (c) (d)
−3 3 6 6 6
y−7= (x − 5) B′ (0, –3)
4 Ans. (d) Y′
−1 The area of the given region
[Qslope of PN = ]
Slope of PA = Area enclosed by ellipse − 4(Area of ∆AOB)
A = {(x, y) : | x | + | y | ≤ 1 , 2y2 ≥ | x | }
⇒ 4y − 28 = − 3x + 15 1 
by the figure due to symmetry = π(2)(3) − 4  × 2 × 3
⇒ 3x + 4y = 43 2 
1/ 2
 x
 43  A =4 ∫ 1 − x − dx x2 y2
Therefore, N =  , 0   2  [Qarea enclosed by ellipse + = 1 is
3  0
a 2
b2
[Put y = 0 in above equation] [Q2y + y − 1 = 0 ⇒(2y − 1)(y + 1) = 0
2
πab sq units]
1

1
Area (A) = × MN × PQ ⇒y = ] = 6 π − 12 = 6( π − 2) sq units
2 2
1 1 1 4 1
194 Consider region R = {(x, y) ∈R 2 : =+ 1− − + + 2− − 1 Now, on drawing the graph of given
3 24 2 2 2 region with the help of equation of curves
x 2 ≤ y ≤ 2x }. 1 1 72 + 8 − 1 79 y = (x – 1)[x] and y = 2 x
=3+ − = = sq unit
If a line y = α divides the area of 3 24 24 24 y
region R into two equal parts, then Hence, option (b) is correct. y=2√x
which of the following is true?
[2020, 2 Sep. Shift-II]
196 The area (in sq. units) of the largest
(a) α 3 − 6α 2 + 16 = 0
rectangle ABCD whose vertices A x–
1
(b) 3α 2 − 8α 3/2 + 8 = 0 y=
(c) 3α 2 − 8α + 8 = 0 and B lie on the X-axis and vertices O 1 2
x

(d) α 3 − 6α 3/2 − 16 = 0 C and D lie on the parabola,


Ans. (b) y = x 2 − 1 below the X-axis, is ∴ Area of given region
1 2
According to the question, [2020, 4 Sep. Shift-II] = ∫ 2 x dx + ∫ (2 x – x + 1)dx
4 2 1 4 0 1
y=x2 y=2x (a) (b) (c) (d) 2
y 1
4 x2 
=  x 3/ 2  +  x 3/ 2 – + x 
3 3 3 3 3 3 3 4
(2, 4)  3  0 3 2
Ans. (a)  1
y=α Equation of given parabola y = x2 – 1 4 8 2 4 1 
= + –2 + 2 – + – 1
x According to symmetry let, the 3  3 3 2 
O
coordinate of A (–a, 0), B (a, 0) C (a,a2 – 1) 8 2 1
and D (–a, a2 – 1). = – sq units.
α y 4 3 2
∫0  y −  dy = ∫ ( y − y /2) dy
2 α
∴Area of rectangle P (a) = 2a (a2 – 1) 198 The area (in sq. units) of the region
α α α2 4(2) 16 α α α2
⇒ − = − − + Y enclosed by the curves y = x 2 − 1
3 /2 4 3 /2 4 3 /2 4
and y = 1 − x 2 is equal to
4 α2 64 − 48 16 4
⇒ α α− + = = y=x2–1
[2020, 6 Sep. Shift-II]
3 2 12 12 3 4 8 7 16
(a) (b) (c) (d)
⇒ 3 α2 − 8 α 3/ 2 + 8 = 0 3 3 2 3
Hence, option (b) is correct. A(–a, 0) B(a, 0) Ans. (b)
X
195 The area (in sq. units) of the region (–1, 0) O (1, 0) From the graph of given curves, due to
symmetry the area of the region
{(x, y) : 0 ≤ y ≤ x 2 + 1, 0 ≤ y ≤ x + 1,
1 D C(a, a2–1) enclosed
≤ x ≤ 2} is (–a, a2–1) (0, –1)
2 [2020, 3 Sep. Shift-I] (0, 1) y=x2–1
23 79 Now, for maxima P ' (a) = 0
(a) (b)
16 24 ⇒ 2(a2 – 1) + 4a2 = 0
79 23 ⇒ 3a2 = 1
(c) (d) (–1, 0) (1, 0)
16 6 1
⇒ a= units
Ans. (b) 3 (0, –1) y=1 –x2
Given region ∴ Area of largest rectangle is
1
{(x, y) :0 ≤ y ≤ x2 + 1, 0 ≤ y ≤ x + 1, ≤ x ≤ 2 } 2 1  4
 – 1 = sq units

1
2 x3 
1
3  3  3 3
Y y=x2+1 = 4∫ (1 − x2 ) dx = 4  x − 
0  3 0
y=x+1 197 The area (in sq. units) of the region
= 4   = sq units
2 8
A = {(x, y) : (x − 1)[x] ≤ y ≤ 2 x ,  3 3
0 ≤ x ≤ 2}, where [t] denotes the
greatest integer function, is 199 The area (in sq. units) of the region
[2020, 5 Sep. Shift-II] {(x, y) ∈R 2 |4x 2 ≤ y ≤ 8x + 12} is
8 1 4 [2020, 7 Jan. Shift-II]
(0, 1) (a) 2− (b) 2 +1
3 2 3 124 125
(a) (b)
X 8 4 1 3 3
(–1, 0) O (1/2, 0) (1, 0) (2, 0) (c) 2 −1 (d) 2− 127 128
3 3 2 (c) (d)
= Shaded region in the figure Ans. (a) 3 3
∴Area of required region is As we know that, Ans. (d)
1 2
=∫ (x2 + 1) dx + ∫1 (x + 1)dx  0, 0 ≤ x < 1 The area of region
1/ 2  {(x, y) ∈ R2 |4x2 ≤ y ≤ 8x + 12} is area of
1 2 y = (x – 1)[x] =  x – 1, 1 ≤ x < 2
 x3   x2  region bounded by curves y = 4x2
=  + x +  + x 2(x – 1), x = 2
3 2  and y = 8x + 12.
  1/ 2  1
To get the point of intersection of ∴The area of the region, enclosed by the ∴ a satisfies the equation
curves, on eliminating y, we get circle x2 + y2 = 2, which is not common x 6 − 12x 3 + 4 = 0
4x2 = 8x + 12 to the region bounded by the parabola Hence, option (b) is correct.
1
⇒ x2 − 2x − 3 = 0 y2 = x and the straight line y = x, is 2 π −
6 202 The area (in sq. units) of the region
⇒ x = − 1, 3 1
= (12 π − 1). {(x, y) ∈R 2 : x 2 ≤ y ≤ 3 − 2x}, is
Y 6 [2020, 8 Jan. Shift-II]
31 32
(a) (b)
+1
2 (3, 36) 201 For a > 0, let the curves C 1 : y 2 = ax 3 3
8x
y=
y=4x2
and C 2 : x 2 = ay intersect at origin O (c)
29
(d)
34
(0, 12) and a point P. Let the line 3 3
(–1, 4)
X′ X
x = b (0 < b < a) intersect the chord Ans. (b)
O OP and the x-axis at points Q and R, The area of the region {(x, y) ∈ R2 :
(–3/2, 0)
respectively. If the line x = b bisects x2 ≤ y ≤ 3 − 2x } is the area of region
Y′
the area bounded by the curves, C 1 bounded by curves y = x2 and y = 3 − 2x.
3 1 Y
So, required area = ∫ (8x + 12 − 4x2 )dx and C 2 , and the area of ∆OQR = ,
−1
2
3 then ‘a’ satisfies the equation (0, 3)
 8x2 x3 
= + 12x − 4  [2020, 8 Jan. Shift-I]
(3/2, 0)
 2 3  −1 (a) x 6 + 6 x 3 − 4 = 0 (b) x 6 − 12 x 3 + 4 = 0 X′ X

= {(4 × 9) + (12 × 3) − (4 × 9)} − 4 − 12 + 


4 (c) x 6 − 6 x 3 + 4 = 0 (d) x 6 − 12 x 3 − 4 = 0 Y′
 3 Ans. (b)
20 128 On solving y = 3 − 2x and y = x2 ,
= 36 + = sq units Given curvesC 1 : y2 = ax, andC2 : x2 = ay,
3 3 (a > 0), intersect each other at origin ‘O’ we have x2 = 3 − 2x ⇒ x2 + 2x − 3 = 0
and a point P (a, a). ⇒ x = −3, 1, then (x, y) = (−3, 9) and (1, 1)
200 The area of the region, enclosed by 1
QO(0, 0), Q and P (a, a) are collinear. So, So, required area = ∫ [(3 − 2x) − x2 ]dx
the circle x 2 + y 2 = 2 which is not Q (b, b).
common to the region bounded by −3
Y 1
x2=ay  2x2 x 3 
the parabola y 2 = x and the straight = 3x − − 
line y = x, is [2020, 7 Jan. Shift-I] x2=ax  2 3  −3
1
(a) (12 π − 1)
1
(b) (12 π − 1)
P (a, a)  1
=  3 − 1 −  − (−9 − 9 + 9)
3 6 Q (b, b)  3
1 1 X′ O X 5 32
(c) (24 π − 1) (d) (6 π − 1) R (b, 0) = 9 + = sq units
6 3 3 3  1
x , 0≤ x <
Ans. (b)  2
Let the area of the region, enclosed by the
(0<b<a)  1 1
x=b 203 Given, f (x) =  , x=
parabola y2 = x and straight line y = x is
1
Y′ 1 −2 x , 1 2
A = ∫ ( x − x)dx Now, as it is given area of ∆OQR =
1  < x≤1

0
Y 2 2
y=x 1 2 1
2 2 y2=x ⇒ b = ⇒ b=1 (Qb > 0)
x +y =2 2 2 1 2
and g (x) =  x −  , x ∈R. Then the
A (1, 1)
QThe line x = b bisects the area bounded  2
X′ O X by the curves,C 1 and C2 , so

area (in sq. units) of the region
x2   x2 
1 a

∫ 
 a x −
a

dx = ∫  a x −  dx bounded by the curves y = f (x) and
1
a
0 y = g (x) between the lines, 2x = 1
1
and 2x = 3, is [2020, 9 Jan. Shift-II]
a
 x 3/ 2 1 x 3   x 3/ 2 1 x 3 
Y′ ⇒ a −  = a − 
1  3 /2 a 3  0  3 /2 a 3  1 1 3
 x 3/ 2 x2   1 1 (a) +
= −  = −  2 a 1 2a a 2
2 a 12 2 4
 3 / 2 2 0  3 / 2 2  ⇒ − = − − +
3 3a 3 3 3 3a 1 3
2 1 (b) +
= −  4 a a2 2 3 4
 3 2 ⇒ = +
3 3 3a 1 3
4−3 1 −
= = ⇒ 4a a = a 3 + 2 (c)
6 6 2 4
⇒ 16a 3 = a 6 + 4 + 4a 3
QArea of circle having radius r = 2 unit 3 1
is πr2 = 2 π ⇒ a 6 − 12a 3 + 4 = 0 (d) −
4 3
2
Ans. (d) Ans. (b)  3  9
⇒ x +  ≥ y + 
On drawing the given curves Given equation of curve is x = 4y, which
2
 2  4
 1 represent a parabola with vertex (0, 0) 0≤ y≤4
x, 0 ≤ x < Since,
 2 and it open upwar(d)
 1 and 0 ≤ x ≤ 3
1
y = f (x) =  , x= x2 ∴The diagram for the given inequalities
 2 2 Y y=
4 is
 1 − x, 1 < x ≤ 1 x+2 Y
 2 y=
4 y=x2+3x
2 B
and y = g (x) =  x −  , we have
1 A
 2 X′ X
–1 O 2 y=4
Coordinate of points –3/2
 3 X
1  3 (–3, 0) O 1 3
A  , 0  , B (1, 0), C  , 1 −  and –9
2   2 2  Y′ 4 x=3
 1 1
D ,  Now, let us find the points of and points of intersection of curves
 2 2
intersection of x2 = 4y and 4y = x + 2 y = x2 + 3x and y = 4 are (1, 4) and (−4, 4)
Y For this consider, x2 = x + 2 Now required area
⇒ x2 − x − 2 = 0 1 3
⇒ (x − 2) (x + 1) = 0 = ∫ (x2 + 3x)dx + ∫ 4 dx
D (1/2, 1/2) g(x) ⇒ x = − 1, x = 2 0 1
y=1–x 1 1
y=x When x = − 1, then y = x 3x 3 2
4 = +  + [4x] 1
3
C (
√3 ,1– √3
2 2 ) and when x = 2, then y = 1  3 2 0
X′ Thus, the points of intersection are 1 3
O A (1/2, 0) B (1, 0)
X = + + 4(3 − 1)
A  − 1,  and B (2, 1).
1 3 2
Y′  4 2+ 9 11
= + 8= + 8
So, required area = area of shaded Now, required area = area of shaded 6 6
region region 59
2 = sq units
3
 1 
2 = ∫ {y (line ) − y (parabola )} dx 6
= ∫ 1 2  1 − x −  x −  dx −1
 2   2 x+2 x2 
2   = ∫  −  dx 206 Let S (α) = {(x, y) : y 2 ≤ x, 0≤ x ≤ α}
−1
3  4 4
and A(α) is area of the region S(α). If
2  − x2 + x −  dx
1
= ∫1 1 −x 2
for λ, 0 < λ < 4, A(λ ) : A(4) = 2 : 5, then
 4 1  x2 x3 
=  + 2x − 
2
42 3  −1 λ equals [2019, 8 April Shift-II]
3
3  4
1/ 3 1/ 3
(b) 4 
= ∫ 1 2  − x2 dx 1 1 1 2
=   2 + 4 −  −  − 2 +
8 (a) 2 
4    25   5
2 4  3 2 3  
1/ 3 1/ 3
4
(d) 2 
3
1 1  2
3 (c) 4 
x3  2 = 8− −3  25   5
=  x −  4  2 
4 3 1 Ans. (c)
2 1 1 9
= 5 − = sq units
3 3 3 3 3 1 4  2  8 Given, S (α) = {(x, y) : y2 ≤ x, 0 ≤ x ≤ α } and
= − − +
8 24 8 24 A(α) is area of the region S (α)
205 The area (in sq units) of the region Y
 3 1
=  −  sq units y2=x
 4 3
A = {(x, y) ∈R × R | 0 ≤ x ≤ 3, 0 ≤ y ≤ 4,
y ≤ x 2 + 3x} is [2019, 8 April Shift-I]
Hence, option (d) is correct.
53
(a) (b) 8
204 The area (in sq units) of the region 6 X
O
bounded by the curve x 2 = 4y and 59 26 A(λ)
(c) (d)
the straight line x = 4y − 2 is 6 3
[2019, 11 Jan. Shift-I] Ans. (c)
(a)
7
(b)
9 Given, y ≤ x2 + 3x x=λ
8 8  3 9
2
λ
λ
5 3 ⇒ y ≤ x +  −  x 3/ 2  4 3/ 2
(c) (d)  2 4 Clearly, A (λ) = 2∫ x dx = 2   = λ
4 4 0  3 /2  0 3
A (λ) 2 Ans. (d)
Since, = , (0 < λ < 4) Y
A (4) 5  y2  (0, 2)
3 2 Given region A = (x, y) : ≤ x ≤ y + 4
λ3/ 2 λ
= ⇒   =  
2 2 2
⇒  
43/ 2 5  4  5 (–2, 0) (2, 0)
y2 X′ X
λ 4
1/ 3 1/ 3 ∴ =x O
4
⇒ =  ⇒ λ = 4  2
4  25   25 
⇒ y2 = 2x …(i)
(0, –2)
and x = y + 4 ⇒y = x − 4 …(ii)
207 The area (in sq units) of the region Graphical representation of A is Y′
A = {(x, y) : x 2 ≤ y ≤ x + 2} is Now, the area of shaded region is equal
Y
[2019, 9 April Shift-I] y2 to the area of a square having side length
=x
13 9 Q 2
(a) (b) (2 − 0)2 + (0 − 2)2 = 2 2 units.
6 2
31 10
(c) (d) X' X 210 The area (in sq units) of the region
6 3 O
bounded by the curves y = 2 x and
Ans. (b)
Given region is A = {(x, y) : x2 ≤ y ≤ x + 2}
P y = | x + 1 |, in the first quadrant is
4
y+

[2019, 10 April Shift-II]


Now, the region is shown in the following
x=

graph Y' 3
(a)
Y y=x+2 On substituting y = x − 4 from Eq. (ii) to 2
Eq. (i), we get 3
x2=y (b) log e 2 +
(x − 4)2 = 2x 2
B(2,4) ⇒ x − 8x + 16 = 2x
2 1
(c)
⇒ x2 − 10 x + 16 = 0 2
3 1
A
(0,2) ⇒ (x − 2)(x − 8) = 0 ⇒ x = 2, 8 (d) −
∴ y = −2, 4 [from Eq. (ii)] 2 log e 2
(–2,0)
X' –1 X' So, the point of intersection of Eqs. (i) Ans. (d)
O 2
and (ii) are P(2, − 2) and Q (8, 4). Given, equations of curves
Y'
Now, the area enclosed by the region A  x + 1 ,x ≥ − 1
For intersecting points A and B  y = 2x and y = | x + 1 | = 
y2 
4
Taking, x2 = x + 2 ⇒x2 − x − 2 = 0 = ∫  (y + 4) − dy − x − 1 , x < − 1
−2 
2 Q The figure of above given curves is
⇒ x2 − 2x + x − 2 = 0
4
⇒ x (x − 2) + 1(x − 2) = 0 y 2
y 3
Y
=  + 4y −  y=x+1
⇒ x = −1, 2 ⇒ y = 1,4  2 6  −2
(1,2)
So, A(−1, 1) and B (2, 4).
 16 64   4 8
2 =  + 16 −  −  − 8 +  y=–x–1 y=2x
2 6  2 6
Now, shaded area = ∫ [(x + 2) − x2 ] dx
32 4 (0,1)
−1 = 8 + 16 − −2+ 8−
2 3 3
 x2 x3  X′ X
=  + 2x −  = 30 − 12 (–1,0) O
2 3  −1
= 18 sq units In first quadrant, the above given curves
4 8  1 1
=  + 4−  −  −2 +  intersect each other at (1, 2).
2 3 2 3 209 The region represented by
1 9 So, the required area
= 8− − | x − y | ≤ 2 and | x + y | ≤ 2 is bounded 1
2 3 = ∫ ((x + 1) − 2x ) dx
by a 0
1 1 9
= 8 − − 3 = 5 − = sq units  x2
1
[2019, 10 April Shift-I]
2x 
2 2 2
(a) rhombus of side length 2 units = + x− 
2 log e 2  0
(b) rhombus of area 8 2 sq units
208 The area (in sq units) of the region  ax 
Q∫ a dx =
(c) square of side length 2 2 units x
+ C
 y2  log
A =  (x, y) : ≤ x ≤ y + 4 is (d) square of area 16 sq units  e a 
 2  Ans. (c) 1 2 1 
=  + 1− + 
[2019, 9 April Shift-II] The given inequalities are 2 log e 2 log e 2 
53 | x − y | ≤ 2 and | x + y | ≤ 2.
(a) 30 (b) 3 1
3 = −
On drawing, the above inequalities, we 2 log e 2
(c) 16 (d) 18 get a square
y + y1
211 If the area (in sq units) of the region 212 If the area (in sq units) bounded by ⇒ = x x 1 − 1, where, x 1 = 2 and y 1 = 3.
2
{(x, y): y 2 ≤ 4x, x + y ≤ 1, x ≥ 0, y ≥ 0} is the parabola y 2 = 4λx and the line
a 2 + b, then a − b is equal to 1 y+3
y = λx, λ > 0, is , then λ is equal to ⇒ = 2x − 1 ⇒ y = 4x − 5
[2019, 12 April Shift-I] 9 2
10 [2019, 12 April Shift-II]
(a) (b) 6 y=x2–1
3 (a) 2 6 (b) 48 (c) 24 (d) 4 3 (2, 3)
8 2 Ans. (c)
(c) (d) −
3 3 Given, equation of curves are
Ans. (b) 2
y2 = 4λx …(i)
Given region is {(x, y) : y2 ≤ 4x, x + y ≤ 1, and y = λx …(ii) (0, –1)
x ≥ 0, y ≥ 0} λ>0 y=4x–5
Area bounded by above two curve is, as
B(0,1)
per figure
P Y
y2=4x A y2=4λx
x+y=1
Now, required area = area of shaded
O region
X X
O A(1,0) 2
= ∫ (y (parabola) − y (tangent)) dx
Now, for point P, put value of y = 1 − x to y=λx
0
2
y2 = 4x, we get = ∫ [(x2 − 1) − (4x − 5)] dx
0
(1 − x)2 = 4x the intersection point A we will get on 2 2
⇒ x + 1 − 2x = 4x
2
the solving Eqs. (i) and (ii), we get = ∫ (x2 − 4x + 4) dx = ∫ (x − 2)2 dx
0 0
⇒ x2 − 6x + 1 = 0 4 2
λ2 x2 = 4λx ⇒ x = , so y = 4 (x − 2) 3
6 ± 36 − 4 λ =
⇒ x= = 3 ± 2 2. 3
So, A  , 4
2 4 0

Since, x-coordinate of P less than λ  (2 − 2) 3 (0 − 2) 3


= −
x-coordinate of point A(1, 0). 4/ λ 3 3
∴ x =3−2 2 Now, required area = ∫ (2 λx − λx) dx 8
= sq units
0
4/ λ 4/ λ
3
Required are  x 3/ 2   x2 
3 −2 2 1 =2 λ   −λ 
=∫ 2 x dx + ∫3− 2 (1 − x) dx  3 /2  0  2 0 214 The area of the region
0 2

4 4 λ
2 A ={(x, y); 0 ≤ y ≤ x | x | + 1 and
3 −2 2

1 4  4  = 32 − 8
x 3/ 2 x2  = λ −   − 1 ≤ x ≤ 1} in sq. units, is
=2 + x −  3 λ λ 2  λ  3λ λ
3 /2 2 3 −2 [2019, 9 Jan. Shift-II]
0  2 32 − 24 8
= = 4 1 2
(a) 2 (b) (c) (d)
(3 − 2 2) 3/ 2 +  1 −  − (3 − 2 2)
4 1 3λ 3λ
= 3 3 3
3  2 1 8 1
It is given that area = ⇒ = Ans. (a)
(3 − 2 2)2 9 3λ 9
+ We have, A = {(x, y) : 0 ≤ y ≤ x | x | + 1 and
2 ⇒ λ = 24
− 1 ≤ x ≤ 1}
4 1 1
= [( 2 − 1) ] + − 3 + 2 2 +
2 3/ 2
When x ≥0, then0 ≤ y ≤ x2 + 1
3 2 2 213 The area (in sq units) bounded by
and when x < 0, then 0 ≤ y ≤ − x2 + 1
(9 + 8 − 12 2) the parabola y = x 2 − 1, the tangent
4 5 17 Now, the required region is the shaded
= ( 2 − 1) 3 − + 2 2 + −6 2 at the point (2, 3) to it and the region.
3 2 2 Y-axis is [2019, 9 Jan. Shift-I] y
4 8 56
= (2 2 − 3(2) + 3( 2) − 1) − 4 2 + 6 (a) (b)
3 3 3 2
4 32 14
= (5 2− 7) − 4 2 + 6 (c) (d)
y=–x +12
y=x2+1
3 3 3 1
8 2 10 Ans. (a)
= − x
3 3 Given, equation of parabola is y = x2 − 1, –1 1
=a 2+b (given) which can be rewritten as x2 = y + 1 or y=0
x2 = ( y − (−1) ).
8 10 [Qy = x + 1 ⇒x = (y − 1), parabola with
2 2
So, on comparing a = and b = − ⇒Vertex of parabola is (0, − 1) and it is
3 3
open upwar(d) vertex (0, 1) and y = − x2 + 1 ⇒x2 = − (y − 1)
8 10 parabola with vertex (0, 1) but open
∴ a −b = + =6 Equation of tangent at (2, 3) is given by
3 3 downward]
T =0
We need to calculate the shaded area, Now, equation of tangent at (2, 5) is The required area = area of shaded
which is equal to y+5 region
0 1 = 2x + 1 3 3
∫−1 (− x + 1)dx + ∫ (x2 + 1) dx 2 = ∫ ((x2 + 2) − (x + 1)) dx = ∫ (x2 − x + 1) dx
2
0 0 0
[Qequation of the tangent at (x 1, y 1) is
0 1 given by 3
 x3   x3   x 3 x2   27 9 
= − + x +  + x 1 =  − + x  =  − + 3 − 0
T = 0. Here, (y + y 1) = xx 1 + 1 ]
 3 − 1  3 0 2  3 2 0  3 2 
  (− 1) 3   1 ⇒ y = 4x − 3 9 9 15
 = 9 − + 3 = 12 − = sq units
= 0 − − + (− 1)   +   + 1 − 0 
    2 2 2
   3 
 3 y= 4x–3

Y
= −  − 1 + = + = 2
1 4 2 4 218 Let g (x) = cos x 2 , f (x) = x and
3  3 3 3
P (2, 5) α, β (α < β) be the roots of the
215 If the area enclosed between the quadratic equation
curves y = kx 2 and x = ky 2 , (k > 0), is 18x 2 − 9π x + π 2 = 0. Then, the area
1 square unit. Then, k is (0, 1) (in sq units) bounded by the curve
[2019, 10 Jan. Shift-I] R
X
y = (gof )(x) and the lines x = α, x = β
1 2 3
O Q (2, 0) and y = 0, is [JEE Main 2018]
(a) 3 (b) (c) (d) 3, 1 1
0
3 3 2 4 (a) ( 3 − 1) (b) ( 3 + 1)
2 2
Ans. (b) 1 1
(c) ( 3 − 2 ) (d) ( 2 − 1)
We know that, area of region bounded by 2 2
the parabolas x2 = 4ay and y2 = 4bx is Required area = Area of shaded region Ans. (a)
16 2
(ab) sq units. = ∫ y (parabola )dx − (Area of ∆PQR)
3 0
On comparing y = kx2 and x = ky2 with 2 We have,
= ∫ (x2 + 1) dx − (Area of ∆PQR)
above equations, we get 0 ⇒ 18x2 − 9 π x + π 2 = 0
1
4a = and 4b =
1 2 ⇒ 18x − 6π x − 3π x + π 2 = 0
2
 x3  1 3
k k =  + x  −  2 −  ⋅ 5 (6x − π )(3x − π ) = 0
1 1 3 0 2 4 π π
⇒ a = and b = ⇒ x= ,
4k 4k 1 6 3
[Qarea of a triangle = × base × height]
Now, α<β
∴ Area enclosed between y = kx2 and 2
π π
x = ky2 is 1  5 ∴ α = ,β =
=  + 2 − 0 −   5
8 6 3
16  1   1  1 3  2  4 g (x) = cos x2 and f (x) = x
  = Given,
3  4k   4k  3k2 y = gOf (x)
14 25 112 − 75 37
1 = − = = ∴ y = g (f (x)) = cos x
⇒ =1 3 8 24 24
3k2 Area of region bounded by x = α,
[given, area = 1 sq.unit] x = β, y = 0 and curve y = g (f (x)) is
1 1 217 The area (in sq units) of the region π/3
⇒ k2 = ⇒ k = ± bounded by the parabola, y = x 2 + 2 ⇒ A = [sin x] ππ // 36
3 3
A= ∫ cos xdx
1 and the lines, y = x + 1, x = 0 and π /6
⇒ k= [Qk > 0] π π
3 x = 3, is [2019, 12 Jan. Shift-I] A = sin − sin =
3 1

15 17 3 6 2 2
(a) (b)
216 The area (in sq units) in the first 2 4  3 − 1
A =  
2 
21 15
quadrant bounded by the parabola, (c) (d) 
2 4
y = x 2 + 1, the tangent to it at the Ans. (a) 219 The area (in sq units) of the region
point (2, 5) and the coordinate axes
Given equation of parabola is y = x2 + 2, {(x, y) : x ≥ 0, x + y ≤ 3, x 2 ≤ 4y and
is [2019, 11 Jan. Shift-II] and the line is y = x + 1
14 187
y ≤ 1 + x } is [JEE Main 2017]
(a) (b) y 59 3
3 24 y=x2 +2 (a) (b)
8 37 12 2
(c) (d) y=x+1
7 5
3 24 (c) (d)
(0,2) 3 2
Ans. (d)
Ans. (d)
Given, equation of parabola is y = x2 + 1, 1
Required area
which can be written as x2 = (y − 1).
x 1 2 2 x2
Clearly, vertex of parabola is (0, 1) and it 1 O (3,0) = ∫ (1 + x ) dx + ∫1 (3 − x) dx − ∫ dx
will open upwar(d)
0 0 4
Y
y=1+√x 221 The area (in sq units) of the region 222 The area of the region described by
(0, 3) (1, 2)
described by {(x, y) : y 2 ≤ 2x and A = {(x, y): x 2 + y 2 ≤ 1and y 2 ≤ 1 − x} is
4y=x 2
y ≥ 4x − 1} is [JEE Main 2015] π 4 π 4
(a) + (b) −
(0, 1) (2, 1) 7 2 3 2 3
(a) units π 2 π 2
x+y=3 32 (c) − (d) +
X′ X 5 2 3 2 3
(0, 0) (1, 0)(2, 0) (3, 0) (b) units [JEE Main 2014]
64
Y′ 15 Ans. (a)
(c) units
1 2 2 64 A = {(x, y) :x2 + y2 ≤ 1 and y2 ≤ 1 − x }
 x 3/ 2   x2   x3  Y
= x + 9
 + 3x −  −   (d) units
 3 /2  0  2  1  12  0 32
 1  8  Ans. (d)
=  1 +  +  6 − 2 − 3 +  −  
2
 3  2   12  Given region is {(x, y) : y2 ≤ 2x and
X′ X
5 3 2 3 5 y ≥ 4x − 1} (–1, 0) (0, 1)
= + − = 1 + = sq units
3 2 3 2 2 y2 ≤ 2x represents a region inside the
parabola
220 The area (in sq units) of the region y2 = 2x …(i)
Y′
{(x, y) : y 2 ≥ 2x and x 2 + y 2 ≤ 4x, x ≥ 0, and y ≥ 4x − 1 represents a region to the
1 1
left of the line Required area = πr2 + 2 ∫ (1 − y2 ) dy
y ≥ 0} is [JEE Main 2016] 2 0
4 8 y = 4x − 1 …(ii) 1
(a) π − (b) π − The point of intersection of the curve (i) 1  y3  π 4
3 3 = π ( 1) 2 + 2  y −  = +
and (ii) is 2  3 0 2 3
4 2 π 2 2
(c) π − (d) − (4x − 1)2 = 2x
3 2 3 ⇒ 16x2 + 1 − 8x = 2x
223 The area (in sq units) bounded by
Ans. (b) ⇒ 16x2 − 10 x + 1 = 0
1 1 the curves y = x , 2y − x + 3 = 0,
Given equations of curves are y2 = 2x ⇒ x= ,
2 8 X-axis and lying in the first
which is a parabola with vertex (0, 0) and quadrant is [JEE Main 2013]
axis parallel to X-axis. ...(i) ∴The points where these curves
1   1 1 (a) 9 (b) 36
And x2 + y2 = 4x intersect, are  , 1 and  , −  .
2   8 2 27
which is a circle with centre (2, 0) and (c) 18 (d)
4
radius = 2 ...(ii) y = 4x – 1
Y 1, Ans. (a)
On substituting y2 = 2x in Eq. (ii), we get 1
2 y2 = 2x Given curves are
x2 + 2x = 4x 1
⇒ x2 = 2x y= x …(i)
1
⇒ x = 0 or x = 2 2 and 2y − x + 3 = 0 …(ii)
–1 1
On solving Eqs. (i) and (ii), we get
⇒ y = 0 or y = ± 2 [using Eq. (i)] –1 2 0 2 1
X′ X 2 x − ( x )2 + 3 = 0
Now, the required area is the area of
shaded region, i.e. 1 , –1 ⇒ ( x )2 − 2 x − 3 = 0
–1 8 2
2 Y
Y
–1
y=√
x
A (2,2) 0
3=
2 2
x +y =4x Y′ – x+
2y
X′ X
X′ X Hence, required area 3
(0, 0) B (2,0) 1  y + 1 y2 
y2=2x = ∫  −  dy –3
2
−1 / 2
 4 2
1
Y′
Y′ 1  y2  1
=  + y  − (y 3) 1−1/ 2 ⇒ ( x − 3) ( x + 1) = 0
4 2  −1 / 2 6
Required area ⇒ x =3 [Q x = − 1 is not possible]
Area of circle 2 1 1   1 1 ∴ y =3
− ∫ 2x dx =  + 1 −  −   − 1 + 
= 1 1
4 4  2   8 2   6 
0 3
8 ∴ Required area = ∫ (x2 − x 1) dy
2
π(2)2 2  x 3/ 2  0
− 2 ∫ x 1/ 2dx = π − 2  =  +  −  
= 1 3 3 1 9 3
 3  y3 
4 0
 3 /2  0 4 2 8  6  8  = ∫ {(2y + 3) − y2 } dy =  y2 + 3y − 
0
 3 0
1 15 3 9
[2 2 − 0] =  π −  sq units
2 2 8 = × − =
=π −
3  3 4 8 16 32 =9+ 9−9=9
224 The area bounded between the Y 227 The area bounded by the curves
y y = cos x and y = sin x between the
parabolas x 2 = and x 2 = 9y and y=x
4 3π
ordinates x = 0 and x = , is
the straight line y = 2, is [AIEEE 2012] 2
10 2 [AIEEE 2009]
(a) 20 2 (b) A
3 (a) (4 2 − 2 ) sq units
20 2 B (b) (4 2 + 2 ) sq units
(c) (d) 10 2 y = 1/x
3 O
X (c) (4 2 − 1) sq units
D (1, 0) C (e, 0)
Ans. (c)
x=e (d) (4 2 + 1) sq units
y
Given Two parabolas x2 = and x2 = 9y Ans. (a)
4 ∴ Area to be calculated in I quadrant
To find The area bounded between the shown as
parabolas and the straight line y = 2. The ∴ Required area = Area of ∆ ODA + Area
required area is equal to the shaded of ABCD 3π/2
π
1 e 1 1
region in the drawn figure. = (1 × 1) + ∫ dx = + [log | x |] 1e O π/2
2 1 x 2
Y
y = 4x 2 1
1
y = x2 = + {log | e | − log 1}
9 2
y=2 1 3 ∴Required area
= + 1 = sq units
2 2 π /4
=∫ (cos x − sin x) dx
0
5π / 4
226 The area bounded by the curves
X y 2 = 4x and x 2 = 4y is [AIEEE 2011]
+ ∫π / 4 (sin x − cos x) dx
3π / 2
(a) 0 (b)
32 +∫ (cos x − sin x) dx
5π / 4
3
The area of the shaded region (which can 16 8 = [sin x + cos x] π0 / 4 + [− cos x − sin x [5π π/ 4/ 4
(c) (d)
be very easily found by using integration) 3 3 + [sin x + cos x] 35ππ //24
is twice the area shaded in first Ans. (c)
 1 1   1 1   −1 1 
quadrant. = + − 1 +   +  − − 
2  
For the point of intersection, solve  2 2   2 2  2
2 y y2 = 4x
∴Required area = 2 ∫  3 y −  dy
0
 2  and x2 = 4y.
2   −1 1 
 x2  +  (− 1 + 0) −  − 
5  ⇒   = 4x ⇒x 4 = 43 x ⇒x = 0, 4 2  
2
=2∫  y  dy   2
0 2  4
8
2 Y = − 2 = (4 2 − 2) sq units
 y 3/ 2  2 2
= 5   x = 4y y2= 4x
 3 /2  0
10 3/ 2
(4, 4) 228 The area of the region bounded by
= (2 − 0)
3 the parabola ( y − 2) 2 = x − 1, the
20 2 tangent to the parabola at the point
= X′ X
3 (0, 0) D (4, 0) (2 , 3) and the X-axis is [AIEEE 2009]
(a) 6 sq units (b) 9 sq units
225 The area of the region enclosed by (c) 12 sq units (d) 3 sq units
1
the curves y = x, x = e, y = and the Ans. (b)
x Y′ The equation of tangent at (2, 3) to the
positive X-axis is [AIEEE 2011] ∴ Area bounded between curves given parabola is
3  
4
x = 2y − 4
(a) 1 sq unit (b) sq units
2 4 x2  x 3/ 2
x 3
Y
= ∫  4x −  dx = 2⋅ − 
5 1 (2, 3)
 4  3 12 
0
(c) sq units (d) sq unit
2 2  2  0
Ans. (b) 4 (4) 3
= ⋅ (4) 3/ 2 −
1 3 12
Given, y = x, x = e and y = , x ≥ 0
x 32 16 16 X′ X
= − = (– 4, 0)
Since, y = x and x ≥ 0 ⇒ y ≥ 0 3 3 3
Y′ (y – 2)2 = (x – 1)
∴ Required area Ans. (c) (a) 4 sq units (b) 3 sq units
3 1
= ∫ {( y − 2) + 1 − 2y + 4}dy
2
A = ∫ ( x − x) dx
Required area, (c) 2 sq units (d) 1 sq unit
0 0
Ans. (d)
3
 ( y − 2) 3  Y 0
= − y2 + 5 y  Required area, A = ∫ log e (x + e) dx
3 y=|x| 1− e
 0
1 8 y2 = x Y x2 = 4y
= − 9 + 15 +
3 3
= 9 sq units y=4 y2 = 4x
X′ X S1
O (1, 0)
S2
229 The area of the plane region X′
S3
X
bounded by the curves x + 2y 2 = 0 O x=4
Y′
and x + 3y 2 = 1 is equal to 1
[AIEEE 2008] 2 x2  2 1 1
=  x 3/ 2 −  = − = sq unit
4 5
(a) sq units (b) sq units 3 2 0 3 2 6
3 3 Y′
1
(c) sq unit
2
(d) sq unit 231 If f (x) is a non-negative continuous Put x + e = t ⇒ dx = dt
e
3 3 function such that the area bounded ∴ A = ∫ log e t dt = [t log e t − t] e1
1
Ans. (a) by the curve y = f (x), X-axis and the
π π = (e − e − 0 + 1) = 1 sq unit
Given curves are x + 3y2 = 1 …(i)
ordinates x = and x = β > is
and x + 2y2 = 0 …(ii) 4 4 233 The parabolas y 2 = 4x and x 2 = 4y
On solving Eqs. (i) and (ii), we get β sin β + π cos β + 2 β . Then,
  divide the square region bounded
y = ± 1 and x = −2  4 
π by the lines x = 4, y = 4 and the
f   is equal to
Y
(–2, 1)  2 coordinate axes. If S 1 , S 2 and S 3 are
[AIEEE 2005]
x + 3y 2 = 1
respectively the areas of these parts
x + 2y 2 = 0  π   π 
(a) 1 − + 2  (b) 1 − − 2  numbered from top to bottom, then
 4   4 
X′ X S 1 : S 2 : S 3 is equal to [AIEEE 2005]
π  π 
(c)  − 2 + 1 (d)  + 2 − 1
(–2, –1) 4  4  (a) 1 : 1 : 1 (b) 2 : 1 : 2
(c) 1 : 2 : 3 (d) 1 : 2 : 1
Ans. (a)
Ans. (a)
Y′ According to the given condition,
β π It is clear from the figure that
∴ Required area =
 (x 1 − x2 ) dy
1

 1−


∫ π / 4 f (x) dx = β sin β + 4 cos β + 2 β 4
S 1 = S 3 = ∫ y dx
0
 1 (1 − 3y2 + 2y2 ) dy  On differentiating both sides w.r.t.β, we
=
4

 ∫−1
get x2
4 1  x3 
 =∫ dx =  
π 0 4 4  3 0
 1
 f(β) = sin β + β cos β − sin β + 2
= (1 − y ) dy
 ∫−1
2
4
 1
 π π π ⇒ S 1 = S 3 = × 64
2 (1 − y ) dy
1
 ∴ f   = 1+ 0− + 2= 1− + 2 12
= 2
 ∫0
2
4 4
 16
= sq units …(i)
[since, integral is an even] 3
1
232 The area enclosed between the 4
 y3  curve y = log e (x + e) and the and S2 + S 3 = ∫ 4x dx
= 2 y −  0

 3 0 coordinate axes is [AIEEE 2005] 4


 x 3/ 2 
=2 
2  1 − 1  
=  4 x = –e
  = sq units  3 /2  0
  3   3 Y
4
(0, 1) =
×8
3
230 The area enclosed between the
32 16
curves y 2 = x and y = | x | is X′ X
⇒ S2 = − [from Eq. (i)]
1–e O 3 3
[AIEEE 2007]
16
2 ⇒ S2 = sq units
(a) sq unit (b) 1 sq unit 3
3
16 16 16
1
(c) sq unit (d)
1
sq unit Y′ ∴ S 1 : S2 : S 3 = : : = 1 : 1 : 1
6 3 3 3 3
2 3
234 The area of the region bounded by = ∫ (2 − x) dx + ∫2 (x − 2) dx Y
1 y=
the curves y = | x − 2|, x = 1, x = 3 and –x
2 3 C (0, 3) 1
the X-axis is [AIEEE 2004]  x2   x2  + x–
= 2x −  +  − 2x  1
y=
(a) 1 sq unit 2 2 (–1, 2) D
 1  2
(b) 2 sq units B (2, 1)
  1   9 
(c) 3 sq units = 4 − 2 −  2 −   + − 6 − (2 − 4)
(d) 4 sq units   2    2  X′
O A y= X

x
+
3 3 (1, 0) 3
=2− − + 2

3
Ans. (a) –
x

y=
3 2 2
Required area = ∫ ydx
1 = 1 sq unit Y′
3
= ∫ | x − 2 | dx On solving, we get
1 235 The area of the region bounded by
y = x − 1 and y = 3 − x
the curves y = | x − 1| and y = 3 − | x | is
Y ⇒ x − 1=3− x ⇒ x =2
[AIEEE 2003]
and y =3−2 ⇒ y = 1
(a) 2 sq units
(b) 3 sq units Now, AB2 = (2 − 1)2 + (1 − 0)2
(c) 4 sq units = 1+ 1=2
X′ X
O (d) 6 sq units ⇒ AB = 2

x=1 x=3 Ans. (c) and BC2 = (0 − 2)2 + (3 − 1)2


 x − 1, x > 1 =4+ 4= 8
Since, y = | x − 1 | = 
Y′ − x + 1, x ≤ 1 ⇒ BC = 2 2
2 3 3 + x, x ≤ 0 ∴Area of rectangle ABCD = AB × BC
= ∫ − (x − 2) dx + and y = 3 − | x | = 
1 ∫2 (x − 2) dx 3 − x, x > 0 = 2 × 2 2 = 4 sq units

You might also like